You are on page 1of 211

1 电路的基本概念与定律

1.5 电源有载工作、开路与短路

1.5.1

在图1中,五个元件代表电源和负载。电流和电压的参考方向如图中所示。
今通过实验测量得知

图 1: 习题1.5.1图

I1 = − 4A I2 = 6A I3 = 10A
U1 = 140V U2 = − U3 = 60V
U4 = − 90V U5 =
30V
1 试标出各电流的实际方向和各电压的实际极性。
80V
2 判断哪些元件是电源?哪些是负载?
3 计算各元件的功率,电源发出的功率和负载取用的功率是否平衡?
[解]:
2 元件1,2为电源;3,4,5为负载。
3 P1 = U1 I1 = 140 × (− 4)W = − 560W 电源发出功率 PE =
P2 = U2 I2 = (− 90) × 6W = − 540W
P3 = U3 I3 = 60 × 10W = 600W
P4 = U4 I1 = (− 80) × (− 4)W =
320W P5 = U5 I2 = 30 × 6W = 180W

P1 + P2 = 1100W
负载取用功率 P = P3 + P4 + P5 = 1100W
两者平衡

1.5.2

在图2中,已知I1 = 3mA,I2 = 1mA.试确定电路元件3中的电流I3 和其两端


电压U3 ,并说明它是电源还是负载。校验整个电路的功率是否平衡。

1
[解] 首先根据基尔霍夫电流定律列出

图 2: 习题1.5.2图

− I1 + I2 − I3 = 0
− 3 + 1 − I3 = 0

可求得I3 = − 2mA, I3 的实际方向与图中的参考方向相反。


根据基尔霍夫电流定律可得

U3 = (30 + 10 × 103 × 3 × 10−3 )V = 60V

其次确定电源还是负载:
1 从电压和电流的实际方向判定:

电路元件3 电流I3 从“+”端流出,故为电源;


80V元件 电流I2 从“+”端流出,故为电源;

30V元件 电流I1 从“+”端流出,故为负载。

2 从电压和电流的参考方向判别:
电路元件3 U3 和I3 的参考方向相同P = U3 I3 = 60 × (− 2) × 10−3 W =
− 120 × 10−3 W (负值),故为电源;
80V元 件 U2 和I2 的 参 考 方 向 相 反P = U2 I2 = 80 × 1 × 10−3 W =
80 × 10−3 W (正值),故为电源;
30V元件 U1 和I1 参考方向相同P = U1 I1 = 30 × 3 × 10−3 W = 90 ×
10−3 W (正值),故为负载。

两者结果一致。 最后
校验功率平衡: 电阻
消耗功率:

PR1 = R1 I12 = 10 × 3 2mW = 90mW


PR2 = R2 I22 = 20 × 1 2mW = 20mW

2
电源发出功率:

PE = U2 I2 + U3 I3 = (80 + 120)mW = 200mW

负载取用和电阻损耗功率:

P = U1 I1 + R1 I12 + R2 I22 = (90 + 90 + 20)mW = 200mW

两者平衡

1.5.3

有一直流电源,其额定功率PN = 200W ,额定电压UN = 50V 。内阻R0 =


0.5Ω,负载电阻R可以调节。其电路如教材图1.5.1所示试求:
1 额定工作状态下的电流及负载电阻;
2 开路状态下的电源端电压;
3 电源短路状态下的电流。
[解]
PN 200 U 50
(1) 额定电流IN = = A = 4A, 负载电阻R = N = Ω = 12.5Ω
UN 50 IN 4

(2) 电源开路电压U0 = E = UN + IN R0 = (50 + 4 × 0.5)V = 52V

E 52
(3) 电源短路电流IS = = A = 104A
R0 0.5

1.5.4

有一台直流稳压电源,其额定输出电压为30V ,额定输出电流为2A,从空载
到额定负载,其输出电压的变化率为千分之一
U0 − UN = 0.1%),试求该电源的内阻。
(即∆U =
UN
[解] 电源空载电压U0 即为其电动势E,故可先求出U0 ,而后由U = E − R0 I ,求
内阻R0 。

U0 − UN
= ∆U
UN
U0 − 30
= 0.1 %
30

由此得

U0 = E = 30.03V

3
再由

U = E − R0 I
30 = 30.03 − R0 × 2

得出

R0 = 0.015Ω

1.5.6

一只110V 、8W 的指示灯,现在要接在380V 的电源上,问要串多大阻值的


电阻?该电阻应选多大瓦数的?
[解] 由指示灯的额定值求额定状态下的电流IN 和电阻RN :
PN 8 UN 110
IN = = A = 0.073A RN = = Ω = 1507Ω
UN 110 IN 0.073
在380V 电源上指示灯仍保持110V 额定电压,所串电阻
U − UN 380 − 110
R = = Ω = 3700Ω
IN 0.073
其额定功率

PN = RIN2 = 3700 × (0.073)2 W = 19.6W

故可选用额定值为3.7K Ω、20W 的电阻。

1.5.8

图3所示的是用变阻器R调节直流电机励磁电流If 的电路。设电机励磁绕组
的电阻为315Ω,其额定电压为220V ,如果要求励磁电流在0.35 ∼ 0.7A的范围内变
动,试在下列三个变阻器中选用一个合适的:
(1) 1000Ω、0.5A;(2) 200Ω、1A;(3) 350Ω、1A。
[解]
当R = 0时
220
I= = 0.7A
315
当I = 0.35A时
220
R + 315 = = 630Ω
0.35
R = (630 − 315) = 315Ω

因此,只能选用350Ω、1A的变阻器。

4
图 3: 习题1.5.8图

1.5.11

图4所示的是电阻应变仪中测量电桥的原理电路。Rx 是电阻应变片,粘附
在被测零件上。当零件发生变形(伸长或缩短)时,Rx 的阻值随之而改变,这
反映在输出信号Uo 上。在测量前如果把各个电阻调节到Rx = 100Ω,R1 = R2 =
Rx R1
200Ω,R3 = 100Ω,这时满足 = 的电桥平衡条件,Uo = 0。在进行测量
R3 R2
时,如果测出:
(1) Uo = +1mV ;(2) Uo = − 1mV ;试计算两种情况下的∆Rx 。Uo 极性的改
变反映了什么?设电源电压U 是直流3V 。
[解] (1) Uo = +1mV

图 4: 习题1.5.11图

应用基尔霍夫电压定律可列出:

Uab + Ubd + Uda = 0


Uab + Uo − Uad = 0

U U
Rx + Uo − = 0
Rx + R3 2
3Rx + 0.001 − 1.5 = 0
Rx + 100

5
解之得

Rx = 99.867 Ω

因零件缩短而使Rx 阻值减小,即

∆Rx = (99.867 − 100)Ω = − 0.133 Ω

(2) Uo = − 1mV
同理
3Rx
− 0.001 − 1.5 = 0
Rx + 100

Rx = 100.133 Ω

因零件伸长而使Rx 阻值增大,即

∆Rx = (100.133 − 100) Ω = +0.133 Ω

Uo 极性的变化反映了零件的伸长和缩短。

1.5.12

图5是电源有载工作的电路。电源的电动势E = 220V ,内阻R0 = 0.2Ω;负


载电阻R1 = 10Ω,R2 = 6.67Ω;线路电阻Rl = 0.1Ω。试求负载电阻R2 并联前
后:(1)电路中电流I ;(2)电源端电压U1 和负载端电压U2 ;(3)负载功率P 。当负载
增大时,总的负载电阻、线路中电流、负载功率、电源端和负载端的电压是如
何变化的?
[解] R2 并联前,电路总电阻

图 5: 习题1.5.12图

R = R0 + 2Rl + R1 = (0.2 + 2 × 0.1 + 10) Ω = 10.4 Ω

(1) 电路中电流

E 220
I= = A = 21.2A
R 10.4

6
(2) 电源端电压

U1 = E − R0 I = (220 − 0.2 × 21.2)V = 216V

负载端电压

U2 = R1 I = 10 × 21.2V = 212V

(3) 负载功率

P = U2 I = 212 × 21.2W = 4490W = 4.49kW

R2 并联后,电路总电阻
R1 R2 10 × 6.67
R = R0 + 2Rl + = (0.2 + 2 × 0.1 + )Ω = 4.4 Ω
R1 + R2 10 + 6.67
(1) 电路中电流
E 220
I= = A = 50A
R 4.4
(2) 电源端电压

U1 = E − R0 I = (220 − 0.2 × 50)V = 210V

负载端电压
R1 R2 10 × 6.67
U2 = I= × 50V = 200V
R1 + R2 10 + 6.67
(3) 负载功率

P = U2 I = 200 × 50W = 10000W = 10kW

可见,当负载增大后,电路总电阻减小,电路中电流增大,负载功率增大,电
源端电压和负载端电压均降低。

1.6 基尔霍夫定律

1.6.2

试求图6所示部分电路中电流I 、I1 和电阻R,设Uab = 0。


[解] 由基尔霍夫电流定律可知,I = 6A。
由于设Uab = 0,可得

I1 = − 1A
6
I2 = I3 = A = 3A
2

7
图 6: 习题1.6.2图

并得出

I4 = I1 + I3 = (− 1 + 3)A = 2A
I5 = I − I4 = (6 − 2)A = 4A

I5 R = I 4 × 1

I4 2
R = = Ω = 0.5Ω
I5 4

1.7 电路中电位的概念及计算
1.7.4

在图7中,求A点电位VA 。
[解]

图 7: 习题1.7.4图

8
I1 − I2 − I3 = 0 (1)
50 − VA
I1 = (2)
10
VA − (− 50)
I2 = (3)
5
VA
I3 = (4)
20
将式(2)、(3)、(4)代入式(1),得

50 − VA VA + 50 VA
− − =0
10 5 20
VA = − 14.3V

9
10
目录
第2章 电路的分析方法 3
第2.1节 电阻串并联接的等效变换 . . . . . . . . . . . . . . . . . . . . . 3
第2.1.1题 . . . . . . . . . . . . . . . . . . . . . . . . . . . . . . . 3第
2.1.2题 . . . . . . . . . . . . . . . . . . . . . . . . . . . . . . . 4第
2.1.3题 . . . . . . . . . . . . . . . . . . . . . . . . . . . . . . . 4第
2.1.5题 . . . . . . . . . . . . . . . . . . . . . . . . . . . . . . . 5第
2.1.6题 . . . . . . . . . . . . . . . . . . . . . . . . . . . . . . . 6第
2.1.7题 . . . . . . . . . . . . . . . . . . . . . . . . . . . . . . . 6第
2.1.8题 . . . . . . . . . . . . . . . . . . . . . . . . . . . . . . . 7 第2.3
节 电源的两种模型及其等效变换 . . . . . . . . . . . . . . . . . . 8第
2.3.1题 . . . . . . . . . . . . . . . . . . . . . . . . . . . . . . . 8第
2.3.2题 . . . . . . . . . . . . . . . . . . . . . . . . . . . . . . . 9第
2.3.4题 . . . . . . . . . . . . . . . . . . . . . . . . . . . . . . . 9 第2.4
节 支路电流法 . . . . . . . . . . . . . . . . . . . . . . . . . . . . 10
第2.4.1题 . . . . . . . . . . . . . . . . . . . . . . . . . . . . . . . 10 第
2.4.2题 . . . . . . . . . . . . . . . . . . . . . . . . . . . . . . . 11 第2.5
节 结点电压法 . . . . . . . . . . . . . . . . . . . . . . . . . . . . 12
第2.5.1题 . . . . . . . . . . . . . . . . . . . . . . . . . . . . . . . 12 第
2.5.2题 . . . . . . . . . . . . . . . . . . . . . . . . . . . . . . . 13 第
2.5.3题 . . . . . . . . . . . . . . . . . . . . . . . . . . . . . . . 14 第2.6
节 叠加定理 . . . . . . . . . . . . . . . . . . . . . . . . . . . . . . 14 第
2.6.1题 . . . . . . . . . . . . . . . . . . . . . . . . . . . . . . . 14 第
2.6.2题 . . . . . . . . . . . . . . . . . . . . . . . . . . . . . . . 15 第
2.6.3题 . . . . . . . . . . . . . . . . . . . . . . . . . . . . . . . 16 第
2.6.4题 . . . . . . . . . . . . . . . . . . . . . . . . . . . . . . . 18 第2.7
节 戴维南定理与诺顿定理 . . . . . . . . . . . . . . . . . . . . . . 19
第2.7.1题 . . . . . . . . . . . . . . . . . . . . . . . . . . . . . . . 19
第2.7.2题 . . . . . . . . . . . . . . . . . . . . . . . . . . . . . . . 19
第2.7.5题 . . . . . . . . . . . . . . . . . . . . . . . . . . . . . . . 20
第2.7.7题 . . . . . . . . . . . . . . . . . . . . . . . . . . . . . . . 21
第2.7.8题 . . . . . . . . . . . . . . . . . . . . . . . . . . . . . . . 22
第2.7.9题 . . . . . . . . . . . . . . . . . . . . . . . . . . . . . . . 22
第2.7.10题 . . . . . . . . . . . . . . . . . . . . . . . . . . . . . . 23
第2.7.11题 . . . . . . . . . . . . . . . . . . . . . . . . . . . . . . 24

1
List of Figures
1 习题2.1.1图 . . . . . . . . . . . . . . . . . . . . . . . . . . . . . . . . 3
2 习题2.1.2图 . . . . . . . . . . . . . . . . . . . . . . . . . . . . . . . . 4
3 习题2.1.3图 . . . . . . . . . . . . . . . . . . . . . . . . . . . . . . . . 4
4 习题2.1.5图 . . . . . . . . . . . . . . . . . . . . . . . . . . . . . . . . 6
5 习题2.1.7图 . . . . . . . . . . . . . . . . . . . . . . . . . . . . . . . . 7
6 习题2.1.8图 . . . . . . . . . . . . . . . . . . . . . . . . . . . . . . . . 7
7 习题2.3.1图 . . . . . . . . . . . . . . . . . . . . . . . . . . . . . . . . 8
8 习题2.3.2图 . . . . . . . . . . . . . . . . . . . . . . . . . . . . . . . . 9
9 习题2.3.4图 . . . . . . . . . . . . . . . . . . . . . . . . . . . . . . . . 9
10 习题2.4.1图 . . . . . . . . . . . . . . . . . . . . . . . . . . . . . . . . 10
11 习题2.4.2图 . . . . . . . . . . . . . . . . . . . . . . . . . . . . . . . . 11
12 习题2.5.1图 . . . . . . . . . . . . . . . . . . . . . . . . . . . . . . . . 13
13 习题2.5.2图 . . . . . . . . . . . . . . . . . . . . . . . . . . . . . . . . 13
14 习题2.5.3图 . . . . . . . . . . . . . . . . . . . . . . . . . . . . . . . . 14
15 习题2.6.1图 . . . . . . . . . . . . . . . . . . . . . . . . . . . . . . . . 15
16 习题2.6.2图 . . . . . . . . . . . . . . . . . . . . . . . . . . . . . . . . 16
17 习题2.6.3图 . . . . . . . . . . . . . . . . . . . . . . . . . . . . . . . . 17
18 习题2.6.4图 . . . . . . . . . . . . . . . . . . . . . . . . . . . . . . . . 18
19 习题2.6.4图 . . . . . . . . . . . . . . . . . . . . . . . . . . . . . . . . 18
20 习题2.7.1图 . . . . . . . . . . . . . . . . . . . . . . . . . . . . . . . . 19
21 习题2.7.2图 . . . . . . . . . . . . . . . . . . . . . . . . . . . . . . . . 20
22 习题2.7.5图 . . . . . . . . . . . . . . . . . . . . . . . . . . . . . . . . 20
23 习题2.7.7图 . . . . . . . . . . . . . . . . . . . . . . . . . . . . . . . . 21
24 习题2.7.8图 . . . . . . . . . . . . . . . . . . . . . . . . . . . . . . . . 22
25 习题2.7.9图 . . . . . . . . . . . . . . . . . . . . . . . . . . . . . . . . 23
26 习题2.7.10图 . . . . . . . . . . . . . . . . . . . . . . . . . . . . . . . 23
27 习题2.7.11图 . . . . . . . . . . . . . . . . . . . . . . . . . . . . . . . 24

2
2 电路的分析方法

2.1 电阻串并联接的等效变换

2.1.1

在 图1所 示 的 电 路 中 ,E = 6V ,R1 = 6Ω,R2 = 3Ω,R3 = 4Ω,R4 =


3Ω,R5 = 1Ω,试求I3 和I4 。
[解]

图 1: 习题2.1.1图

本 题 通 过 电 阻 的 串 联 和 并 联 可 化 为 单 回 路 电 路 计 算 。R1 和R4 并 联 而 后
与R3 串联,得出的等效电阻R1,3,4 和R2 并联,最后与电源及R5 组成单回路电路,
于是得出电源中电流

E
I=
R1 R4
R2 (R3 + )
R1 + R4
R5 +
R1 R4
R2 + (R3 + )
R1 + R4
6
= = 2A
6×3
3 × (4 + )
1+ 6+3
6×3
3 + (4 + )
6+3

而后应用分流公式得出I3 和I4

R2 3 2
I3 = I=
6×3
× 2A = A
R1 R4 3
R2 + R3 + 3+4+
R1 + R4 6+3
R1 6 2 4
I4 = − I3 = − × A =− A
R1 + R4 6+3 3 9

I4 的实际方向与图中的参考方向相反。

3
2.1.2

有 一 无 源 二 端 电 阻 网 络[图2(a)], 通 过 实 验 测 得 : 当U = 10V 时 ,I =
2A;并已知该电阻网络由四个3Ω的电阻构成,试问这四个电阻是如何连接的?
[解]

图 2: 习题2.1.2图

按题意,总电阻为

U 10
R = = Ω = 5Ω
I 2

四个3Ω电阻的连接方法如图2(b)所示。

2.1.3

在图3中,R1 = R2 = R3 = R4 = 300Ω,R5 = 600Ω,试求开关S断开和闭和


时a和b之间的等效电阻。
[解]

图 3: 习题2.1.3图 当开关S断开时,R1 与R3 串联

后与R5 并联,R2 与R4 串联后也与R5 并联,故

4

Rab = R5 //(R1 + R3 )//(R2 + R4 )


1
=
1 1 1
+ +
600 300 + 300 300 + 300
= 200 Ω

当S闭合时,则有

Rab = [(R1 //R2 ) + (R3 //R4 )]//R5


1
=
1 1
+
R5 R1 R2 R3 R4
+
R1 + R2 R3 + R4
1
= 1
1
+
600 300 × 300 300 × 300
+
300 + 300 300 + 300
= 200 Ω

2.1.5

[图4(a)]所示是一衰减电路,共有四挡。当输入电压U1 = 16V 时,试计算各


挡输出电压U2 。
[解]
a挡: U2a = U1 = 16V
b挡: 由末级看,先求等效电阻R [见图4(d)和(c)]
0

0 (45 + 5) × 5.5 275


R = Ω= Ω = 5Ω
(45 + 5) + 5.5 55.5
同样可得
00
R = 5 Ω。
于是由图4(b)可求U2b ,即
U1 16
U2b = ×5= × 5V = 1.6V
45 + 5 50
c挡:由图4(c)可求U2c ,即
U2b 1.6
U2c = ×5= × 5V = 0.16V
45 + 5 50
d挡:由图4(d)可求U2d ,即
U2c 0.16
U2d = ×5= × 5V = 0.016V
45 + 5 50
5
图 4: 习题2.1.5图

2.1.6

下图所示电路是由电位器组成的分压电路,电位器的电阻RP = 270 Ω,两


边 的 串 联 电 阻R1 = 350 Ω,R2 = 550 Ω。 设 输 入 电 压U1 = 12V , 试 求 输 出 电
压U2 的变化范围。
[解]
当箭头位于RP 最下端时,U2 取最小值
R2
U2min = U
R1 + R2 + RP 1
550 × 12
=
350 + 550 + 270
= 5.64V
当箭头位于RP 最上端时,U2 取最大值
R2 + RP
U2max = U
R1 + R2 + RP 1
550 + 270 × 12
=
350 + 550 + 270
= 8.41V
由此可得U2 的变化范围是:5.64 ∼ 8.41V 。

2.1.7

试用两个6V 的直流电源、两个1kΩ的电阻和一个10kΩ的电位器连接成调压范
围为− 5V ∼ +5V 的调压电路。

6
[解]

图 5: 习题2.1.7图

所联调压电路如图5所示。
6 − (− 6) = 1 × 10−3 A = 1mA
I=
(1 + 10 + 1) × 103
当滑动触头移在a点

U = [(10 + 1) × 103 × 1 × 10−3 − 6]V = 5V

当滑动触头移在b点

U = (1 × 103 × 1 × 10−3 − 6)V = − 5V

2.1.8

在图6所示的电路中,RP 1 和RP 2 是同轴电位器,试问当活动触点 a,b 移到最


左端、最右端和中间位置时,输出电压Uab 各为多少伏?
[解]

图 6: 习题2.1.8图

同轴电位器的两个电位器RP 1 和RP 2 的活动触点固定在同一转轴上,转动转


轴时两个活动触点同时左移或右移。当活动触点a,b在最左端时,a点接电源
正极,b点接负极,故Uab = E = +6V ;当活动触点在最右端时,a点接电源负
极,b点接正极,故Uab = − E = − 6V ;当两个活动触点在中间位置时,a,b两
点电位相等,故Uab = 0。

7
2.3 电源的两种模型及其等效变换
2.3.1

在图7中,求各理想电流源的端电压、功率及各电阻上消耗的功率。
[解]

图 7: 习题2.3.1图

设流过电阻R1 的电流为I3

I3 = I2 − I1 = (2 − 1)A = 1A

(1) 理想电流源1

U1 = R1 I3 = 20 × 1V = 20V
P1 = U1 I1 = 20 × 1W = 20W (取用)

因为电流从“+”端流入,故为负载。
(2) 理想电流源2

U2 = R1 I3 + R2 I2 = (20 × 1 + 10 × 2)V = 40V


P2 = U2 I2 = 40 × 2W = 80W (发出)

因为电流从“+”端流出,故为电源。
(3) 电阻R1

PR1 = R1 I32 = 20 × 12 W = 20W

(4) 电阻R2

PR2 = R2 I 2 = 10 × 22 W = 40W
2

校验功率平衡:

80W = 20W + 20W + 40W

8
图 8: 习题2.3.2图

2.3.2

计算图8(a)中的电流I3 。
[解]
计算本题应用电压源与电流源等效变换最为方便,变换后的电路如图8(b)所
示。由此得

2+1 3
I = A= A = 1.2A
1 + 0.5 + 1 2.5
1.2
I3 = A = 0.6A
2
2.3.4

计算图9中的电压U5 。
[解]

图 9: 习题2.3.4图

R2 R3 6×4
R1,2,3 = R1 + = (0.6 + )Ω = 3Ω
R2 + R3 6+4
将U1 和R1,2,3 与U4 和R4 都化为电流源,如图9(a)所示。

9
将图9(a)化简为图9(b)所示。其中

IS = IS1 + IS2 = (5 + 10)A = 15A


R1,2,3 R4 3 × 0.2 3
R0 = = Ω= Ω
R1,2,3 + R4 3 + 0.2 16
3
R0 45
I5 = I = 16 × 15A = A
R0 + R5 S 3 19
+1
16
45
U5 = R5 I5 = 1 × V = 2.37V
19

2.4 支路电流法

2.4.1

图10是两台发电机并联运行的电路。已知E1 = 230V ,R01 = 0.5 Ω,E2 =


226V ,R02 = 0.3 Ω,负载电阻RL = 5.5 Ω,试分别用支路电流法和结点电压法
求各支路电流。
[解]

图 10: 习题2.4.1图

10
(1) 用支路电流法

I1 + I2 = IL
E1 = R01 I1 + RL IL
E2 = R02 I2 + RL IL

将已知数代入并解之,得
I1 = 20A, I2 = 20A, IL = 40A
(2) 用结点电压法

E1 E2 230 226
+ +
R01 R02 0.5 0.3 V = 220V
U = =
1 1 1 1 1 1
+ + + +
R01 R02 RL 0.5 0.3 5.5
E1 − 230 − 220
I1 = U = A = 20A
0.5
R01
226 − 220
I2 = E2 − = A = 20A
0.3
U
R02
U 220
IL = = A = 40A
RL 5.5

2.4.2

试 用 支 路 电 流 法 和 结 点 电 压 法 求 图11所 示 电 路 中 的 各 支 路 电 流 , 并 求
三 个 电 源 的 输 出 功 率 和 负 载 电 阻RL 取 用 的 功 率 。 两 个 电 压 源 的 内 阻 分 别
为0.8 Ω和0.4 Ω。
[解]

图 11: 习题2.4.2图

(1) 用支路电流法计算 本题中有四个支路电流,其中一个是已知的,故列出


三个方程即可,即
120 − 0.8I1 + 0.4I2 − 116 = 0
120 − 0.8I1 − 4I11= 0
I1 + I2 + 10 − I = 0
解之,得

I1 = 9.38A
I2 = 8.75A
I = 28.13A

(2) 用结点电压法计算
120 116
+ + 10
Uab = 0.8
1
0.4
1 1 V = 112.5V
+ +
0.8 0.4 4
而后按各支路电流的参考方向应用有源电路的欧姆定律可求得

120 − 112.5
I1 = A = 9.38A
0.8
116 − 112.5
I2 = A = 8.75A
0.4
Uab 112.5
I = = A = 28.13A
RL 4

(3) 计算功率
三个电源的输出功率分别为

P1 = 112.5 × 9.38W = 1055W


P2 = 112.5 × 8.75W = 984W
P3 = 112.5 × 10W = 1125W
P1 + P2 + P3 = (1055 + 984 + 1125)W = 3164W

负载电阻RL 取用的功率为
P = 112.5 × 28.13W = 3164W
两者平衡。

2.5 结点电压法

2.5.1

试用结点电压法求图12所示电路中的各支路电流。
[解]

12
图 12: 习题2.5.1图

25 100 25
+ +
UO 0 O = 50 50 50 V = 50V
1 1 1
+ +
50 50 50
25 − 50
Ia = A = −0.5A
50
100 − 50
Ib = A = 1A
50
25 − 50
Ic = A = −0.5A
50

Ia 和Ic 的实际方向与图中的参考方向相反。

2.5.2

用结点电压法计算图13所示电路中A点的电位。

[解]

图 13: 习题2.5.2图

13
50 − 50
+
VA = 110 1 5 1 V = − 14.3V
+ +
50 5 20

2.5.3

电路如图14(a)所示,试用结点电压法求电阻RL 上的电压U ,并计算理想电流


源的功率。
[解]

图 14: 习题2.5.3图

将与4A理想电流源串联的电阻除去(短接)和与16V 理想电压源并联的8Ω电
阻除去(断开),并不影响电阻RL 上的电压U ,这样简化后的电路如图14(b)所
示,由此得
16
4+
U= 4 V = 12.8V
1 1 1
+ +
4 4 8
计算理想电流源的功率时,不能除去4Ω电阻,其上电压U4 = 4 × 4V = 16V ,并
由此可得理想电流源上电压US = U4 + U = (16 + 12.8)V = 28.8V 。理想电流源
的功率则为
PS = 28.8 × 4W = 115.2W (发出功率)

2.6 叠加定理

2.6.1

在 图15中 ,(1)当 将 开 关S合 在a点 时 , 求 电 流I1 、I2 和I3 ;(2)当 将 开 关S合


在b点时,利用(1)的结果,用叠加定理计算电流I1 、I2 和I3 。
[解]

14
图 15: 习题2.6.1图

(1) 当将开关S合在a点时,应用结点电压法计算:
130 120
+
U = 12 1 2 1 V = 100V
+ +
2 2 4
130 − 100
I1 = A = 15A
2
120 − 100
I2 = A = 10A
2
100
I3 = A = 25A
4
(2) 当将开关S合在b点时,应用叠加原理计算。在图15(b)中是20V 电源单独
作用时的电路,其中各电流为
4
I10 = × 6A = 4A
2+4
20
I 02 = A = 6A
2×4
2+
2+4
2
I30 = × 6A = 2A
2+4
130V 和120V 两个电源共同作用(20V 电源除去)时的各电流即为(1)中的
电流,于是得出

I1 = (15 − 4)A = 11A


I2 = (10 + 6)A = 16A
I3 = (25 + 2)A = 27A

2.6.2

电路如图16(a)所示,E = 12V ,R1 = R2 = R3 = R4 ,Uab = 10V 。若将理想

15
电压源除去后[图16(b)],试问这时Uab 等于多少?
[解]

图 16: 习题2.6.2图 将图16(a)

分为图16(b)和图16(c)两个叠加的电路,则应有

0 00
Uab = Uab + Uab

R3 1
U 00 = E = × 12V = 3V
ab
R1 + R2 + R3 + R4 4

0
Uab = (10 − 3)V = 7V

2.6.3

应用叠加原理计算图17(a)所示电路中各支路的电流和各元件(电源和电阻)
两端的电压,并说明功率平衡关系。
[解]
(1) 求各支路电流 电压源单独
作用时[图17(b)]

E 10
= I4 = = A = 2A
2
R +R 1+4
I0 0
2 4

E 10
I30 = = A = 2A
R3 5
IE0 = I20 + I30 16
= (2 + 2)A = 4A
图 17: 习题2.6.3图

电流源单独作用时[图17(c)]
R4 4
I 002 = IS = × 10A = 8A
R2 + R4 1+4
R2 1 × 10A = 2A
I400 = IS =
R2 + R4 1+4
IE00 = I200 = 8A
I300 = 0

两者叠加,得

I2 = I 02 − I200 = (2 − 8)A = − 6A
I3 = I 03 + I 003 = (2 + 0)A = 2A
I4 = I 40 + I 400 = (2 + 2)A = 4A
IE = I 0 − I = (4 − 8)A = − 4A
00
E E

可见,电流源是电源,电压源是负载。
(2) 求各元件两端的电压和功率
电流源电压 US = R1 IS + R4 I4 = (2 × 10 + 4 × 4)V = 36V
各电阻元件上电压可应用欧姆定律求得
电流源功率 PS = US IS = 36 × 10W = 360W (发出) 电
压源功率 PE = EIE = 10 × 4W = 40W (取用)
电阻R1 功率 PR1 = R1 I S2 = 2 × 102 W = 200W (损耗)
电阻R2 功率 PR2 = R2 I 22 = 1 × 6 W = 36W
2
(损耗)

17
电阻R3 功率 PR3 = R3 I 33 = 5 × 22 W = 20W (损耗)
电阻R4 功率 PR4 = R4 I 42 = 4 × 42 W = 64W (损耗)
两者平衡。

2.6.4

图18所示的是R − 2RT 形网络,用于电子技术的数模转换中,试用叠加原理


证明输出端的电流I 为
UR
I= (23 + 22 + 21 + 20 )
3R × 24
[解]

图 18: 习题2.6.4图

图 19: 习题2.6.4图

本题应用叠加原理、电阻串并联等效变换及分流公式进行计算求证。任何一
个电源UR 起作用,其他三个短路时,都可化为图19所示的电路。四个电源从右
到左依次分别单独作用时在输出端分别得出电流:
UR UR UR UR
, , ,
3R × 2 3R × 4 3R × 8 3R × 16
所以
UR UR UR UR
I= + + +
3R × 2 1 3R × 2 2 3R × 2 3 3R × 24
UR
= (23 + 22 + 21 + 20 )
3R × 2 4

18
2.7 戴维南定理与诺顿定理
2.7.1

应用戴维宁定理计算图20(a)中1Ω电阻中的电流。
[解]

图 20: 习题2.7.1图

将 与10A理 想 电 流 源 串 联 的2Ω电 阻 除 去 ( 短 接 ) , 该 支 路 中 的 电 流 仍
为10A; 将 与10V 理 想 电 压 源 并 联 的5Ω电 阻 除 去 ( 断 开 ) , 该 两 端 的 电 压 仍
为10V 。因此,除去这两个电阻后不会影响1Ω电阻中的电流I ,但电路可得到简
化[图20(b)],计算方便。
应用戴维宁定理对图20(b)的电路求等效电源的电动势(即开路电压U0 )和
内阻R0 。
由图20(c)得
U0 = (4 × 10 − 10)V = 30V
由图20(d)得 所以1Ω
R0 = 4Ω
电阻中的电流
U0 30
I= = A = 6A
R0 + 1 4+1

2.7.2

应用戴维宁定理计算图21中2Ω电阻中的电流I 。
[解]

19
图 21: 习题2.7.2图

求开路电压Uab0 和等效电阻R0 。
12 − 6
Uab0 = Uac + Ucd + Udb = (− 1 × 2 + 0 + 6 + 3 × )V = 6V
3+6
3×6
R 0 = (1 + 1 + )Ω = 4Ω
3+6
由此得
6
I= A = 1A
2+4

2.7.5

用戴维宁定理计算图22(a)所示电路中的电流I 。
[解]

图 22: 习题2.7.5图

(1) 用戴维宁定理将图22(a)化为等效电源,如图22(b)所示。

20
(2) 由图22(c)计算等效电源的电动势E,即开路电压U0

U0 = E = (20 − 150 + 120)V = − 10V

(3) 由图22(d)计算等效电源的内阻R0

R0 = 0

(4) 由图22(b)计算电流I
E − 10A = − 1A
I= =
R0 + 10 10

2.7.7

在图23中,(1)试求电流I ;(2)计算理想电压源和理想电流源的功率,并说明
是取用的还是发出的功率。
[解]

图 23: 习题2.7.7图

(1) 应用戴维宁定理计算电流I

Uab0 = (3 × 5 − 5)V = 10V


R0 = 3Ω
10
I = A = 2A
2+3
(2) 理想电压源的电流和功率
5
IE = I4 − I = ( − 2)A = − 0.75A
4
IE 的实际方向与图中相反,流入电压源的“+”端,故该电压源为负载。

PE = 5 × 0.75W = 3.75W (取用)

理想电流源的电压和功率为

US = [2 × 5 + 3(5 − 2)]V = 19V


PS = 19 × 5W = 95W (发出)

21
2.7.8

电路如图24(a)所示,试计算电阻RL 上的电流IL ;(1)用戴维宁定理;(2)用诺


顿定理。
[解]

图 24: 习题2.7.8图

(1) 应用戴维宁定理求IL

E = Uab0 = U − R3 I = (32 − 8 × 2)V =


16V R0 = R3 = 8Ω
E 16
IL = = A = 0.5A
RL + R0 24 + 8
(2) 应用诺顿定理求IL
U 32
IS = IabS = − I = ( − 2)A = 2A
R3 8
R0 8
IL = IS = × 2A = 0.5A
RL + R0 24 + 8

2.7.9

电路如图25(a)所示,当R = 4Ω时,I = 2A。求当R = 9Ω时,I 等于多少?


[解]
把电路ab以左部分等效为一个电压源,如图25(b)所示,则得
E
I=
R0 + R
R0 由图25(c)求出,即
R0 = R2 //R4 = 1Ω
所以
E = (R0 + R)I = (1 + 4) × 2V = 10V

当R = 9Ω时
10
I= A = 1A
1+9

22
图 25: 习题2.7.9图

2.7.10

试求图26所示电路中的电流I 。
[解]

图 26: 习题2.7.10图

用戴维宁定理计算。
(1) 求ab间的开路电压U0
a点电位Va 可用结点电压法计算
− 24
48
+
Va = 6 6 V = 8V
1 1 1
+ +
6 6 6
b点电位
12 − 24
+
Vb = 12 1 3 1 V = − 2V
+ +
2 6 3
U0 = E = Va − Vb = [8 − (− 2)]V =
10V

(2) 求ab间开路后其间的等效内阻R0
将电压源短路后可见,右边三个6Ω电阻并联,左边2Ω,6Ω,3Ω三个电阻
23
也并联,而后两者串联,即得

1 1
R0 = + kΩ = (2 + 1)kΩ = 3kΩ
1 1 1 1 1 1
+ + + +
6 6 6 2 6 3
(3) 求电流I

U0 10
I= = A = 2 × 10−3 A = 2mA
R0 + R (3 + 2) × 103

2.7.11

两个相同的有源二端网络N 和N 0 联结如图27(a)所示,测得U1 = 4V 。若联结


如图27(b)所示,则测得I1 = 1A。试求联结如图27(c)所示时电流I1 为多少?
[解]

图 27: 习题2.7.11图 有源二端网络可用等效

电源代替,先求出等效电源的电动势E和内阻R0
(1) 由图27(a)可知,有源二端网络相当于开路,于是得开路电压

E = U0 = 4V

(2) 由图27(b)可知,有源二端网络相当于短路,于是得短路电流

I1 = IS = 1A

由开路电压和短路电流可求出等效电源的内阻
E 4
R0 = = Ω = 4Ω
IS 1

(3) 于是,由图27(c)可求得电流I1
4
I1 = A = 0.8A
4+1

24
25
目录
第3章 电路的暂态分析 3
第3.2节 储能元件与换路定则 . . . . . . . . . . . . . . . . . . . . . . . 3
第3.2.1题 . . . . . . . . . . . . . . . . . . . . . . . . . . . . . . . 3第
3.2.2题 . . . . . . . . . . . . . . . . . . . . . . . . . . . . . . . 4 第3.3
节 RC电路的响应 . . . . . . . . . . . . . . . . . . . . . . . . . . . 5第
3.3.1题 . . . . . . . . . . . . . . . . . . . . . . . . . . . . . . . 5第
3.3.3题 . . . . . . . . . . . . . . . . . . . . . . . . . . . . . . . 5第
3.3.4题 . . . . . . . . . . . . . . . . . . . . . . . . . . . . . . . 6 第3.4
节 一阶线性电路暂态分析的三要素法 . . . . . . . . . . . . . . . 7
第3.4.1题 . . . . . . . . . . . . . . . . . . . . . . . . . . . . . . . 7第
3.4.2题 . . . . . . . . . . . . . . . . . . . . . . . . . . . . . . . 8第
3.4.3题 . . . . . . . . . . . . . . . . . . . . . . . . . . . . . . . 10 第
3.4.4题 . . . . . . . . . . . . . . . . . . . . . . . . . . . . . . . 11 第
3.4.5题 . . . . . . . . . . . . . . . . . . . . . . . . . . . . . . . 12 第3.6
节 RL电路的响应 . . . . . . . . . . . . . . . . . . . . . . . . . . . 13
第3.6.1题 . . . . . . . . . . . . . . . . . . . . . . . . . . . . . . . 13
第3.6.2题 . . . . . . . . . . . . . . . . . . . . . . . . . . . . . . . 14
第3.6.4题 . . . . . . . . . . . . . . . . . . . . . . . . . . . . . . . 16
第3.6.5题 . . . . . . . . . . . . . . . . . . . . . . . . . . . . . . . 17

1
List of Figures
1 习题3.2.1图 . . . . . . . . . . . . . . . . . . . . . . . . . . . . . . . . 3
2 习题3.2.2图 . . . . . . . . . . . . . . . . . . . . . . . . . . . . . . . . 4
3 习题3.3.1图 . . . . . . . . . . . . . . . . . . . . . . . . . . . . . . . . 5
4 习题3.3.3图 . . . . . . . . . . . . . . . . . . . . . . . . . . . . . . . . 6
5 习题3.3.4图 . . . . . . . . . . . . . . . . . . . . . . . . . . . . . . . . 6
6 习题3.4.1图 . . . . . . . . . . . . . . . . . . . . . . . . . . . . . . . . 7
7 习题3.4.2图 . . . . . . . . . . . . . . . . . . . . . . . . . . . . . . . . 8
8 习题3.4.2图 . . . . . . . . . . . . . . . . . . . . . . . . . . . . . . . . 9
9 习题3.4.3图 . . . . . . . . . . . . . . . . . . . . . . . . . . . . . . . . 10
10 习题3.4.4图 . . . . . . . . . . . . . . . . . . . . . . . . . . . . . . . . 11
11 习题3.4.5图 . . . . . . . . . . . . . . . . . . . . . . . . . . . . . . . . 12
12 习题3.4.5图 . . . . . . . . . . . . . . . . . . . . . . . . . . . . . . . . 13
13 习题3.6.1图 . . . . . . . . . . . . . . . . . . . . . . . . . . . . . . . . 14
14 习题3.6.2图 . . . . . . . . . . . . . . . . . . . . . . . . . . . . . . . . 15
15 习题3.6.4图 . . . . . . . . . . . . . . . . . . . . . . . . . . . . . . . . 16
16 习题3.6.5图 . . . . . . . . . . . . . . . . . . . . . . . . . . . . . . . . 17

2
3 电路的暂态分析

3.2 储能元件与换路定则

3.2.1

图1所示各电路在换路前都处于稳态,试求换路后其中电流 i 的初始值i(0+ )和
稳态值i(∞ ).
[解]

图 1: 习题3.2.1图

(1) 对图1(a)所示电路

6
iL (0+ ) = iL (0− ) = A = 3A
2
2
i(0+ ) = × 3A = 1.5A
2+2
i(∞ ) = 6 1
× A = 3A
×2 2
2+2

(2) 对图1(b)所示电路

uc (0+ ) = uc (0− ) = 6V
6− 6
i(0+ ) = A =0
2
i(∞ ) = 6
A = 1.5A
2+2

3
(3) 对图1(c)所示电路

iL1 (0+ ) = iL1 (0− ) = 6A


iL2 (0+ ) = iL2 (0− ) = 0
i(0+ ) = iL1 (0+ ) − iL2 (0+ ) = (6 − 0)A = 6A
i(∞ ) = 0

(4) 对图1(d)所示电路

6
uc (0+ ) = uc (0− ) = × 2V = 3V
2+2
6− 3
i(0+ ) = A = 0.75A
2+2
i(∞ ) = 6
A = 1A
2+2+2

3.2.2

图2所示电路在换路前处于稳态,试求换路后iL ,uc 和iS 的初始值和稳态值。


[解]

图 2: 习题3.2.2图

15 30 1 30 1
iL (0+ ) = iL (0− ) = × A= × A= A
15 × 30 30 + 15 2 30 + 15 3
10 + 10 +
15 + 30
uc (0+ ) = uc (0− ) = (15 − 10 × 0.5)V = 10V
uc (0+ ) 10 1 2
iS (0+ ) = i1 (0+ ) − iL (0+ ) = − iL (0+ ) = ( − )A = A
10 10 3 3
30Ω电阻被短接,其中电流的初始值为零。

iL (∞ ) = 0
15
uC (∞ ) = 10 × V = 7.5V
10 + 10
15 3
iS (∞ ) = A= A
10 + 10 4

4
3.3 RC电路的响应
3.3.1

在图3中,I = 10mA,R1 = 3kΩ,R2 = 3kΩ,R3 = 6kΩ,C = 2µF 。在开关S闭合


前电路已处于稳态。求在t ≥ 0时uC 和i1 ,并作出它们随时间的变化曲线。
[解]

图 3: 习题3.3.1图

uc (0+ ) = uc (0− ) = R3 I = 6 × 10 3 × 10 × 10−3 V = 60V = U0

与电容元件串联的等效电阻

R2 R3 3×6
R = R1 + = (3 + )kΩ = 5kΩ
R2 + R3 3+6

时间常数
τ = RC = 5 × 103 × 2 × 10−6 s = 0.01s

本题求的是零输入响应(电流源已被短接),故得
t t
− −
uc = U0 e τ = 60e 0.01 = 60e−100t V
t
duC U0 − 60
i1 = −C = e τ = e− 100t = 12e−100t mA
dt R 5 × 103

3.3.3

电路如图4所示,在开关S闭合前电路已处于稳态,求开关闭合后的电压uc 。
[解]

uc (0+ ) = uc (0− ) = 6 × 10 3 × 9 × 10−3 V = 54V


6×3
τ = × 103 × 2 × 10−6 s = 4 × 10−3 s
6+3

5
图 4: 习题3.3.3图

本题是求全响应uc :先令9mA理想电流源断开求零输入相应u0c;而后令uc (0+ ) =


0求零状态响应u00c;最后得uc = u0c + u00c。

t t
− −
uc0 = U0 e τ = 54e 4 × 10−3 V = 54e−250t V
t

u00c = U (1 − e τ ) = 18(1 − e−250t )V
3×6 −3
式中 U = u c(∞ ) = × 103 × 9 × 10 V = 18V
3+6
uc = (18 + 36e−250t )V

3.3.4

有 一 线 性 无 源 二 端 网 络N [图5(a)], 其 中 储 能 元 件 未 储 有 能 量 , 当 输 入 电
流i[其波形如图5(b)所示]后,其两端电压u的波形如图5(c)所示。(1)写出u的指数
式;(2)画出该网络的电路,并确定元件的参数值。
[解]

图 5: 习题3.3.4图

6
(1) 由图5(c)可得

t = 0 ∼ τ时
t

u = 2(1 − e τ )V
u(τ ) = 2(1 − 0.368)V = 2 × 0.632V = 1.264V
t = τ ∼ ∞时
(t − 1)

u = 1.264e τ V

(2) 该网络的电路如图5(d)所示。因
u(∞ ) = Ri = 2V
R ×1= 2 R = 2Ω

τ = RC 1 = 2C C = 0.5F

3.4 一阶线性电路暂态分析的三要素法
3.4.1

在 图6(a)所 示 的 电 路 中 ,u为 一 阶 跃 电 压 , 如 图6(b)所 示 , 试 求i3 和uc 。


设uc (0− ) = 1V
[解]

图 6: 习题3.4.1图

应用三要素法计算。
(1) 求uc

uc (0+ ) = uc (0− ) = 1V
u 4
uc (∞ ) = R3 =2× V = 2V
R + R3
? 1 ? 2 + 2? ?
R1 R3 2×2
τ = R2 + C = 1+ × 103 × 1 × 10−6 s
R1 + R3 2+2
−3
= 2 × 10 s

7
由此得

t

uc = uc (∞ ) + [uc (0+ ) − uc (∞ )]e τ
t

= [2 + (1 − 2)e 2 × 10−3 ]V = (2 − e−500t )V

(2) 求i3

u uc (0+ ) ? ? 4 1
+ 1 + 1 3
i3 (0+ ) = 2 1 = 2 1 × mA = mA
1 1 1 R3 1 1 1 2 4
+ + + +
2 1 2 2 1 2
u 4
i3 (∞ ) = = mA = 1mA
R1 + R3 2+2

由此得

t

i3 = i3 (∞ ) + [i3 (0+ ) − i3 (∞ )]e τ
3
= 1 + ( − 1)e−500t mA = (1 − 0.25e−500t )mA
4

3.4.2

电路如图7所示,求t ≥ 0时(1)电容电压uc ,(2)B点电位vB 和(3)A点电位vA 的


变化规律。换路前电路处于稳态。
[解]

图 7: 习题3.4.2图

(1) 求t ≥ 0时的电容电压uc
t = 0− 和t = 0+ 的电路如图8(a)、(b)所示,由此得

8
图 8: 习题3.4.2图

0 − (− 6)
uc (0+ ) = uc (0− ) = × 5 × 103V = 1V
(5 + 25) × 103
6 − (− 6)
uc (∞ ) = × 5 × 10 3V = 1.5V
(10 + 5 + 25) × 103
τ = [(R1 + R3 )//R2 ]C = 0.44 × 10−6 s

t

uc = [1.5 + (1 − 1.5)e 0.44 × 10−6 ]V
= (1.5 − 0.5e−2.3×10 t )V
6

(2) 求t ≥ 0时的B点电位vB
? ?
6 − (− 6) − 1
VB (0+ ) = 6− × 10 × 10 V
3
(10 + 25) × 103
= (6 − 3.14)V = 2.86V
? ?
VB (∞ ) = 6 − 6 − (− 6)
× 10 × 10 V
3
(10 + 5 + 25) × 103
= (6 − 3)V = 3V


vB = [3 + (2.86 − 3)e−2.3×106 t ]V
= (3 − 0.14e−2.3×106 t )V

注意:(1)VB (0− ) = 0,而VB (0+ ) = 2.86V = VB (0− );(2)在t = 0+ 的电路


中,电阻10kΩ和25kΩ中通过同一电流,两者串联,而电阻5kΩ中通过另
一电流,因此它与10kΩ或25kΩ不是串联的,在t = ∞ 的电路中,三者才
相串联;(3)在t = 0+ 的电路中,计算电阻10kΩ或25kΩ中电流的式子是
6 − (− 6) − 1

A
(10 + 25) × 103
9
(3) 求t ≥ 0时A点电位vA
? ?
6 − (− 6) − 1
VA (0+ ) = × 25 × 10 + (− 6) V
3
(10 + 25) × 103
= (7.86 − 6)V = 1.86V
? ?
6 − (− 6)
VA (∞ ) = × 25 × 10 + (− 6) V
3
(10 + 5 + 25) × 103
= (7.5 − 6)V = 1.5V

vA = [1.5 + (1.86 − 1.5)e−2.3×10 t ]V


6

= (1.5 + 0.36e−2.3×10 t )V
6

3.4.3

电路如图9所示,换路前已处于稳态,试求换路后(t ≥ 0)的uc 。
[解]

图 9: 习题3.4.3图

本题应用三要素法计算。
(1) 确定初始值

uc (0+ ) = uc (0− ) = (20 × 10 3 × 1 × 10−3 − 10)V = 10V

(2) 确定稳态值
? ?
10
uc (∞ ) = × 1 × 10 − 3 × 20 × 103
− 10 V = − 5V
10 + 10 + 20
(3) 确定时间常数
将理想电流源开路,理想电压源短路。从电容元件两端看进去的等效电
阻为
20 × (10 + 10)
R0 = kΩ = 10kΩ
20 + (10 + 10)

10
故 于是
τ = R0 C = 10 × 103 × 10 × 10−6 s = 0.1s
得出
t

uc = uc (∞ ) + [uc (0+ ) − uc (∞ )]e τ
t

= − 5 + [10 − (− 5)]e 0.1
= (− 5 + 15e−10t )V

3.4.4

有一RC 电路[图10(a)],其输入电压如图10(b)所示。设脉冲宽度T = RC 。试
求负脉冲的幅度U− 等于多大才能在t = 2T 时使uc = 0。设uc (0− ) = 0。
[解]

图 10: 习题3.4.4图

由t = 0到t = T 期间
t

uc = 10(1 − e τ )V
uc (T ) = 10(1 − e−1 ) = 6.32V

由t = T 到t = 2T 期间
t− T

= U− + [uc (T ) − U− ]e T
u0c
t = 2T 时u0c = 0,即
2T − T

U− + [uc (T ) − U− ]e T =0
U− + (6.32 − U− ) × 0.368 = 0
U− = − 3.68V

11
3.4.5

在 图11中 , 开 关S先 合 在 位 置1, 电 路 处 于 稳 态 。t = 0时 , 将 开 关 从 位


置1合 到 位 置2, 试 求t = τ 时uc 之 值 。 在t = τ 时 , 又 将 开 关 合 到 位 置1, 试
求t = 2 × 10−2 s时uc 之值。此时再将开关合到2,作出的uc 变化曲线。充电电路
和放电电路的时间常数是否相等?
[解]

图 11: 习题3.4.5图

(1) t = 0时,将开关从1合到2

uc (0− ) = uc (0+ ) = 10V


t

uc = 10e τ1
1
τ1 = (20 + 10) × 103 × × 10−6 s = 10−2 s = 0.01s
3
uc (τ1 ) = 10e−1 V = 10 × 0.368V = 3.68V

12
(2) t = τ 时又将开关合到1

uc (τ1 ) = 3.68V
uc (∞ ) = 10V
τ2 = 10 × 103 × 1 × 10−6 s = 1 × 10−2 s = 0.0033s
3 3
(t − 0.01)

uc = 10 + (3.68 −10)e τ2 V

(t − 0.01)

= (10 − 6.32e τ2 )V
(0.02 − 0.01)

uc (0.02s) = 10 − 0.0033 V
6.32e

= (10 − 6.32e−3 )V
= (10 − 6.32 × 0.05)V
= 9.68V

(3) t = 0.02s时,再将开关合到2
(t − 0.02)

τ1 V
uc = 9.68e

uc 的变化曲线如图12所示。

图 12: 习题3.4.5图

3.6 RL电路的响应
3.6.1

在图13中,R1 = 2Ω,R2 = 1Ω,L1 = 0.01H ,L2 = 0.02H ,U = 6V 。(1)试


求S1 闭合后电路中电流i1 和i2 的变化规律;(2)当闭合S1 后电路到达稳定状态时再
[解]

13
闭合S2 ,试求i1 和i2 的变化规律。

[解]

14
图 13: 习题3.6.1图

(1) 当开关S1 闭合前,i1 (0− ) = i2 (0− ) = 0,故以零状态响应计算,即


t
U −
i1 = i2 = (1 − e τ1 )
R1 + R2
式中
L1 + L2 0.01 + 0.02
τ1 = = s = 0.01s
R1 + R2 1+2

t
6 −
i1 = i2 = (1 − e 0.01 )A = 2(1 − e−100t)A
1+2
电路到达稳态时,
i1 (∞ ) = i2 (∞ ) = 2A

(2) 到达稳态时闭合S2 后,i1 (0+ ) = i2 (0+ ) = 2A。闭合S2 后到达稳态时,


U 6
i1 (∞ ) = = A = 3Ai2 (∞ ) = 0 时间常数分别为
R1 2
L1 0.01
τ10 = = s = 0.005s
R1 2
L2 0.02
τ2 = = s = 0.02s
R2 1
于是得出
t

i1 = [3 + (2 − 3)e 0.005 ]A = (3 − e−200t )A
t

i2 = [0 + (2 − 0)e 0.02 ]A = 2e−50t A

3.6.2

电路如图14所示,在换路前已处于稳态。当将开关从1的位置扳到2的位置
后,试求i和iL 。

[解]

15
图 14: 习题3.6.2图

(1) 确定初始值

−3 9
i(0− ) = A =− A
2×1 5
1+
2+1
2 9 6
iL (0+ ) = iL (0− ) = × (− )A = − A
2+1 5 5

在此注意,i(0+ ) = i(0− )。
i(0+ )由基尔霍夫电压定律计算,即

3 = ×i(0+ ) + 2[i(0+ ) − iL (0+ )]


6
3 = i(0+ ) + 2[i(0+ ) + ]
5
12
3 = 3i(0+ ) +
5
1
i(0+ ) = A
5

(2) 确定稳态值

3 9
i(∞ ) = A= A
2 ×1 5
1+
2+1
2 9 6
iL (∞ ) = × A= A
2+1 5 5

(3) 确定时间常数

L 3 9
τ = = s= s
R0 2 ×1 5
1+
2+1

15
于是得

t
i = i(∞ ) + [i(0+ ) − i(∞ )]e− τ
5 5
9 1 9 − t 9 8 − t
= + ( − )e 9 A= − 9 A
5 5 5 5 5
e
t

= (1.8 − 1.6e 1.8 )A
5 t
6 6 6 − t −
iL = + (− − 9 A = (1.2 − 1.8 )A
5 5 2.4e
)e
5

3.6.4

电路如图15所示,试用三要素法求t ≥ 0时的i1 ,i2 及iL 。换路前电路已处于


稳态。
[解]

图 15: 习题3.6.4图

(1) 确定初始值

12
iL (0+ ) = iL (0− ) = A = 2A
6

注 意 :i1 和i2 的 初 始 值 应 按t = 0+ 的 电 路 计 算 , 不 是 由t = 0− 的 电 路 计
算。由t = 0+ 的电路应用基尔霍夫定律列出

i1 (0+ ) + i2 (0+ ) = iL (0+ ) = 2


6i1 (0+ ) − 3i2 (0+ ) = 12 − 9 = 3

解之得

i1 (0+ ) = i2 (0+ ) = 1A
16
(2) 确定稳态值 稳态时电感元件可视
为短路,故

i1 (∞ ) = 12 A = 2A
6
i2 (∞ ) = 9
A = 3A
3
iL (∞ ) = i1 (∞ ) + i2 (∞ ) = (2 + 3)A = 5A

(3) 确定时间常数

L 1
τ = = s = 0.5s
R0 6×3
6+3

于是得出

t

i1 = [2 + (1 − 2)e 0.5 ]A = (2 − e−2t )A
t

i2 = [3 + (1 − 3)e 0.5 ]A = (3 − 2e−2t )A
t

iL = [5 + (2 − 5)e 0.5 ]A = (5 − 3e−2t )A

3.6.5

当 具 有 电 阻R = 1Ω及 电 感L = 0.2H 的 电 磁 继 电 器 线 圈 ( 图16) 中 的 电


流i = 30A时 , 继 电 器 即 动 作 而 将 电 源 切 断 。 设 负 载 电 阻 和 线 路 电 阻 分 别
为RL = 20Ω和Rl = 1Ω,直流电源电压U = 220V ,试问当负载被短路后,需要
经过多少时间继电器才能将电源切断?
[解]

图 16: 习题3.6.5图

17
200
i(0+ ) = i(0− ) = A = 10A
1 + 20 + 1
220
i(∞ ) = A = 110A
1+1
0.2
τ = s = 0.1s
1+1
于是得
t

i = [110 + (10 − 110)]e 0.1 A = (110 − 100e−10t )A

当i = 30A时

30 = 110 − 100e−10t
110 − 30
e−10t = = 0.8s
100
1 1
t = ln s = 0.02s
10 0.8
经过0.02s继电器动作而将电源切断。

18
19
目录
第4章 正弦交流电路 3
第4.3节 单一参数的交流电路 . . . . . . . . . . . . . . . . . . . . . . . 3
第4.3.2题 . . . . . . . . . . . . . . . . . . . . . . . . . . . . . . . 3 第4.4
节 电阻、电感与电容元件串联的交流电路 . . . . . . . . . . . . . 4
第4.4.2题 . . . . . . . . . . . . . . . . . . . . . . . . . . . . . . . 4第
4.4.3题 . . . . . . . . . . . . . . . . . . . . . . . . . . . . . . . 4第
4.4.4题 . . . . . . . . . . . . . . . . . . . . . . . . . . . . . . . 5第
4.4.5题 . . . . . . . . . . . . . . . . . . . . . . . . . . . . . . . 5第
4.4.6题 . . . . . . . . . . . . . . . . . . . . . . . . . . . . . . . 6第
4.4.7题 . . . . . . . . . . . . . . . . . . . . . . . . . . . . . . . 6第
4.4.8题 . . . . . . . . . . . . . . . . . . . . . . . . . . . . . . . 7第
4.4.9题 . . . . . . . . . . . . . . . . . . . . . . . . . . . . . . . 8第
4.4.11题 . . . . . . . . . . . . . . . . . . . . . . . . . . . . . . 8 第4.5
节 阻抗的串联与并联 . . . . . . . . . . . . . . . . . . . . . . . . . 9
第4.5.1题 . . . . . . . . . . . . . . . . . . . . . . . . . . . . . . . 9第
4.5.3题 . . . . . . . . . . . . . . . . . . . . . . . . . . . . . . . 10 第
4.5.4题 . . . . . . . . . . . . . . . . . . . . . . . . . . . . . . . 11 第
4.5.5题 . . . . . . . . . . . . . . . . . . . . . . . . . . . . . . . 12 第
4.5.6题 . . . . . . . . . . . . . . . . . . . . . . . . . . . . . . . 13 第
4.5.7题 . . . . . . . . . . . . . . . . . . . . . . . . . . . . . . . 13 第
4.5.11题 . . . . . . . . . . . . . . . . . . . . . . . . . . . . . . 14 第
4.5.12题 . . . . . . . . . . . . . . . . . . . . . . . . . . . . . . 15 第4.7
节 交流电路的频率特性 . . . . . . . . . . . . . . . . . . . . . . . 16
第4.7.4题 . . . . . . . . . . . . . . . . . . . . . . . . . . . . . . . 16 第
4.7.5题 . . . . . . . . . . . . . . . . . . . . . . . . . . . . . . . 17 第
4.7.6题 . . . . . . . . . . . . . . . . . . . . . . . . . . . . . . . 17 第4.8
节 功率因数的提高 . . . . . . . . . . . . . . . . . . . . . . . . . . 19
第4.8.2题 . . . . . . . . . . . . . . . . . . . . . . . . . . . . . . . 19
第4.8.3题 . . . . . . . . . . . . . . . . . . . . . . . . . . . . . . . 20

1
List of Figures
1 习题4.3.2图 . . . . . . . . . . . . . . . . . . . . . . . . . . . . . . . . 3
2 习题4.3.2图 . . . . . . . . . . . . . . . . . . . . . . . . . . . . . . . . 3
3 习题4.4.6图 . . . . . . . . . . . . . . . . . . . . . . . . . . . . . . . . 6
4 习题4.4.7图 . . . . . . . . . . . . . . . . . . . . . . . . . . . . . . . . 7
5 习题4.4.8图 . . . . . . . . . . . . . . . . . . . . . . . . . . . . . . . . 7
6 习题4.4.9图 . . . . . . . . . . . . . . . . . . . . . . . . . . . . . . . . 8
7 习题4.5.1图 . . . . . . . . . . . . . . . . . . . . . . . . . . . . . . . . 10
8 习题4.5.3图 . . . . . . . . . . . . . . . . . . . . . . . . . . . . . . . . 11
9 习题4.5.4图 . . . . . . . . . . . . . . . . . . . . . . . . . . . . . . . . 11
10 习题4.5.5图 . . . . . . . . . . . . . . . . . . . . . . . . . . . . . . . . 12
11 习题4.5.6图 . . . . . . . . . . . . . . . . . . . . . . . . . . . . . . . . 13
12 习题4.5.7图 . . . . . . . . . . . . . . . . . . . . . . . . . . . . . . . . 14
13 习题4.5.11图 . . . . . . . . . . . . . . . . . . . . . . . . . . . . . . . 14
14 习题4.5.12图 . . . . . . . . . . . . . . . . . . . . . . . . . . . . . . . 15
15 习题4.7.5图 . . . . . . . . . . . . . . . . . . . . . . . . . . . . . . . . 17
16 习题4.7.6图 . . . . . . . . . . . . . . . . . . . . . . . . . . . . . . . . 18
17 习题4.8.2图 . . . . . . . . . . . . . . . . . . . . . . . . . . . . . . . . 19
18 习题4.8.3图 . . . . . . . . . . . . . . . . . . . . . . . . . . . . . . . . 20

2
4 正弦交流电路

4.3 单一参数的交流电路

4.3.2

在电容为64µF 的电容器两端加一正弦电压u = 220 2 sin 314 t V ,设电压和 电
T T T
流的参考方向如图1所示,试计算在t = ,t = 和t = 瞬间的电流和电压
6 4 2
的大小。
[解]
电压与电流的正弦曲线如图2所示。

图 1: 习题4.3.2图

图 2: 习题4.3.2图


Um = 220 2V U = 220V
U
I = = U ωC = 220 × 314 × 64 × 10− 6 A = 4.42A
1
ωC

3
T
(1) t = 时
6
√ 1 T √ √
u = 220 2 sin(2π × × )V = 220 2 sin 60◦ V = 110 6V
√ T 6 √
◦ ◦
i = 4.42 2 sin(60 + 90 )A = 2.21 2A

T
(2) t = 时
4
√ 1 T √ √
u = 220 2 sin(2π × × )V = 220 2 sin 90◦ V = 220 2V
√ T 4
◦ ◦
i = 4.42 2 sin(90 + 90 )A = 0

T
(3) t = 时
2
√ 1 T √
u = 220 2 sin(2π × × )V = 220 2 sin 180◦ V = 0
√ T 2 √
◦ ◦
i = 4.42 2 sin(180 + 90 )A = −4.42 2A

4.4 电阻、电感与电容元件串联的交流电路

4.4.2

有 一C J 0 − 10A交 流 接 触 器 , 其 线 圈 数 据 为380V 、30mA、50H z, 线 圈 电


阻1.6kΩ,试求线圈电感。
[解]
这是RL串联电路,其阻抗模为
p U 380
|Z | = R2 + (ωL)2 = = −3
Ω = 12700Ω = 12.7kΩ
I 30 × 10
q √
1 1
L = |Z |2 − R2 = 12.72 − 1.62 × 103 H = 40H
2πf 2π × 50

4.4.3

一个线圈接在U = 120V 的直流电源上,I = 20A;若接在f = 50H z,U =


220V 的交流电源上,则I = 28.2A。试求线圈的电阻R和电感L。
[解]
U 120
接在直流电源上电感L不起作用,故电阻R = = Ω = 6Ω。接在交流电
I 20
源上时,
p U 220
|Z | = R2 + (ωL)2 = = Ω = 7.8Ω
q I 28.2
1 1 √
L = |Z |2 − R2 = 7.82 − 62 H = 15.9mH
2πf 2π × 50

4
4.4.4

有一J Z 7型中间继电器,其线圈数据为380V 、50H z,线圈电阻2kΩ,线圈电


感43.3H ,试求线圈电流及功率因数。
[解]
线圈阻抗为

Z = R + jωL = (2 × 103 + j2π × 50 × 43.3)Ω = 13.8 × 103 ∠81.6◦ Ω


U 380
I = = A = 27.6mA
|Z | 13.8 × 103
cos ϕ = cos 81.6◦ = 0.15

4.4.5

日光灯管与镇流器串联接到交流电压上,可看作RL串联电路。如已知某灯
管的等效电阻R1 = 280Ω,镇流器的电阻和电感分别为R2 = 20Ω和L = 1.65H ,
电源电压U = 220V ,试求电路中的电流和灯管两端与镇流器上的电压。这两个
电压加起来是否等于220V ?电源频率为50H z。
[解]
电路总阻抗

Z = (R1 + R2 ) + jωL = [(280 + 20) + j2π × 50 × 1.65]Ω


= (300 + j518)Ω = 599∠59.9◦ Ω

电路中电流
U 220
I= = A = 0.367A
|Z | 599

灯管两端电压

UR = R1 I = 280 × 0.367V = 103V

镇流器的阻抗

Z2 = R2 + jωL = (2 + j518)Ω = 518∠87.8◦ Ω

镇流器电压

U2 = |Z2 | I = 518 × 0.367V = 190V


UR + U2 = (103 + 190)V = 293V > 220V

因为U̇ = U˙R + U̇2 ,不能有效值相加。

5
4.4.6

无源二端网络(图3)输入端的电压和电流为

u = 220 2 sin(314t + 20◦ )V

i = 4.4 2 sin(314t − 33◦ )A
试求此二端网络由两个元件串联的等效电路和元件的参数值,并求二端网络的
功率因数及输入的有功功率和无功功率。
[解]
二端网络阻抗为

图 3: 习题4.4.6图

U̇ 220∠20◦ ◦
Z =
˙I
= ◦ Ω = 50∠53 Ω = (30 + j40)Ω
4.4∠−33

则其参数为R = 30Ω XL = 40Ω


由此得出电感
XL 40
L= = H = 0.127H
ω 314
功率因数为
R 30
cos ϕ = = = 0.6
|Z | 50
输入的有功功率为
P = U I cos ϕ = 220 × 4.4 × 0.6W = 580W
无功功率为
Q = U I sin ϕ = XL I 2 = 40 × 4.42 var = 774var

4.4.7

有一RC 串联电路,如图4(a)所示,电源电压为u,电阻和电容上的电压分别
为uR 和uc ,已知电路阻抗模为2000Ω,频率为1000H z,并设u与uc 之间的相位差
为30°,试求R和C ,并说明在相位上uc 比u超前还是滞后。
[解]
按题意做电压和电流的相量图[图4(b)],由相量图可见

6
图 4: 习题4.4.7图

R = |Z | cos 60◦ = (2000 × 0.5)Ω = 1000Ω


XC = |Z | sin 60◦ = (2000 × 0.866)Ω = 1732Ω
1 1
C = = F = 0.1µF
XC ω 1732 × 2π × 1000

uc 滞后于u 30°.

4.4.8

图5(a)是一移相电路。如果C = 0.01µF ,输入电压u1 = 2 sin 6280t V ,今
欲使输出电压u2 在相位上前移60◦ ,问应配多大的电阻R?此时输出电压的有效
值U2 等于多少?
[解]
按题意做电压和电流的相量图[图5(b)],由相量图可见,

图 5: 习题4.4.8图

7
U2 = UR = U1 cos 60◦ = 1 × 0.5V = 0.5V
U R R
tan 30◦ = R = = = R × 6280 × 0.01 × 10−6
UC XC 1
6280 × 0.01 × 10−6
由上式可求出 R = 9.2kΩ

4.4.9

图6(a)是一移相电路。已知R = 100Ω,输入信号频率为500H z。如要求输出


电压u2 与输入电压u1 间的相位差为45◦ ,试求电容值。同上题比较,u2 与u1 在相
位上(滞后和超前)有何不同?
[解]
做电压和电流的相量图[图6(b)],由相量图可见

图 6: 习题4.4.9图

UR R 100
tan 45◦ = = = = C × 2π × 500 × 100
UC XC 1
2π × 500 × C
由上式可求出 C = 3.18µF
上题从电阻上输出,u2 超前于u1 ;本题从电容上输出,u2 滞后于u1 。

4.4.11

有一220V 、600W 的电炉,不得不用在380V 的电源上。欲使电炉的电压保持


在220V 的额定值,(1)应和它串联多大的电阻?或(2)应和它串联感抗为多大的电
感线圈(其电阻可忽略不计)?(3)从效率和功率因数上比较上述两种方法。串
联电容器是否也可以?
[解]

8
(1) 电炉的额定电流

600
IN = A = 2.73A
220
串联的电阻

380 − 220
R = Ω = 58.7Ω
2.73
(2) 电炉的电阻

220
RL = Ω = 80.7Ω
2.73
为保持电炉的额定值不变,则串联电感线圈后电路的阻抗模为
380
|Z | = Ω = 139.3Ω
2.73
线圈感抗为
q q
XL = |Z | − R L = (139.3)2 − (80.7)2 Ω = 114Ω
2

线圈电感为
XL 114
L= = H = 0.36H
ω 314
(3) 串联电阻时,cos ϕ = 1,效率为

PN 600
η= 2
= = 0.58
PN + RIN 600 + 58.7 × (2.73)2

串联线圈时,η = 1,功率因数为
RL 80.7
cos ϕ = = = 0.58
|Z | 139.3

从节能角度看应采用后一种方法。也可串联电容,其容抗为114Ω,电容
1 1
值为C = = F = 28µF ,并可提高电网功率因数。
ωXC 314 × 114

4.5 阻抗的串联与并联
4.5.1

在图7中,试求电流表A0 和电压表V0 的读数。


[解]
以U̇1 为参考正弦量,则Ż1 较U̇1 超前90◦ ,于是

9
图 7: 习题4.5.1图

100∠0◦
I˙0 = I˙1 + I˙2 = (j10 + )A
5 + j5

= (j10 + 10 2∠ − 45◦ )A = (j10 + 10 − j10)A
= 10A

A0 的读数为10A。

U̇0 = −j10I˙0 + U̇1 = (−j10 × 10 + 100∠0◦ )V


= (100 − j100)V = 141∠ − 45◦ V

V0 的读数为141V 。

4.5.3

在图8(a)中,I1 = 10A,I2 = 10 2A,U = 200V ,R = 5Ω,R2 = XL ,试
求I ,XC ,XL 及R2 。
[解]
解本题用相量图分析较为方便。在图8(b)所示的电路中,以并联支路上的电
压U̇ 1 为参考相量,而后作I˙1 ,I˙1 比U̇ 1 超前90◦ ;因R2 = XL ,故I˙2 比U̇ 1 滞后45◦ ;

又因I1 = 10A,I2 = 10 2A,故I˙与U̇1 同相,而且I = 10A;U̇R 也与U̇1 同相。
而U̇ = U̇R + U̇1 ,三者同相,即U = UR + U1 。

UR = RI = 5 × 10V = 50V
U1 = U − UR = (200 − 50)V = 150V
由此得 XC = U1 = 150 Ω = 15Ω
I1 10
q
U1 150 15
|Z2 | = R22 + XL2 = = √ Ω= √ Ω
I2 10 2 2
10
图 8: 习题4.5.3图

因为I˙2 与U̇1 的相位差为45◦ ,所以

|Z | 1 15
R2 = XL = √2 = √ × √ Ω = 7.5Ω
2 2 2

4.5.4

在图9(a)中,I1 = I2 = 10A,U = 100V ,u与i同相,试求I ,R,XC


及XL 。
[解]
作相量图:以U̇R 为参考相量,I˙2 与U̇R 同相,I˙1 超前于U̇R 90◦ ;I˙ = I˙1 + I˙2 =

图 9: 习题4.5.4图

11

10 2∠45◦ A,U̇L 超前于I˙ 90◦ ;U̇ 与I˙同相;U̇ = U̇L + U̇R 。于是得出

I = 10 2A√= 14.1A√
UR 2U 2 × 100
R = = = Ω = 14.1Ω
I2 I2 10

UR 2 × 100
XC = = Ω = 14.1Ω
I1 10
UL U 100
XL = = = √ Ω = 7.07Ω
I I 10 2

4.5.5

计 算 图10(a)中 的 电 流I˙和 各 阻 抗 元 件 上 的 电 压U̇1 与U̇2 , 并 作 相 量 图 ; 计 算


图10(b)中各支路电流I˙1 与I˙2 和电压U̇ ,并作相量图。
[解] 对图10(a)所示电路

图 10: 习题4.5.5图

U̇ 10∠0◦ A = 2∠ − 36.9◦ A
I˙ = =
Z1 + Z2 2 + 2 + j3
U̇1 = Z1 I˙ = 2 × 2∠ − 36.9◦ V = 4∠ − 36.9◦ V

U̇2 = Z2 I˙ = (2 + j3) × 2∠ − 36.9◦ V = 13∠56.3◦ × 2∠ − 36.9◦ V
= 7.21∠19.4◦ V

对图10(b)所示电路
Z2 1− j √
I˙1 = I˙ = × 2∠0◦ A = (1 − j)A = 2∠ − 45◦ A
Z1 + Z2 1+j +1− j
Z1 1+j √
I˙2 = I˙ = × 2∠0◦ A = (1 + j)A = 2∠45◦ A
Z1 + Z2 1+j +1− j
√ √
U̇ = Z1 I˙1 = 2∠45◦ 2∠ − 45◦ V = 2V

相量图略。

12
4.5.6

在图11中,已知U = 220V ,R1 = 10Ω,X1 = 10 3Ω,R2 = 20Ω,试求各个
电流和平均功率。
[解]
设U̇ = U ∠0◦ 为参考相量

图 11: 习题4.5.6图

U̇ 220∠0◦ 220∠0◦
I˙1 = = √ A= ◦
A = 11∠ − 60◦ A
R1 + jX1 10 + j10 3 20∠60

U̇ 220
I˙2 = = A = 11A
R2 20

I˙ = I˙1 + I˙2 = (11∠ − 60◦ + 11)A = 11 3∠ − 30◦ A
P = R1 I 21 + R2 I 22 = (10 × 112 + 20 × 112 )W = 3630W

4.5.7

在 图12(a)中 , 已 知u = 220 2 sin 314t V ,i1 = 22 sin(314t − 45◦ )A,i2 =

11 2 sin(314t + 90◦ )A,试求各仪表读数及电路参数R,L和C 。
[解]
由图12(b)的相量图可知


◦ 22 2
I2 = I1 sin 45 = √ × A = 11A
2 2
22
I1 = √ A = 15.6A
2
13
图 12: 习题4.5.7图

因I˙和U̇ 同相,故
s? ?2
q
22
I = I12 − I22 = √ − 112 A = 11A
2
U 220
XC = = Ω = 20Ω
I2 11
1 1
C = = F = 159µF
ωXC 314 × 20
U 220 √
|Z1 | = = Ω = 10 2Ω
I1 22

2

√ 2
R = |Z1 | cos 45◦ = 10 2 × Ω = 10Ω
2
XL = |Z1 | sin 45◦ = 10Ω
XL 10
L = = H = 0.0318H = 31.8mH
ω 314

4.5.11

在图13所示的电路中,已知U̇ C = 1∠0◦ V ,求U̇ 。


[解]

图 13: 习题4.5.11图

14
1∠0◦
I˙C = A = j0.5A
−j2
1∠0◦
I˙R = A = 0.5A
2
I˙ = I˙R + I˙C = (0.5 + j0.5)A
U̇1 = (2 + j2)(0.5 + j0.5)V = j2V
√ √
U̇ = U̇C + U̇1 = (1 + j2)V = 12 + 22 ∠ arctan 2V = 5∠63.4◦ V

4.5.12
1 = 10Ω,Z 为电
在图14所示的电路中,已知Uab = Ubc ,R = 10Ω,XC = ab
ωC
感性负载。试求U̇ 和I˙同相时Zab 等于多少?
[解]
令Zab = R1 + jXL

图 14: 习题4.5.12图

又求得
−jRXC −j10 × 10
Zbc = = Ω = (5 − j5)Ω
R − jXC 10 − j10
所以

U̇ = U̇ab + U̇bc = (Zab + Zbc )I˙


= [(R1 + jXL ) + (5 − j5)]I˙ = [(R1 + 5) + j(XL − 5)]I˙

若U̇ 和I˙同相,则上式的虚部必为零,即
XL − 5 = 0 XL = 5Ω
又因Uab = Ubc ,则|Zab | = |Zbc |
q √
R21 + XL2 = 52 + 52

解之得 R1 = 5Ω
于是 Zab = (5 + j5)Ω

15
4.7 交流电路的频率特性
4.7.4

有 一R,L,C 串 联 电 路 , 接 于 频 率 可 调 节 的 电 源 上 , 电 源 电 压U 保 持
在10V , 当 频 率 增 加 时 , 电 流 从10mA(500H z)增 加 到 最 大 值60mA(1000H z)。
试求(1)电阻R,电感L和电容C 的值;(2)在谐振时电容器两端的电压UC ;(3)谐
振时磁场中和电场中所储的最大能量。
[解]
(1) 谐振时 f0 = 1000H z
U 10
|Z0 | = R = = Ω = 166.67Ω
I0 60 × 10−3
非谐振时 f = 500H z
U 2 10
(XL − XC )2 = |Z |2 − R2 = ( ) − R2 = ( )2 − 166.672
I 10 × 10−3
XL − XC = ±985.4Ω

因f < f0 ,电路呈电容性,故取

XL − XC = −985.4Ω

列谐振时和非谐振时的方程,联立之
1
2πf0 L − =0
2πf0 C
1
2πf L − = −985.4
2πf C
1
6.28 × 1000L − =0
6.28 × 1000C
1
6.28 × 500L − = −985.4
6.28 × 500C
解之得

L = 0.105H C = 0.24µF

(2)
1
UC = XC I0 = × 60 × 10 −3 V = 39.56V
6.28 × 1000 × 0.24 × 10−6
(3)
1 1
W = WL + WC = LI02 + C UC2
2 2
1 1
= [ × 0.105 × 0.062 + × 0.24 × 10−6 × 39.562 ]J
2 2
= (1.87 × 10−4 + 1.88 × 10−4 )J = 3.8 × 10−4 J

16
4.7.5

在图15所示的电路中,R1 = 5Ω。今调节电容C 值使电流I 为最小,并此时测


得:I1 = 10A,I2 = 6A,Uz = 113V ,p = 1140W 。求阻抗Z 。
[解]
调C 值使I 最小,这时并联电路发生谐振,I˙与U̇ab 同相。由相量图可得

图 15: 习题4.7.5图

q √
I= I 12 − I 22 = 102 − 62 A = 8A

因为

P = RI 2 + R1 I12

所以
P − R1 I12 1140 − 5 × 102
R = = Ω = 10Ω
I2 82
√ U 113
|Z | = R2 + X 2 = Z = Ω = 14.1Ω
I 8
由此得
q
X = ± |Z |2 − R2 = ±10Ω
Z = R + jX = (10 ±j10)Ω

Z = (10 + j10)Ω 或Z = (10 − j10)Ω

4.7.6

电路如图16所示,已知R = R1 = R2 = 10Ω,L = 31.8mH ,C = 318µF ,


f = 50H z,U = 10V ,试求并联支路端电压Uab 及电路的P 、Q、S及cos ϕ.

17
图 16: 习题4.7.6图

[解]
感抗

XL = ωL = 314 × 31.8 × 10−3 Ω = 10Ω

容抗
1 1
XC = = Ω = 10Ω
ωC 314 × 318 × 10−6
并联支路的等效阻抗
Z1 Z2 (10 + j10)(10 − j10)
Zab = = Ω = 10Ω
Z1 + Z2 10 + j10 + 10 − j10
可见,并联支路呈电阻性,且其阻值与R相等,故
U 10
Uab = = V = 5V
2 2
并联支路电流
Uab U 5 1
I1 = I 2 = = ab = √ A = √ A
|Z1 | |Z2 | 10 2 2 2
总电流
Uab 5
I= = A = 0.5A
|Zab | 10
电路的P 、Q、S、cos ϕ

P = RI 2 + R1 I 21 + R2 I 22
" ? ?2 ? ? ? ?2 #
1 1 2 1
= 10 × + 10 × √ + 10 × √ W = 5W
2 2 2 2 2
" ? ?2 ? ?2 #
1 1
Q = XL I12 − XC I 22 = 10 × √ − 10 × √ var = 0
2 2 2 2
p √
S = 2 + Q2 = 52 + 0 V ·A = 5 V ·A
P 5
cos ϕ = = =1
S 5
可见,本电路处于谐振状态。

18
4.8 功率因数的提高
4.8.2

用 图17所 示 的 电 路 测 得 无 源 线 性 二 端 网 络N 的 数 据 如 下 :U = 220V ,I =
5A,P = 500W 。又知当与N 并联一个适当数值的电容C 后,电流I 减小,而其他
读数不变.试确定该网络的性质,等效参数及功率因数。f = 50H z.
[解]
因为与N 并联C 后,I 减小,所以N 是一电感性网络。

图 17: 习题4.8.2图


P = U I cos ϕ
得出
P 500
cos ϕ = = = 0.45
UI 220 × 5
ϕ = 63◦

又因为
U 220
|Z | = = Ω = 44Ω
I 5

Z = |Z | ∠ϕ = 44∠63◦ Ω = (20 + j39.2) Ω

其中

R = 20Ω XL = 39.2Ω

由此得

XL 39.2
L= = H = 125 × 10−3 H = 125mH
ω 314

19
4.8.3

在 图18中,U = 220V ,f = 50H z,R1 = 10Ω,X1 = 10 3Ω,R2 = 5Ω,X2 =

5 3Ω.(1)求 电 流 表 的 读 数I 和 电 路 功 率 因 数cos ϕ1 ;(2)欲 使 电 路 的 功 率 因 数 提
高到0.866,则需并联多大电容?(3)并联电容后电流表的读数为多少?
[解]

图 18: 习题4.8.3图

(1) 设U̇ = 220∠0◦ V


已知

Z1 = R1 + jX1 = (10 + j10 3)Ω = 20∠60◦ Ω

Z2 = R2 + jX2 = (5 + j5 3)Ω = 10∠60◦ Ω

于是得

U̇ 220∠0◦
I˙1 = = ◦
A = 11∠−60◦ A
Z1 20∠60
U̇ 220∠0◦
I˙2 = = A = 22∠−60◦ A
Z2 10∠60◦
I˙ = I˙1 + I˙2 = (11∠−60◦ + 22∠−60◦ )A = 33∠−60◦ A

电流表的读数为33A,电路功率因数cos ϕ1 = cos 60◦ = 0.5


(2)

P = U I cos ϕ1 = 220 × 33 × 0.5W = 3630W


cos ϕ = 0.866 ϕ = 30◦
P
C = (tan ϕ1 − tan ϕ)F
ωU 2
3630
= 2
(tan 60◦ − tan 30◦ )F
314 × 220
= 275.7µF

20
(3) 并联C 后

1 106
XC = = Ω = 11.6Ω
ωC 314 × 275.7
U̇ 220∠0◦ A = 18.96∠90◦ A
I˙C = =
−jXC −j11.6
I˙ = I˙1 + I˙2 + I˙C = (33∠−60◦ + 18.96∠90◦ )A
= (16.5 − j28.58 + j18.96)A = (16.5 − j9.62)A
= 19.1∠−30.2◦ A

电流表的读数为19.1A,减小了。

21
22
目录
第5章 三相电路 3
第5.2节 负载星 形联结的三相电路 . . . . . . . . . . . . . . . . . . . . . 3
第5.2.1题 . . . . . . . . . . . . . . . . . . . . . . . . . . . . . . . 3第
5.2.3题 . . . . . . . . . . . . . . . . . . . . . . . . . . . . . . . 4第
5.2.4题 . . . . . . . . . . . . . . . . . . . . . . . . . . . . . . . 5第
5.2.5题 . . . . . . . . . . . . . . . . . . . . . . . . . . . . . . . 6第
5.2.6题 . . . . . . . . . . . . . . . . . . . . . . . . . . . . . . . 7 第5.3
节 负载三角形联结的三相电路 . . . . . . . . . . . . . . . . . . . 8
第5.3.1题 . . . . . . . . . . . . . . . . . . . . . . . . . . . . . . . 8
第5.4节 三相功率 . . . . . . . . . . . . . . . . . . . . . . . . . . . . . . 9
第5.4.3题 . . . . . . . . . . . . . . . . . . . . . . . . . . . . . . . 9
第5.4.4题 . . . . . . . . . . . . . . . . . . . . . . . . . . . . . . . 10
第5.4.5题 . . . . . . . . . . . . . . . . . . . . . . . . . . . . . . . 11

1
List of Figures
1 习题5.2.1图 . . . . . . . . . . . . . . . . . . . . . . . . . . . . . . . . 3
2 习题5.2.1图 . . . . . . . . . . . . . . . . . . . . . . . . . . . . . . . . 4
3 习题5.2.3图 . . . . . . . . . . . . . . . . . . . . . . . . . . . . . . . . 4
4 习题5.2.4图 . . . . . . . . . . . . . . . . . . . . . . . . . . . . . . . . 5
5 习题5.2.5图 . . . . . . . . . . . . . . . . . . . . . . . . . . . . . . . . 6
6 习题5.2.6图 . . . . . . . . . . . . . . . . . . . . . . . . . . . . . . . . 7
7 习题5.3.1图 . . . . . . . . . . . . . . . . . . . . . . . . . . . . . . . . 8
8 习题5.3.1图 . . . . . . . . . . . . . . . . . . . . . . . . . . . . . . . . 8
9 习题5.4.3图 . . . . . . . . . . . . . . . . . . . . . . . . . . . . . . . . 9
10 习题5.4.4图 . . . . . . . . . . . . . . . . . . . . . . . . . . . . . . . . 10
11 习题5.4.4图 . . . . . . . . . . . . . . . . . . . . . . . . . . . . . . . . 10

2
5 三相电路

5.2 负载星形联结的三相电路

5.2.1

如图1所示的是三相四线制电路,电源线电压Ul = 380V 。三个电阻性负载联


成星形,其电阻为R1 = 11Ω,R2 = R3 = 22Ω.(2)如无中性线,求负载相电压及
中性点电压;(3)如无中性线,当L1 相短路时,求各相电压和电流,并做出它们的
相量图.
[解]

图 1: 习题5.2.1图

(2) 设U̇1 = U1 ∠0◦ = 220∠0◦ V

U̇1 U̇2 U̇3 220∠0◦ 220∠−120◦ 220∠120◦


+ + + +
R1 R2 R3 11 22 22
U̇N 0 N = = V = 55∠0◦ V
1 1 1 1 1 1
+ + + +
R1 R2 R3 11 22 22

U̇ 100 = U̇1 − U̇N 0 N = (220∠◦ − 55∠0◦ )V = 165∠0 ◦V



U̇ 20 = U̇2 − U̇N 0 N = (220∠ − 55∠0◦ )V = 252∠−131 ◦V
−120

30 = U̇3 − U̇N 0 N = (220∠


U̇ 120 ◦ − 55∠0◦ )V = 252∠131◦ V

(3) 如无中性线,L1 相短路,则各相负载电压为

U̇ 10 = 0

U̇ 20 =
= U̇ 0
3
3
U̇21 = −U̇12 = 380∠−150 V
U̇31 = 380∠150◦ V

4
各相负载电流为

U̇ 20 380∠−150◦
I˙2 = = A = 17.3∠−150◦ A
R2 22
˙0 ◦
˙I3 = U3 = 380∠150 A = 17.3∠150◦ A
R3 22
˙I1 = −(I˙2 + I˙3 ) = −(17.3∠−150◦ + 17.3∠150◦ )A = 30∠0◦ A

相量图如图2所示。

图 2: 习题5.2.1图

5.2.3

有一台三相发电机,其绕组联成星形,每相额定电压为220V 。在一次试验
时,用电压表量得相电压U1 = U2 = U3 = 220V ,而线电压则为U12 = U31 =
220V ,U23 = 380V ,试问这种现象是如何造成的?
[解]
由于发电机的L1 相绕组接反,电压相量图则如图3所示。可见U12 = U31 =

图 3: 习题5.2.3图

220V ,U23 = 380V .

5
5.2.4

在图4(a)所示的电路中,三相四线制电源电压为380/220V ,接有对称星形联
结的白炽灯负载,其总功率为180W .此外,在L3 相上接有额定电压为220V ,功
率 为40W ,功 率 因 数cos ϕ = 0.5的 日 关 灯 一 支 。 试 求 电 流I˙1 ,I˙2 ,I˙3 及I˙N 。 设U̇1 =
220∠0◦ V .
[解] 设

图 4: 习题5.2.4图

U̇1 = 220∠0◦ V
U̇2 = 220∠−120◦ V
U̇3 = 220∠120◦ V

每相白炽灯的功率
1
P = × 180W = 60W
3
每相白炽灯的电流

P 60
I= = A = 0.273A
UP 220

I˙1 = 0.273∠0◦ A
I˙2 = 0.273∠−120◦ A

I˙30 = 0.273∠120 A

日光灯的电流
P 40
I 00
=U = A = 0.364 A
3
P cos ϕ 220 × 0.5
6
因cos ϕ = 0.5,ϕ = 60◦ ,I˙300 比U̇3 滞后60◦ ,即比U̇1 超前60◦ ,故得

I˙300 = 0.364∠60 ◦A
I˙3 = I˙30 + I˙300 = (0.273∠120◦ + 0.364∠60◦ )A
= 0.553∠85.3◦ A

中性线电流

I˙N = I˙1 + I˙2 + I˙30 + I˙300


= I˙300 = 0.364∠60◦ A

电压和电流的相量图如图4(b)所示。

5.2.5

图5(a)是两相异步电动机(见第9章)的电源分相电路,O是铁心线圈的中心
抽头。试用相量图说明U̇ 12 和U̇o3 之间的相位差为90◦ 。
[解]
由图5(a)可见

图 5: 习题5.2.5图

1
U̇o3 = U̇13 − U̇1o = −U̇31 − U̇12
2
1
= U1 ∠−60◦ − Ul ∠0◦
2
1
= Ul (cos 60◦ − j sin 60◦ ) − Ul
2
= −j0.0866Ul = 0.866Ul ∠−90◦ V

7
5.2.6

图6(a)是小功率星形对称电阻性负载从单相电源获得三相对称电压的电路。
已知每相负载电阻R = 10Ω,电源频率f = 50H z,试求所需的L和C 的数值。
[解]
求解本题时,可先进行相量分析,而后计算。

图 6: 习题5.2.6图

按题意,U̇1 、U̇2 、U̇3 是三相对称电压,故可画出其相量图。


从图6(a)的电路图上可得

U̇L = U̇1 − U̇2 U̇C = U̇2 − U̇3

其相量如图6(b)所示.
I˙L 滞后于U̇L 90◦ ,I˙C 超前于U̇C 90◦ ,因此相量I˙L 和I˙C 也可画出。而后再画出I˙2 的相
量。
可见:
U̇ 、U̇L 、U̇C 构成一正三角形,故U = UL = UC ;
I˙2 、I˙L 、I˙C 构成一正三角形,故I2 = IL = IC .
于是

U
√ √
3 UL U
= = C 3R = XL = XC
R XL XC

√ 10 3
10 3 = 314L L= H = 0.055H = 55mH
314
√ 1 1
10 3 = C= √ F = 184 × 10−6 F = 184µF
314C 314 × 10 3

8
5.3 负载三角形联结的三相电路
5.3.1

在线电压为380V 的三相电源上,接两组电阻性对称负载,如图7所示,试求
线电流I 。
[解]

图 7: 习题5.3.1图

星形联结

220
IlY = IpY = A = 22A
10
三角形联结

380 √ √
Ip∆ = A = 10A, Il∆ = 3Ip∆ = 10 3A
38
由于U̇1 滞后于U̇12 30◦ ,I˙lY 与U̇1 同相,I˙p∆ 与U̇12 同相,而I˙l∆ 也滞后于I˙p∆ 30◦ ,故I˙l∆ 与I˙lY
同相,于是

I = IlY + Il∆ = (22 + 10 3)A = 39.3A

电压与电流的相量图如图8所示。

图 8: 习题5.3.1图

9
5.4 三相功率
5.4.3

在图9中,对称负载联成三角形,已知电源电压Ul = 220V ,电流表读数Il =


17.3A,三相功率P = 4.5kW ,试求:(1)每相负载的电阻和感抗;(2)当L1 L2 相断
开时,图中各电流表的读数和总功率P ;(3)当L1 线断开时,图中各电流表的读
数和总功率。
[解]

图 9: 习题5.4.3图

(1) 相电流
Il 17.3
IP = √ = √ A = 10A
3 3
P 4.5 × 103
cos ϕ = √ =√ = 0.683
3Ul Il 3 × 220 × 17.3
Up 220
|Z | = = Ω = 22Ω
Ip 10
R = |Z | cos ϕ = 22 × 0.683Ω = 15Ω
p √
XL = |Z | sin ϕ = |Z | 1 − cos ϕ2 = 22 × 1 − 0.6832 Ω = 16.1Ω

(2) 当L1 L2 相断开时

I1 = I2 = 10A I3 = 17.3A P = 3kW

10
(3) 当L1 线 断 开 时 ,Z12 与Z31 串 联 在 电 压U˙23 上 , 其 中 流 过 电 流 为 A =
2
5A,且与L2 L3 相中的电流(10A)同相,于是得出

I1 = 0 I2 = I3 = (5 + 10)A = 15A

总功率
1
P = Ip2 R + ( Ip )2 ·2R = (102 × 15 + 52 × 2 × 15)W = 2250W
2

10
5.4.4

在图10所示的电路中,电源线电压Ul = 380V ,频率f = 50H z,对称电感


性 负 载 的 功 率P = 10kW , 功 率 因 数cos ϕ1 = 0.5。 为 了 将 线 路 功 率 因 数 提 高
到cos ϕ = 0.9,试问在两图中每相并联的补偿电容器的电容值各为多少?采用哪
种联结(三角形或星形)方式较好?
[解]
由图11所示的功率三角形可知,所需补偿电容器的无功功率为

图 10: 习题5.4.4图

图 11: 习题5.4.4图

Q = P tan ϕ1 − P tan ϕ = P (tan ϕ1 − tan ϕ)

(1) 电容器三角形联结流过电容器的相电流为

Ipc = ωC Ul

11
线电流
√ √
Ilc = 3Ipc = 3ωC Ul

电容器的无功功率为
√ √ √
Q= 3Ul Ilc = Ul 3ωC Ul = 3ωC U l2

于是得每相电容器的电容
Q
C=
3ωU l2

在本题中

cos ϕ1 = 0.5, ϕ1 = 60◦ , tan ϕ1 = 1.73


cos ϕ = 0.9, ϕ = 26◦ , tan ϕ = 0.48
Q = P (tan ϕ1 − tan ϕ) = 10 × 103 (1.73 − 0.48)var = 12.5 × 103 var

由此得
Q 12.5 × 103
C= 2
= 2
F = 92 × 10−6 F = 92µF
3ωU l 3 × 314 × 380

(2) 电容器星形联结
与(1)不同者,在图10(b)中每相电容器上加的是电源相电压,因此每相电

Q 12.5 × 103
C= = F = 274 × 10−6 F = 274µF
3ωU p2 3 × 314 × 2202

所用电容量比三角形联结时要大三倍,故提高三相电感性电路的功率因
数时,电容器常联结成三角形。

5.4.5

如果电压相等,输送功率相等,距离相等,线路功率损耗相等,则三相输电
3
线(设负载对称)的用铜量为单相输电线的用铜量的 。试证明之。
4
[解]
因为输送功率相等,即

3U3 I3 cos ϕ = U1 I1 cos ϕ

则得

I1 = 3I3

12
因为线路功率损耗相等,即
3I 23 R3 = 2I12 R1
则得
1
R1 = R3 或S1 = 2S3
2
因此,用铜量之比为

(用铜量)3 3S3 lρ 3
= =
(用铜量)1 2S1 lρ 4

上 列 各 式 中 ,R是 每 根 输 电 线 的 电 阻 ,S是 输 电 线 的 截 面 积 ,l是 输 电 线 的 长


度,ρ是导线材料的密度。

13
14
目录
第6章 磁路与铁心线圈电路 3
第6.1节 磁路及其分析方法 . . . . . . . . . . . . . . . . . . . . . . . . . 3
第6.1.1题 . . . . . . . . . . . . . . . . . . . . . . . . . . . . . . . 3第
6.1.2题 . . . . . . . . . . . . . . . . . . . . . . . . . . . . . . . 3第
6.1.3题 . . . . . . . . . . . . . . . . . . . . . . . . . . . . . . . 3第
6.1.4题 . . . . . . . . . . . . . . . . . . . . . . . . . . . . . . . 4 第6.2
节 交流铁心线圈电路 . . . . . . . . . . . . . . . . . . . . . . . . . 4
第6.2.3题 . . . . . . . . . . . . . . . . . . . . . . . . . . . . . . . 4
第6.3节 变压器 . . . . . . . . . . . . . . . . . . . . . . . . . . . . . . . 5
第6.3.2题 . . . . . . . . . . . . . . . . . . . . . . . . . . . . . . . 5
第6.3.4题 . . . . . . . . . . . . . . . . . . . . . . . . . . . . . . . 6
第6.3.6题 . . . . . . . . . . . . . . . . . . . . . . . . . . . . . . . 7
第6.4节 电磁铁 . . . . . . . . . . . . . . . . . . . . . . . . . . . . . . . 7
第6.4.1题 . . . . . . . . . . . . . . . . . . . . . . . . . . . . . . . 7

1
List of Figures
1 习题6.3.4图 . . . . . . . . . . . . . . . . . . . . . . . . . . . . . . . . 6
2 习题6.3.6图 . . . . . . . . . . . . . . . . . . . . . . . . . . . . . . . . 7

2
6 磁路与铁心线圈电路
6.1 磁路及其分析方法

6.1.1

有 一 线 圈 , 其 匝 数N = 1000,绕 在 由 铸 钢 制 成 的 闭 合 铁 心 上 , 铁 心 的 截
面 积SF e = 20cm2 ,铁 心 的 平 均 长 度lF e = 50cm,如 要 在 铁 心 中 产 生 磁 通Φ =
0.002W b,试问线圈中应通入多大的直流电流?
[解]
先计算磁感应强度
Φ 0.002
B = = T = 1T
SF e 20 × 10−4

而后查铸钢的磁化曲线(教材图6.1.5),得出

H = 0.7 × 103 A/m

于是可得
H lF e 0.7 × 103 × 50 × 10−2
I= = A = 0.35A
N 1000

6.1.2

如果上题的铁心中含有一长度为δ = 0.2cm的空气隙(与铁心柱垂直),由
于空气隙较短,磁通的边缘扩散可忽略不计,试问线圈中的电流必需多大才可
使铁心中的磁感应强度保持上题中的数值?
[解]
B0
N I = H l F e + H 0 δ = H lF e + δ
µ0
1
= (0.7 × 103 × 50 × 10−2 + −7
× 0.2 × 10−2 )A = 1942A
4π × 10
NI 1942
I = = A = 1.94A
N 1000

6.1.3

在题6.1.1中,如将线圈中电流调到2.5A,试求铁心中的磁通。
[解]
磁场强度
NI 1000 × 2.5
H = = A/m = 5000A/m
lF e 50 × 10−2

3
从教材图6.1.5所示的磁化曲线b查得相应得磁感应强度
B = 1.6T
铁心中的磁通

Φ = BSF e = 1.6 × 20 × 10−4 W b = 0.0032W b

6.1.4

有一铁心线圈,试分析铁心中的磁感应强度、线圈中的电流和铜损RI 2 ,在
下列几种情况下将如何变化:

(1) 直 流 励 磁 - 铁 心 截 面 积 加 倍 , 线 圈 的电 阻 和 匝 数 以 及 电 源 电 压 保 持 不
变;
(2) 交流励磁-同(1)
(3) 交流励磁-频率和电源电压的大小减半。 假设在上述各种情况下工作点
在磁化曲线的直线段。在交流励磁的情况下,设 电源电压与感应电动势在数
值上近于相等,且忽略磁滞和涡流。铁心是闭合 的,截面均匀。
[解]
(1) 电流和铜损不变,即
U
I= ∆PC u = RI 2
R
l
由Rm = 可知,磁阻Rm 减半,而磁通势N I 不变,故磁通
µS
NI
Φ=
Rm
加倍,磁感应强度
Φ
B =
S
不变。
(2) 由U ≈ 4.44f N Φm = 4.44f N Bm S可 知 , 铁 心 中 的 磁 感 应 强 度 的 最 大
值Bm 减半。 因工作点在磁化曲线的直线段,H 与B成正比,故Hm 也减

半。由 2N I =
1
Hm l可知,线圈中电流I 减半,而铜损RI 2 则减小到原来的 。
(3) Bm 、I 及RI 2 均不变。 4

6.2 交流铁心线圈电路

6.2.3

将 一 铁 心 线 圈 接 于 电 压U = 100V 、 频 率f = 50H z的 正 弦 电 源 上 , 其 电

4
流I1 = 5A,cos ϕ1 = 0.7。若将此线圈中铁心抽出,再接于上述电源上,则线圈
中I2 = 10A,cos ϕ2 = 0.05。试求此线圈在具有铁心时的铜损和铁损。
[解]
线圈有铁心时的功率:

P1 = U I1 cos ϕ1 = 100 × 5 × 0.7W = 350W

线圈无铁心时的功率:

P2 = U I2 cos ϕ2 = 100 × 10 × 0.05W = 50W

由此可求线圈的电阻:
P2 50
R = 2 =
Ω = 0.5Ω
I2 102

线圈有铁心时的铜损:

∆PC u = RI12 = 0.5 × 52 W = 12.5W

线圈有铁心时的铁损:

∆PF e = P1 − ∆PC u = (350 − 12.5)W = 337.5W

6.3 变压器

6.3.2

SJ L型三相变压器铭牌数据如下:SN = 180kV ·A,U1N = 10kV ,U2N =


400V ,f = 50H z,联接Y /Y0 。已知每匝线圈感应电动势为5.133V ,铁心截面积
为160cm2 。试求:(1)一次、二次绕组的匝数;(2)变压比;(3)一次、二次绕组
的额定电流;(4)铁心中磁感应强度Bm 。
[解]
电路见教材图6.3.4(a)
(1)

U1N 10 × 103
Up1 = √ = √ V = 5.78 × 103V
3 3
Up1 5.78 × 103
N1 = = 匝 = 1126匝
e 5.133
U2N 400
Up2 = √ = √ V = 231V
3 3
Up2 231
N2 = = 匝 = 45匝
e 5.133

5
(2)

U1N 10 × 103
K = = = 25
U2N 400
(3)

SN 180 × 103
I1N = √ =√ A = 10.4A
3U1N 3 × 10 × 103
SN 180 × 103
I2N = √ =√ A = 260A
3U2N 3 × 400
(4)

Up1 5.78 × 103


Bm = = T
4.44f N1 S 4.44 × 50 × 1126 × 160 × 10−4
= 1.45T

6.3.4

在图1中,输出变压器的二次绕组有中间抽头,以便接8Ω或3.5Ω的扬声器,
N2
两者都能达到阻抗匹配。试求二次绕组两部分匝数之比 。
N3
[解]

图 1: 习题6.3.4图

? ?2 ? ?2
N1 N1
×8= × 3.5
N2 + N3 N3
? ?2
N2 + N3 8
=
N3 3.5
? ?2
N2 8
+1 =
N3 3.5
r
N2 8 1
= − 1 ≈ 1.5 − 1 =
N3 3.5 2

6
6.3.6

图2所示的是一电源变压器,一次绕组有550匝,接220V 电压。二次绕组有
两个:一个电压36V 、负载36W ;一个电压12V 、负载24W 。两个都是纯电阻负
载。试求一次电流I1 和两个二次绕组的匝数。
[解]

图 2: 习题6.3.6图

(1)
N1 U1 U2 36
= N2 = N1 = × 550匝 = 90匝
N2 U2 U1 220
N1 U1 U3 12
= N3 = N1 = × 550匝 = 30匝
N3 U3 U1 220
(2)
P2 36
I2 = = A = 1A
U2 36
P3 24
I3 = = A = 2A
U3 12
(3) 因i2 和i3 相位相同,故用下式计算I1 :
N2 I2 N3 I3 90 × 1 30 × 2
I1 = + =( + )A
N1 N1 550 550
= (0.16 + 0.11)A = 0.27A

6.4 电磁铁
6.4.1

有一交流接触器C J 0 − 10A,其线圈电压为380V ,匝数为8750匝,导线直径


为0.09mm。今要用在220V 电源上,问应如何改装?即计算线圈匝数和换用直径
为多少毫米的导线。
[解]

7
(1)改装前后吸力不变,磁通最大值Φm 应保持不变;(2)Φm 保持不变,改装前后
磁通势应该相等;(3)电流与导线截面积成正比。
由U ≈ 4.44f N Φm 可知,当Φm 与f 保持不变时,U 和N 成正比,于是改装后线圈
的匝数为
220 × 8750匝 = 5066匝
N =
380
改装后的电流为I ,改装前的电流为I1 ,而电流正比于导线截面,于是得出

N I = 8750I1
d2 N = (0.09)2 × 8750
(0.09)2 × 8750
d2 = = 0.014
5066
导线直径 d = 0.12mm

8
9
目录
第7章 交流电动机 2
第7.3节 三相异步电动机的电路分析 . . . . . . . . . . . . . . . . . . . 2
第7.3.2题 . . . . . . . . . . . . . . . . . . . . . . . . . . . . . . . 2 第7.4
节 三相异步电动机的转矩与机械特性 . . . . . . . . . . . . . . . 2
第7.4.1题 . . . . . . . . . . . . . . . . . . . . . . . . . . . . . . . 2第
7.4.2题 . . . . . . . . . . . . . . . . . . . . . . . . . . . . . . . 3第
7.4.4题 . . . . . . . . . . . . . . . . . . . . . . . . . . . . . . . 4 第7.5
节 三相异步电动机的起动 . . . . . . . . . . . . . . . . . . . . . . 5
第7.5.2题 . . . . . . . . . . . . . . . . . . . . . . . . . . . . . . . 5

1
7 交流电动机
7.3 三相异步电动机的电路分析

7.3.2

有一台四极,50H z,1425r/min的三相异步电动机,转子电阻R2 = 0.02Ω,


感抗X20 = 0.08Ω,E1 /E20 = 10,当E1 = 200V 时,试求:(1)电动机起动初始瞬
间(n = 0, s = 1)转子每相电路的电动势E20 ,电流I20 和功率因数cos ϕ20 ;(2)额定
转速时的E2 ,I2 和cos ϕ2 。比较在上述两种情况下转子电路的各个物理量(电动
势、频率、感抗、电流及功率因数)的大小。
[解]
(1) 起动初始瞬间
E1 200
E20 = = V = 20V
10 10
E20 20
I20 = p 2 2
= A = 243A
.02 2 + 0.082

2 + X20
R2 0.02
cos ϕ20 = p 2 = = 0.243
2√ .022 + 0.082
2 + X20

(2) 额定转速时

1500 − 1425
SN = = 0.05
1500
E2 = sE20 = 0.05 × 20V = 1V
E2 1
I2 = p =p A = 49A
2 + (sX )2
2 20 0.02 + (0.05 × 0.08)2
2

R2 0.02
cos ϕ2 = p =p = 0.98
2
2 + (sX20 )
2 0.022 + (0.05 × 0.08)2
f2 = sf1 = 0.05 × 50H z = 2.5H z

7.4 三相异步电动机的转矩与机械特性

7.4.1

已知Y 100L1 − 4型异步电动机的某些额定技术数据如下:

2.2kW 380V 星形接法


1420r/min cos ϕ = 0.82 η = 81%

试计算:(1)相电流和线电流的额定值及额定负载时的转矩;(2)额定转差率及额
定负载时的转子电流频率。设电源频率为50H z.
[解]
2
(1) 线电流的额定值
P2N 2.2 × 103
I1N = √ cos ϕ · = A = 5A
l √ 380 × 0.82 × 0.81
η 3×
3U
相电流的额定值

IpN = IlN = 5A

额定转矩
P2N 2.2
TN = 9550 = 9550 × N · m = 14.8N · m
nN 1420
(2) 额定转差率和额定负载时的转子电流频率
1500 − 1420
sN = = 0.053
1500
f2 = sN f1 = 0.053 × 50H z = 2.65H z

7.4.2

有台三相异步电动机,其额定转速为1470r/min,电源频率为50H z。在a.起
2
动瞬间,b.转子转速为同步转速的 时,c.转差率为0.02时三种情况下,试求:
3
(1) 定子旋转磁场对定子的转速n1−1 ;

(2) 定子旋转磁场对转子的转速n1−2 ;

(3) 转子旋转磁场对转子的转速n2−2 ;

(4) 转子旋转磁场对定子的转速n2−1 ;

(5) 转子旋转磁场对定子旋转磁场的转速。
[解]
转子旋转磁场和定子旋转磁场是按同一方向在空间旋转。
60f1
定子旋转磁场对定子(即对空间)的相对转速n1−1 = n0 = ,决定于定子电
p
流的频率f1 ;同理,转子旋转磁场对转子的相对转速n2−2 决定于转子电流的频
率f2 ,即
60f2 60sf1
n2−2 = = = sn0
p p
但转子本身是以n的转速在空间转动(转动方向与磁场旋转方向相同),于是转
子旋转磁场对定子的相对转速为
n2−1 = n2−2 + n = sn0 + (1 − s)n0 = n0
3
由此可见,不论转子转速n等于多大,转子旋转磁场和定子旋转磁场总是以同一
转速在空间旋转着。两者的相对转速为零。
a. n = 0, s = 1
60f1 60 × 50
(1) n1−1 = n0 = = r/min = 1500r/min
p 2
(2) n1−2 = n0 − n = (1500 − 0)r/min = 1500r/min
(3) n2−2 = sn0 = 1500r/min
(4) n2−1 = n0 = 1500r/min
(5) 0
2 1500 − 1000 1
b. n = × 1500r/min = 1000r/min, s = =
3 1500 3
(1) n1−1 = n0 = 1500r/min
(2) n1−2 = n0 − n = (1500 − 1000)r/min = 500r/min
1
(3) n2−2 = sn0 = × 1500r/min = 500r/min
3
(4) n2−1 = n0 = 1500r/min
(5) 0
c. s = 0.02, n = (1 − s)n0 = 0.98 × 1500r/min = 1470r/min
(1) n1−1 = n0 = 1500r/min
(2) n1−2 = n0 − n = (1500 − 1470)r/min = 30r/min
(3) n2−2 = sn0 = 0.02 × 1500r/min = 30r/min
(4) n2−1 = n0 = 1500r/min
(5) 0

7.4.4

已知Y 132S − 4型三相异步电动机的额定技术数据如下:


功率 转速 电压 效率 功率因数 Ist /IN Tst /TN Tmax /TN
5.5kW 1440r/min 380V 85.5% 0.84 7 2.2 2.2
电源频率为50H z。试求额定状态下的转差率sN ,电流IN 和转矩TN ,以及起动
电流Ist ,起动转矩Tst ,最大转矩Tmax 。
[解]
目前4 ∼ 100kW 的异步电动机都已设计为380V 三角形联结,所以连接方式是
知道的。但计算本题时毋须知道是何种联结。
(1) 因为nN = 1440r/min,可判定同步转速

n0 = 1500r/min

则磁极对数为
60f1 60 × 50
p= = =2
n0 1500

4
额定转差率

n0 − n 1500 − 1440
sN = = = 0.04
n0 1500
(2) 额定电流指定子线电流,即

P2 5.5 × 103
IN = √ = A = 11.6A
N η cos ϕ 1.73 × 380 × 0.84 × 0.855
3U
(3) 额定转矩
P2 5.5
TN = 9550 = 9550 × N · m = 36.5N · m
nN 1440
注意,式中P2 的单位为kW
(4) 起动电流
Ist
Ist = ( )IN = 7 × 11.6A = 81.2A
IN
(5) 起动转矩

Tst
Tst = ( )TN = 2.2 × 36.5N · m = 80.3N · m
TN
(6) 最大转矩
Tmax )T = 2.2 × 36.5N · m = 80.3N · m
Tmax = ( N
TN

7.5 三相异步电动机的起动
7.5.2

某四极三相异步电动机的额定功率为30kW ,额定电压为380V ,三角形连


接,频率为50H z。在额定负载下运行时,其转差率为0.02,效率为90%,线电
T I
流为57.5A,并已知 st = 1.2, st = 7。如果采用自耦变压器降压起动,而使电
TN IN
动机的起动转矩为额定转矩的85%,试求:(1)自耦变压器的变比;(2)电动机的
起动电流和线路上的起动电流各为多少?
[解]
电动机的额定转速

nN = (1 − s)n0 = (1 − 0.02) × 1500r/min = 1470r/min

额定转矩
30
TN = 9550 × N · m = 194.9N · m
1470
5
(1) 采用自耦变压器降压起动,而使起动转矩为额定转矩的85%,即

Tst0 = 194.9 × 85%N · m = 165.7N · m

而直接起动时

Tst = 1.2TN = 1.2 × 194.9N · m = 233.9N · m

因起动转矩与电压的平方成正比,故变压器的变比为
s r
Tst 233.9
K = 0
= = 1.19
Tst 165.7

(2) 电动机的起动电流即为起动时自耦变压器二次侧的电流

0 Ist 7 × 57.5
Ist = = A = 338.2A
K 1.19
线路上的起动电流即为起动时自耦变压器一次侧的电流

00 Ist0 338.2
Ist = = A = 284.2A
K 1.19

6
7
目录
第10章 继电接触器控制系统 3
第10.2节 笼型电动机直接起动的控制线路 . . . . . . . . . . . . . . . . 3
第10.2.1题 . . . . . . . . . . . . . . . . . . . . . . . . . . . . . . 3第
10.2.3题 . . . . . . . . . . . . . . . . . . . . . . . . . . . . . . 3第
10.4节 行程控制 . . . . . . . . . . . . . . . . . . . . . . . . . . . . . 4
第10.4.1题 . . . . . . . . . . . . . . . . . . . . . . . . . . . . . . 4第
10.4.2题 . . . . . . . . . . . . . . . . . . . . . . . . . . . . . . 4第
10.5节 时间控制 . . . . . . . . . . . . . . . . . . . . . . . . . . . . . 5
第10.5.1题 . . . . . . . . . . . . . . . . . . . . . . . . . . . . . . 5
第10.5.2题 . . . . . . . . . . . . . . . . . . . . . . . . . . . . . . 5

1
List of Figures
1 习题10.2.1图 . . . . . . . . . . . . . . . . . . . . . . . . . . . . . . . . 3
2 习题10.4.2图 . . . . . . . . . . . . . . . . . . . . . . . . . . . . . . . . 4
3 习题10.4.2图 . . . . . . . . . . . . . . . . . . . . . . . . . . . . . . . . 5
4 习题10.5.1图 . . . . . . . . . . . . . . . . . . . . . . . . . . . . . . . . 6
5 习题10.5.1图 . . . . . . . . . . . . . . . . . . . . . . . . . . . . . . . . 6
6 习题10.5.1图 . . . . . . . . . . . . . . . . . . . . . . . . . . . . . . . . 6
7 习题10.5.1图 . . . . . . . . . . . . . . . . . . . . . . . . . . . . . . . . 6
8 习题10.5.2图 . . . . . . . . . . . . . . . . . . . . . . . . . . . . . . . . 7
9 习题10.5.2图 . . . . . . . . . . . . . . . . . . . . . . . . . . . . . . . . 7

2
10 继电接触器控制系统
10.2 笼型电动机直接起动的控制线路

10.2.1

试画出三相笼型电动机既能连续工作,又能点动工作的控制线路。
[解]
在图1中,SB2 是连续工作的起动按钮。SB3 是双联按钮,用于点动工作。

图 1: 习题10.2.1图

按下SB3 时,K M 通电,主触点闭合,电动机起动。因SB3 的常闭触点同时断


开,无自锁作用。松开SB3 ,K M 断电,电动机停车。

10.2.3

根据教材图10.2.2接线做实验时,将开关Q合上后按下起动按钮SB2 ,发现
有下列现象,试分析和处理故障:(1)接触器K M 不动作;(2)接触器K M 动作,
但电动机不转动;(3)电动机转动,但一松手电机就不转;(4)接触器动作,但
吸合不上;(5)接触器触点有明显颤动,噪声较大;(6)接触器线圈冒烟甚至烧
坏;(7)电动机不转动或者转的较慢,并有“嗡嗡”声。
[答]
(1) 接触器K M 不动作的故障原因可能有下列几种:
(a) 三相电源无电;
(b) 有关相中熔断器的熔丝已断,控制电路不通电;
(c) 热继电器F R的动断触点动作后未复位;
(d) 停止按钮SB1 接触不良;
(e) 控制电路中电器元件的接线端接触不良或连接导线端有松动。
(2) 接触器K M 动作,但电动机不转动的故障原因可能有下列几种(问题不在
控制电路,应查主电路);
(a) 接触器的主触点已损坏;

3
(b) 从接触器主触点到电动机之间的导线有断线处或接线端接触不良;
(c) 电动机已损坏。
(3) 电动机转动,但一松手电机就不转,其原因是自锁触点未接上或该段电路
有断损和接触不良之处。
(4) 接触器动作,但吸和不上,主要由于电压过低,也可能因某种机械障碍造
成的。
(5) 接触器触点有明显颤动,主要由于铁心端面的短路环断裂所致,也可能由
于电压过低,吸力不够。
(6) 接触器线圈冒烟甚至烧坏,其原因有:
(a) 电压过高;
(b) 由于(4)中原因接触器吸和不上,导致线圈过热而烧坏。
(7) 电动机不转动或转的较慢,并有“嗡嗡”声,这是由于某种原因而造成电动
机单相起动所致。

10.4 行程控制

10.4.1

将教材图10.4.2(b)的控制电路怎样改一下,就能实现工作台自动往复运动?
[解]
只需将原位行程开关SQa 的动合触点与正转按钮SBF 并联即可。当电动机
反转到达原位时,挡块压下SQa ,不仅将它的动断触点压开,使电动机停止反
转,同时将上述动合触点压合,使电动机正转,接着就实现工作台自动往复运
动。按下停止按钮SB1 ,电动机才停车。

10.4.2

在图2中,要求按下起动按钮后能顺序完成下列动作:(1)运动部件A从1到2;(2)接
着B从3到4;(3)接 着A从2回 到1;(4)接 着B从4回 到3。 试 画 出 控 制 线 路 。 ( 提 示:
用四个行程开关,装在原位和终点,每个有一动合触点和一动断触点。)
[解]
电动机M1 和M2 的主电路按教材图10.3.2正反转连接,其正反转接触器分别

图 2: 习题10.4.2图

4
图 3: 习题10.4.2图

为K M1F ,K M1R 和K M2F ,K M2R 。控制电路如图3所示。

10.5 时间控制

10.5.1

根据下列要求分别绘出控制电路(M1 和M2 都是三相笼型电动机); (2)


电动机M1 先起动后,M2 才能起动,且M2 能点动; (4)M1 先起动,经过一定
延时后M2 能自行起动,M2 起动后,M1 立即停车; (5)起动时,M1 起动后
M2 才能起动;停止时,M2 停止后M1 才能停止。
[解] 图4是电动机M1 和M2 的主电路。图5,图6和图7分别为习题10.5.1(2).(4).(5)
的 控制线路。

10.5.2

试绘出笼型电动机定子串联电阻降压起动的控制线路。
[解]
主电路和控制电路如图8所示。
控制电路接上电源−→ K T 通电−→动断延时闭合触点瞬时断开
(
电动机串联电阻R降压起动
按SB2 −→ K M1 通电 延时
K T 断电 −−→ K M2 通电(起动完毕)

5
图 4: 习题10.5.1图 图 5: 习题10.5.1图

图 6: 习题10.5.1图 图 7: 习题10.5.1图

6
图 8: 习题10.5.2图

这里要注意时间继电器的动作:

图 9: 习题10.5.2图

(1)它是断电延时的; (2)它的动断延时闭合触点的
动作顺序。 本题也可采用通电延时,其控制电路
如图9所示。

7
8
目录
第14章 二极管和晶体管 3
第14.3节 二极管 . . . . . . . . . . . . . . . . . . . . . . . . . . . . . . 3
第14.3.2题 . . . . . . . . . . . . . . . . . . . . . . . . . . . . . . 3
第14.3.5题 . . . . . . . . . . . . . . . . . . . . . . . . . . . . . . 4
第14.4节 稳压二极管 . . . . . . . . . . . . . . . . . . . . . . . . . . . . 5
第14.4.2题 . . . . . . . . . . . . . . . . . . . . . . . . . . . . . . 5
第14.5节 晶体管 . . . . . . . . . . . . . . . . . . . . . . . . . . . . . . 5
第14.5.1题 . . . . . . . . . . . . . . . . . . . . . . . . . . . . . . 5
第14.5.3题 . . . . . . . . . . . . . . . . . . . . . . . . . . . . . . 6
第14.5.4题 . . . . . . . . . . . . . . . . . . . . . . . . . . . . . . 7

1
List of Figures
1 习题14.3.2图 . . . . . . . . . . . . . . . . . . . . . . . . . . . . . . . . 3
2 习题14.3.2图 . . . . . . . . . . . . . . . . . . . . . . . . . . . . . . . . 3
3 习题14.3.5图 . . . . . . . . . . . . . . . . . . . . . . . . . . . . . . . . 4
4 习题14.4.2图 . . . . . . . . . . . . . . . . . . . . . . . . . . . . . . . . 5
5 习题14.5.3图 . . . . . . . . . . . . . . . . . . . . . . . . . . . . . . . . 6
6 习题14.5.4图 . . . . . . . . . . . . . . . . . . . . . . . . . . . . . . . . 7

2
14 二极管和晶体管

14.3 二极管

14.3.2

在图1所示的各电路图中,E = 5V ,ui = 10 sin ω tV ,二极管D的正向压降


可忽略不计,试分别画出输出电压u0 的波形。
[ 解]

图 1: 习题14.3.2图

(a) ui 为正半周时,ui > E,D导通;ui < E,D截止。ui 为负半周时,D截


止。
D导通时,u0 = E;D截止时,uo = ui 。
(b) ui 为 正 半 周 时 ;ui > E,D导 通 ;ui < E,D截 止 。ui 为 负 半 周 时 ,D截
止。
D导通时,u0 = ui ;D截止时,u0 = E。
u0 的波形分别如图2(a)和(b)所示。

图 2: 习题14.3.2图

3
14.3.5

在图3中,试求下列几种情况下输出端电位VY 及各元件中通过的电流。(1)VA =
+10V ,VB = 0V ;(2)VA = +6V ,VB = +5.8V ;(3)VA = VB = +5V .设二极管的正
向电阻为零,反向电阻为无穷大。
[解]

图 3: 习题14.3.5图

(1) 二极管DA 优先导通,则

10
VY = 9 × V = 9V
1+9
VY 9
IDA = IR = = A = 1 × 10 −3 A = 1mA
R 9 × 10 3

DB 反向偏置,截止,IDB = 0
(2) 设DA 和DB 两管都导通,应用结点电压法计算VY :

6 5.8
+ 11.8 × 9
VY = 1 1 V = V = 5.59V < 5.8V
1 1 1 19
+ +
1 1 9

可见DB 管也确能导通。

6 − 5.59
IDA = A = 0.41 × 10−3A = 0.41mA
1 × 10 3
5.8 − 5.59
IDB = A = 0.21 × 10−3A = 0.21mA
1 × 103
5.59
IR = A = 0.62 × 10−3 A = 0.62mA
9 × 103

4
(3) DA 和DB 两管都能导通

5 5
+
VY = 1 1 V = 4.74V
1 1 1
+ +
1 1 9
VY 4.74
IR = = A = 0.53 × 10 −3 A = 0.53mA
R 9 × 103
IR 0.53
IDA = IDB = = mA = 0.26mA
2 2

14.4 稳压二极管
14.4.2

有两个稳压二极管DZ 1 和DZ 2 ,其稳定电压分别为5.5V 和8.5V ,正向压降都


是0.5V 。如果要得到3V 的稳定电压,应如何连接?
[解]
应按图4(a)或(b)连接,UZ = 3V 。图中R1 、R2 是限流电阻。

图 4: 习题14.4.2图

14.5 晶体管

14.5.1

有两个晶体管分别接在电路中,今测得它们管脚的电位(对“地”)分别如
下表所列:

晶体管I 晶体管II
管脚 1 2 3 管脚 1 2 3
电位/V 4 3.4 9 电位/V − 6 − 2.3 − 2
试判别管子的三个电极,并说明是硅管还是锗管?是N P N 型还是P N P 型?
[解]

5
N P N 型:集电极电位最高,发射极电位最低,UBE > 0;P N P 型;发射极电位
最高,集电极电位最低,UBE < 0。
硅管:基极电位与发射极电位大约相差0.6V 或0.7V ;锗管:基极电位与发射极
电位大约相差0.2V 或0.3V 。
由此可知:
晶体管I:N P N 型,硅管,1 − B、2 − E、3 − C ;
晶体管II :P N P 型,锗管,1 − C 、2 − B、3 −
E。

14.5.3

如何用万用表判断出一个晶体管是N P N 型还是P N P 型?如何判断出管子


的三个管脚?锗管或硅管又如何通过实验区别出来?
[解]
(1) 先判断基极
将插入万用表“-”(实为表内电源正极)插孔的测试笔轮流接任一管脚,
而后将另一测试笔分别接另外两个管脚,如果两次测得管脚间的电阻同
为低电阻(BE极间和BC 极间的P N 结上加正向电压)或同为高电阻(上
述极间的P N 结上加反向电压),则接万用表“-”插孔的是基极。
(2) 判断是N P N 型管还是P N P 型管 在(1)中,测得管脚间的电阻同为低电阻
时,则为N P N 型管;测得同为高 电阻时,则为P N P 型。
(3) 判断集电极
对 已 知 的N P N 型 管 或P N P 型 管 照 图5所 示 的 两 种 方 法 接 线 , 未 知 管

图 5: 习题14.5.3图 脚1和2用测试笔分别接万用表

的“+”,“-”插孔(注意,“-”插孔接表内电

6
源的正极),比较两种接法1,2管脚间的电阻高低。对N P N 型管,电阻
较低时接“-”插孔的是集电极;对P N P 型管,电阻较低时接“+”插孔的是
集电极。
(4) 判断是锗管还是硅管
B,E极间正向压降在0.6 ∼ 0.7V 时为硅管;在0.2 ∼ 0.3V 时为锗管。

14.5.4

图 6: 习题14.5.4图

在图6所示的各个电路中,试问晶体管工作于何种状态?设UBE = 0.6V 。
[解]
计算结果见下表。

(a)管 (b)管 (c)管


UC C
IC (sat) ≈ 12mA 8mA
RC
IC (sat)
IB0 = 0.24mA 0.2mA
β
6 − 0.6 12 − 0.6
IB mA = 0.11mA mA = 0.24mA IB < 0
50 47
状态 0
IB < I B 放大 IB > I B0 饱和 截止

7
8
目录
第15章 基本放大电路 3
第15.2节 放大电路的静态分析 . . . . . . . . . . . . . . . . . . . . . . 3
第15.2.3题 . . . . . . . . . . . . . . . . . . . . . . . . . . . . . . 3第
15.3节 放大电路的动态分析 . . . . . . . . . . . . . . . . . . . . . . 3
第15.3.2题 . . . . . . . . . . . . . . . . . . . . . . . . . . . . . . 3第
15.3.3题 . . . . . . . . . . . . . . . . . . . . . . . . . . . . . . 4第
15.3.4题 . . . . . . . . . . . . . . . . . . . . . . . . . . . . . . 5第
15.3.5题 . . . . . . . . . . . . . . . . . . . . . . . . . . . . . . 6第
15.4节 静态工作点的稳定 . . . . . . . . . . . . . . . . . . . . . . . . 6
第15.4.2题 . . . . . . . . . . . . . . . . . . . . . . . . . . . . . . 6第
15.4.5题 . . . . . . . . . . . . . . . . . . . . . . . . . . . . . . 7第
15.6节 射极输出器 . . . . . . . . . . . . . . . . . . . . . . . . . . . . 9
第15.6.1题 . . . . . . . . . . . . . . . . . . . . . . . . . . . . . . 9第
15.6.2题 . . . . . . . . . . . . . . . . . . . . . . . . . . . . . . 9第
15.7节 差分放大电路 . . . . . . . . . . . . . . . . . . . . . . . . . . 11
第15.7.3题 . . . . . . . . . . . . . . . . . . . . . . . . . . . . . . 11 第
15.9节 场效晶体管及其放大电路 . . . . . . . . . . . . . . . . . . . . 13
第15.9.2题 . . . . . . . . . . . . . . . . . . . . . . . . . . . . . . 13
第15.9.3题 . . . . . . . . . . . . . . . . . . . . . . . . . . . . . . 14

1
List of Figures
1 习题15.2.3图 . . . . . . . . . . . . . . . . . . . . . . . . . . . . . . . . 3
2 习题15.3.3图 . . . . . . . . . . . . . . . . . . . . . . . . . . . . . . . . 4
3 习题15.3.5图 . . . . . . . . . . . . . . . . . . . . . . . . . . . . . . . . 6
4 习题15.4.5图 . . . . . . . . . . . . . . . . . . . . . . . . . . . . . . . . 8
5 习题15.6.1图 . . . . . . . . . . . . . . . . . . . . . . . . . . . . . . . . 9
6 习题15.6.2图 . . . . . . . . . . . . . . . . . . . . . . . . . . . . . . . . 10
7 习题15.7.3图 . . . . . . . . . . . . . . . . . . . . . . . . . . . . . . . . 11
8 习题15.9.2图 . . . . . . . . . . . . . . . . . . . . . . . . . . . . . . . . 13
9 习题15.9.2图 . . . . . . . . . . . . . . . . . . . . . . . . . . . . . . . . 14

2
15 基本放大电路

15.2 放大电路的静态分析

15.2.3

在图1中,若UC C = 10V ,今要求UC E = 5V ,IC = 2mA,试求RC 和RB


的阻值。设晶体管的β = 40。
[解]

图 1: 习题15.2.3图

由UC E = UC C − RC IC 可求

UC C − UC E 10 − 5
RC = = Ω = 2.5kΩ
IC 2 × 10−3
IC 2
IB ≈ = mA = 0.05mA
β 40
RB ≈ UC C 10
= kΩ = 200kΩ
IB 0.05

15.3 放大电路的动态分析

15.3.2

在习题1图所示的固定偏置放大电路中,UC C = 9V ,晶体管的β = 20,IC =


1mA。今要求|Au | ≤ 100,试计算RC ,RB 及UC E 。

3
[解]
IC 1
IB ≈ = mA = 0.05mA
β 20
≈ U CC 9
RB = kΩ = 180kΩ
IB 0.05
26
rbe = [200 + (20 + 1) × ]Ω = 720Ω = 0.72kΩ
1.05
|Au | = βRC (空载时 |Au | 最大)
rbe
|Au | rbe 100 × 0.72
RC = = kΩ = 3.6kΩ
β 20
UC E = UC C − RC IC = (9 − 3.6 × 1)V = 5.4V

15.3.3

有一放大电路如习题1图所示,其晶体管的输出特性以及放大电路的交、直
流负载线如图2所示。试问:(1)RB ,RC ,RL 各为多少?(2)不产生失真的最大输入
电压UiM 为多少?(3)若不断加大输入电压的幅值,该电路首先出现何种性质的
失真?调节电路中哪个电阻能消除失真?将阻值调大还是调小?(4)将电阻RL 调
大,对交、直流负载线会产生什么影响?(5)若电路中其他参数不变,只将晶体
管换一个β值小一半的管子,这时IB ,IC ,UC E 及|Au |将如何变化?
[解]

图 2: 习题15.3.3图

由图2可知,静态值为

IC = 2mA, IB = 40µA, UC E = 5V
电源电压为 电
UC C = 10V
流放大系数为
IC 2
β= = = 50
IB 0.04

4
(1)
RB ≈ UC C = 10 kΩ = 250kΩ
IB 0.04
UC C − UC E 10 − 5
RC = = kΩ = 2.5kΩ
IC 2
由交流负载线可得
1 2 1
tan α0 = , = , R0L = 1.5kΩ
RL0 8− 5 RL0
由此得
RC RL RC R 0 2.5 × 1.5
R0L = RL = RC − LR0 = 2.5 − 1.5kΩ = 3.75kΩ
RC + RL L

(2) 由图2可知

8 − UC EQ = (8 − 5)V = 3V UC EQ − UC ES = (5 − 0.3)V = 4.7V

不失真的最大输出电压约为UoM = 3V ,先求出|Au |后,再求不产生失真


的最大输入电压UiM
26(mV ) 26
rbe = 200(Ω) + (1 + β) = [200 + (1 + 50) × ]Ω = 0.86kΩ
IE (mA) 2
|Au | = βRL = 50 × 1.5 = 87
0

rbe 0.86
于是
UoM 3
UiM = = V = 34.5mV
|Au | 87
(3) 首先产生截止失真,这时可调节RB ,减小其阻值以增大IB ,将静态工作
点Q上移一点。
(4) 将RL 阻 值 增 大 , 不 影 响 直 流 负 载 线 , 通 过Q点 的 交 流 负 载 线 与 横 轴
的α0 角将有所减小。
(5) IB 不变,IC 约减小一半,UC E 增大,|Au |将减小一半。

15.3.4

已知某放大电路的输出电阻为3.3kΩ,输出端开路电压的有效值Uo0 = 2V ,
试问该放大电路接有负载电阻RL = 5.1kΩ时,输出电压将下降到多少?
[解]
RL RL
UoL = Eo = Uo0
ro + RL ro + RL

RL 5.1
UoL = Uo0 = × 2V = 1.2V
ro + RL 3.3 + 5.1

5
15.3.5

在图3中,UC C = 12V ,RC = 2kΩ,RE = 2kΩ,RB = 300kΩ,晶体管的β =


50。电路有两个输出端。试求:(1)电压放大倍数Au1 = U̇o1 和Au2 = U̇o2 ;(2)输
U̇i U̇i
出电阻ro1 和ro2 。
[解]

图 3: 习题15.3.5图

UC C − UBE 12 − 0.6
IB = = mA = 0.028mA
RB + (1 + β)RE 300 + (1 + 50) × 2
IE = (1 + β)IB = (1 + 50) × 0.028mA = 1.43mA
26
rbe = [200 + (1 + 50) × ]Ω = 1127Ω ≈ 1.13kΩ
1.43
从集电极输出:
U̇o1 = − βRC 50 × 2
Au1 = =− ≈ −1
U̇i rbe + (1 + β)RE 1.13 + (1 + 50) × 2
ro1 ≈ RC = 2kΩ
从发射极输出:
U̇o2 (1 + β)RE
Au2 = = ≈1
U̇i rbe + (1 + β)RE
ro2 ≈ rbe + R0S r 1130
≈ be = Ω = 22.6Ω
β β 50
式中,R0S = RS //RB ,设信号源内阻RS ≈ 0,则RS0 ≈ 0。

15.4 静态工作点的稳定
15.4.2

在 教 材 图15.4.1所 示 的 分 压 式 偏 置 放 大 电 路 中 , 已 知UC C = 24V ,RC =


3.3kΩ,RE = 1.5kΩ,RB1 = 33kΩ,RB2 = 10kΩ,RL = 5.1kΩ,β = 66, 并

6
设RS ≈ 0。(1)试求静态值IB ,IC 和UC E ;(2)画出微变等效电路;(3)计算晶体管
的输入电阻rbe ;(4)计算电压放大倍数Au ;(5)计算放大电路输出端开路时的电
压放大倍数,并说明负载电阻RL 对电压放大倍数的影响;(6)估算放大电路的输
入电阻和输出电阻。
[解]
(1)

UC C 24
VB = RB2 = × 10V = 5.58V
RB1 + RB2 33 + 10
IC ≈ IE = VB − UBE = 5.58 − 0.6 mA = 3.32mA
RE 1.5
IB ≈ IC = 3.32 mA = 0.05mA
β 60
UC E = UC C − (RC + RE )IC = [24 − (3.3 + 1.5) × 3.32]V = 8.06V

(2)

26(mV )
rbe = 200(Ω) + (1 + β)
IE (mA)
26
= [200 + (1 + 66) × ]Ω = 0.72kΩ
3.32
(3)
? ?
Au = − β RL = − 66 × 3.3 × 5.1 × 1 = − 183.7
0

rbe 3.3 + 5.1 0.72

(4)

RC 3.3
A u= −β = − 66 × = − 302.5
r be 0.72

(5)

ri = rbe //RB1 //RB2 ≈ rbe = 0.72kΩ


ro ≈ RC = 3.3kΩ

15.4.5

设 计 一 单 管 晶 体 管 放 大 电 路 , 已 知RL = 3kΩ。 要 求|Au | ≥ 60,ri ≥


1kΩ,ro < 3kΩ, 工 作 点 稳 定 。 建 议 选 用 高 频 小 功 率 管3GD100, 其 技 术 数
据见教材附录C ,β值可选在50 ∼ 100之间。最后核查静态工作点是否合适。求
得的各电阻值均采用标称值(查教材附录H )。
[解]

7
图 4: 习题15.4.5图

(1) 选择放大电路和晶体管 要求工作点稳定,可选用分压偏置放大电路(教


材图15.4.1),选UC C =
12V ; 按 建 议 选 用 晶 体 管3GD100, 设β = 50; 并 设|Au | = 60,ri =
1kΩ。
(2) 参数计算
26
由式r be ≈ [200 + (1 + β) ]Ω ≈ ri 可求
IE
26(1 + β) 26 × 51
IC ≈ IE = mA = 1.66mA
≈ ri − 200 1000 − 200

β RL0
由式|A u | = 可求
r be
60 × 1
R0L = 50
kΩ = 1.2kΩ
RC RL
RL0 =
RC + RL

R0L RL 1.2 × 3
RC = = kΩ = 2kΩ
RL − RL 0
3 − 1.2
设VB = 4V
VB − UBE 4 − 0.6
RE = IE = kΩ ≈ 2kΩ
1.66

基极电流IB ≈ IC = 1.66 mA = 0.033mA


β 50
设I2 = 10IB ,即
I2 = 10 × 0.033mA = 0.33mA ≈ I1


VB 4
RB2 = = kΩ = 12.12kΩ(取12kΩ)
I2 0.33
UC C − VB 12 − 4
RB1 = = kΩ = 24.24kΩ(取24kΩ)
I1 0.33

8
(3) 核查静态工作点
由UC E = UC C − (RC + RE )IC 做直流负载线(图4)

IC = 0 UC E = UC C = 12V
UC C 12
UC E = 0 IC = = mA = 3mA
RC + RE 2+2
UC E = [12 − (2 + 2) × 1.66]V = 5.4V

静态工作点合适,在小信号情况下,不会产生失真。

15.6 射极输出器

15.6.1

在图5所示的射极输出器中,已知RS = 50Ω,RB1 = 100kΩ,RB2 = 30kΩ,RE =


1kΩ,晶体管的β = 50,rbe = 1kΩ,试求Au ,ri 和ro 。
[解]

图 5: 习题15.6.1图

(1 + β)RE (1 + 50) × 1
Au = = = 0.98
rbe + (1 + β)RE 1 + (1 + 50) × 1
ri = RB1 //RB2 //[rbe + (1 + β)RE ] = 16kΩ

ro ≈ rbe + R0S 1000 + 50


= Ω = 21Ω
β 50
式中

R0S = RS //RB1 //RB2 ≈ 50Ω

15.6.2

两级放大电路如图6所示,晶体管的β1 = β2 = 40,rbe1 = 1.37kΩ,rbe2


= 0.89kΩ。(1)画出直流通路,并估算各级电路的静态值(计算UC E1 时忽略IB2 );

9
(2)画出微变等效电路,并计算Au1 ,Au2 和Au ;(3)计算ri 和ro 。
[解]

图 6: 习题15.6.2图

(1) 前极静态值
20
VB1 = × 8.2V = 4V
33 + 8.2
4 − 0.6
IC 1 ≈ IE1 = mA = 1mA
3 + 0.39
1
IB1 ≈ mA = 25µA
40
UC E1 ≈ 20 − (10 + 3 + 0.39) × 1 = 6.6V

后极静态值
IC 2 ≈ IE2 = UC 1 − UBE2 = (20 − 10 × 1) − 0.6 mA = 1.8mA
RE2 5.1
1.8
IB2 = mA = 45µA
40
UC E2 = (20 − 5.1 × 1.8)V = 10.8V

(2) 前级电压放大倍数
R0L1 9.1 = − 21
A u1 = −β 1 = −40 ×
rbe + (1 + β1 )R E1
00 1.37 + (1 + 40) × 0.39
式中
? ?
RE2 RL
R0L1 = RC 1 // rbe2 + (1 + β2 ) ·
RE2 + RL
后级电压放大倍数
(1 + β2 )R0L (1 + 40) × 2.5
Au2 = = = 0.99
rbe2 + (1 + β2 )RL
0
0.89 + (1 + 40) × 2.5
两级电压放大倍数
Au = Au1 · Au2 = − 21 × 0.99 = − 20.8

10
(3)

ri = ri1 = RB1 //RB2 //[rbe1 + (1 + β1 )R00E1 ] = 4.77kΩ


ro = ro2 ≈ rbe2 + RC 1 = 0.89 + 10kΩ = 272Ω
β2 40

前级的集电极电阻RC 1 即为后级的基极电阻。
从本例的两级放大电路看,提高了输入电阻,降低了输出电阻。

15.7 差分放大电路

15.7.3

在 图7所 示 的 差 分 放 大 电 路 中 ,β = 50,UBE = 0.7V , 输 入 电 压ui1 =


7mV ,ui2 = 3mV 。
(1)计算放大电路的静态值IB ,IC 及各电极的电位VE ,VC 和VB ; (2)把输
入电压Ui1 ,ui2 分解为共模分量uic1 ,uic2 和差模分量uid1 ,uid2 ; (3)求单
端共模输出uoc1 和uoc2 ;
(4)求单端差模输出uod1 和uod2 ; (5)求单端总输出uo1 和uo2 ;
(6)求双端共模输出uoc ,双端差模输出uod 和双端总输出uo 。
[解]

图 7: 习题15.7.3图

(1) 静态时,ui1 = ui2 = 0,由教材图15.7.5的单管直流通路可得

RB IB + UBE + 2RE IE = UEE


UEE − UBE
IB =
RB + 2(1 + β)RE

11
于是
6 − 0.7
IB = A
10 × + 2 × (1 + 50) × 5.1 × 103
103
= 0.01 × 10−3 A = 0.01mA
IC = βIB = 50 × 0.01mA = 0.5mA
IE = (1 + β)IB = 51 × 0.01mA = 0.51mA
VC = UC C − RC IC = [6 − 5.1 × 103 × 0.5 × 10−3 ]V = 3.45V
VE = − 6 + 2RE IE = [− 6 + 2 × 5.1 × 103 × 0.51 × 10−3 ]V = −
0.798V VB = − RB IB = − 10 × 103 × 0.01 × 10−3 V = − 0.1V

(2)
ui1 + ui2 7+3
uic1 = uic2 = = mV = 5mV
2 2
ui1 − ui2 7 − 3
uid1 = − uid2 = = mV = 2mV
2 2
(3) 由习题15.7.2所证明的公式得出
RC
uoc1 = uoc2 = − β u
RB + rbe + 2(1 + β)RE ic1
式中
26
rbe = [200 + (1 + 50) × ]Ω = 2.8kΩ
0.51
于是
5.1
uoc1 = uoc2 = − 50 × × 5mV
10 + 2.8 + 2(1 + 50) × 5.1
= − 2.39mV

(4)
uod1 = − βRC u = − 50 × 5.1 × 2mV = − 39.8mV
RB + rbe id1 10 + 2.8
50 × 5.1
uod2 = − βRC u = − × (− 2)mV = +39.8mV
R id2
10 + 2.8
B + rbe

(5)

uo1 = uoc1 + uod1 = [(− 2.39) + (− 39.8)]mV = − 42.2mV


uo2 = uoc2 + uod2 = [(− 2.39) + 39.8]mV = +37.4mV

(6)
uoc = uoc1 − uoc2 = 0
uod = uod1 − uod2 = (− 39.8 − 39.8)mV = − 79.6mV
uo = uo1 − uo2 = (− 42.2 − 37.4)mV = − 79.6mV = uod

12
15.9 场效晶体管及其放大电路
15.9.2

在图8所示的源极输出器中,已知UDD = 12V ,RS = 12kΩ,RG1 = 1M Ω,RG2 =


500kΩ,RG = 1M Ω。 试 求 静 态 值 、 电 压 放 大 倍 数 、 输 入 电 阻 和 输 出 电 阻 。
设VG ≈ VS ,gm = 0.9mA/V 。
[解]

图 8: 习题15.9.2图

RG2 UDD
VS ≈ VG =
RG1 + RG2
500 × 12
= V = 4V
1000 + 500
UDS = UDD − VS = (12 − 4)V = 8V
VS 4
ID = = A = 0.33 × 10−3 A = 0.33mA
RS 12 × 103
U̇o gm U˙ gs RS gm RS
Au = = = ≈1
U̇i U̇gs + gm U̇gs RS 1 + gm RS
? ?
1 × 0.5
ri = RG + (RG1 //RG2 ) = 1 + MΩ = 1.33MΩ
1 + 0.5
求输出电阻ro (图9):
将输入端短路(设信号源内阻很小,略去),输出端加一交流电压U̇o (如接有
负载电阻RL ,则除去),可求电流
U̇o
I˙o = − I˙d = U̇o − gm U̇gs
RS RS

U̇gs = − U̇o

U̇o 1
I˙o = + gm U̇o = (gm + )U̇o
RS RS

13
图 9: 习题15.9.2图

U̇o 1 1
ro = = = //RS
I˙o (gm +
1
)
gm
RS
1
当RS ? 时
gm
1
ro ≈
gm
在本题中
1 1
RS = 12kΩ = = 1.1kΩ
gm 0.9


ro ≈ 1.1kΩ

15.9.3

场效晶体管差分放大电路如教材图15.11所示,已知gm = 1.5mA/V ,求电压


uo
放大倍数Au = 。
ui
[解]
uo 与ui 反相,所以
uo 1 1
Au = = − gm RD = − × 1.5 × 15 = − 11.25
ui 2 2

14
15
目录
第16章 集成运算放大器 3
第16.2节 运算放大器在信号运算方面的应用 . . . . . . . . . . . . . . . 3
第16.2.2题 . . . . . . . . . . . . . . . . . . . . . . . . . . . . . . 3第
16.2.3题 . . . . . . . . . . . . . . . . . . . . . . . . . . . . . . 3第
16.2.5题 . . . . . . . . . . . . . . . . . . . . . . . . . . . . . . 4第
16.2.6题 . . . . . . . . . . . . . . . . . . . . . . . . . . . . . . 5第
16.2.10题 . . . . . . . . . . . . . . . . . . . . . . . . . . . . . 5第
16.2.12题 . . . . . . . . . . . . . . . . . . . . . . . . . . . . . 6第
16.2.13题 . . . . . . . . . . . . . . . . . . . . . . . . . . . . . 7第
16.2.15题 . . . . . . . . . . . . . . . . . . . . . . . . . . . . . 8第
16.2.17题 . . . . . . . . . . . . . . . . . . . . . . . . . . . . . 9第
16.2.18题 . . . . . . . . . . . . . . . . . . . . . . . . . . . . . 9第
16.2.20题 . . . . . . . . . . . . . . . . . . . . . . . . . . . . . 10 第
16.2.22题 . . . . . . . . . . . . . . . . . . . . . . . . . . . . . 11 第
16.2.25题 . . . . . . . . . . . . . . . . . . . . . . . . . . . . . 12 第
16.3节 运算放大器在信号处理方面的应用 . . . . . . . . . . . . . . . 13
第16.3.3题 . . . . . . . . . . . . . . . . . . . . . . . . . . . . . . 13

1
List of Figures
1 习题16.2.2图 . . . . . . . . . . . . . . . . . . . . . . . . . . . . . . . . 3
2 习题16.2.3图 . . . . . . . . . . . . . . . . . . . . . . . . . . . . . . . . 4
3 习题16.2.5图 . . . . . . . . . . . . . . . . . . . . . . . . . . . . . . . . 5
4 习题16.2.6图 . . . . . . . . . . . . . . . . . . . . . . . . . . . . . . . . 5
5 习题16.2.10图 . . . . . . . . . . . . . . . . . . . . . . . . . . . . . . . 6
6 习题16.2.12图 . . . . . . . . . . . . . . . . . . . . . . . . . . . . . . . 6
7 习题16.2.13图 . . . . . . . . . . . . . . . . . . . . . . . . . . . . . . . 7
8 习题16.2.15图 . . . . . . . . . . . . . . . . . . . . . . . . . . . . . . . 8
9 习题16.2.17图 . . . . . . . . . . . . . . . . . . . . . . . . . . . . . . . 9
10 习题16.2.18图 . . . . . . . . . . . . . . . . . . . . . . . . . . . . . . 10
11 习题16.2.18图 . . . . . . . . . . . . . . . . . . . . . . . . . . . . . . 10
12 习题16.2.20图 . . . . . . . . . . . . . . . . . . . . . . . . . . . . . . 11
13 习题16.2.22图 . . . . . . . . . . . . . . . . . . . . . . . . . . . . . . 11
14 习题16.2.25图 . . . . . . . . . . . . . . . . . . . . . . . . . . . . . . 12
15 习题16.3.3图 . . . . . . . . . . . . . . . . . . . . . . . . . . . . . . . 13
16 习题16.3.3图 . . . . . . . . . . . . . . . . . . . . . . . . . . . . . . . 14

2
16 集成运算放大器
16.2 运算放大器在信号运算方面的应用

16.2.2

在 图1所 示 的 同 相 比 例 运 算 电 路 中 , 已 知R1 = 2kΩ,RF = 10kΩ,R2 =


2kΩ,R3 = 18kΩ,ui = 1V ,求uo 。
[解]

图 1: 习题16.2.2图

ui 1
u+ = R3 = 18 × V = 0.9V
R2 + R3 2 + 18

于是得
? ? ? ?
RF 10 × 0.9V = 5.4V
uo = 1 + u+ = 1 +
R1 2

16.2.3

为了获得较高的电压放大倍数,而又可避免采用高值电阻RF ,将反相比例
运算电路改为图2所示的电路,并设RF ? R4 ,试证:
? ?
uo RF R3
Auf = =− 1+
ui R1 R4

[证]
因i1 ≈ if ,u− ≈ u+ =
0
故得 ui uA RF
=− uA = − ui
R1 RF R1

3
图 2: 习题16.2.3图

if + i3 = i4 或 i1 + i3 = i4
ui uo − uA uA
+ =
R1 R3 R4
将uA 代入,整理后得

uo = − RF R4 + R3 (RF + R4 ) ui
R 1R 4
因RF ? R4 ,故

uo = − RF R4 + RF R3 ui
R 1R 4

于是得出
? ?
uo RF R3
Auf = =− 1+
ui R1 R4

16.2.5

电路如图3所示,已知ui1 = 1V ,ui2 = 2V ,ui3 = 3V ,ui4 = 4V ,R1 = R2 =


2kΩ,R3 = R4 = RF = 1kΩ,求uo 。
[解]
应用结点电压法求u+ :
ui3 ui4
+
R3 R4 3+4
u+ = 1 = V = 3.5V
1 2
+
R3 R4

4
图 3: 习题16.2.5图

应用叠加原理求uo :
? ?
RF u+ − RF u − RF u
uo = 1+ i1 i2
R1 //R2 R1 R2
?? ? ?
1 1 1
= 1+ × 3.5 − × 1 − × 2 V
1 2 2
= 5.5V

16.2.6

求图4所示电路的uo 与ui 的运算关系式。


[解]

图 4: 习题16.2.6图

RF
u o1 = − ui
R1
R 2RF
uo = − uo1 − uo1 = − uo1 − uo1 = − 2uo1 = ui
R R1

16.2.10

图5是利用两个运算放大器组成的具有较高输入电阻的差分放大电路。试求
出uo 与ui1 、ui2 的运算关系式。

5
[解]

图 5: 习题16.2.10图

? ?
K R2 ui2 − K R2
uo = 1+ uo1
R2 R2
? ?
R1 /K
= (1 + K )ui2 − K 1 + ui1
R1
= (1 + K )ui2 − (1 + K )ui1
= (1 + K )(ui2 − ui1 )

16.2.12

为了用低值电阻实现高放大倍数的比例运算,常用一T 形网络代替RF ,电路


如图6所示。试证明
uo R2 + R3 + R2 R3 /R4
=−
ui R1
[证]
图中,i1 = i2

图 6: 习题16.2.12图

6

ui uD
= −
R1 R2
R2
uD = − ui
R1

i3 = i2 + i4 = i1 + i4
uD − uo ui uD
= −
R3 R1 R4
uo uD ui u
− =− + − D
R3 R3 R1 R4

R2 u 代入,则
将u D = − i
R1

uo R2 1 R2
− = ui + ui + ui
R3 R1 R3 R1 R1 R4
R R + R3 R4 + R2 R3
uo = − 2 4 ui
R 1R 4
R + R3 + R2 R3 /R4
= − 2 ui
R1

16.2.13

电路如图7所示,试证明uo = 2ui 。
[证]

图 7: 习题16.2.13图

7
? ?
R
uo2 = 1 + ui2 = 2ui2
R
uo1 − ui1 u − uo2
= i1
R R

于是得出

uo = uo1 = 2ui1 − 2ui2


= 2(ui1 − ui2 ) = 2ui

16.2.15

电路如图8所示,试证明iL = ui 。
RL
[证]
A1 接成同相比例运算电路;A2 为电压跟随器,uo2 = uo 。

图 8: 习题16.2.15图

ui − uo2 ui − uo2
u1+ = R3 + uo2 = × 10 + uo2
R2 + R3 10 + 10
ui + uo
=
? 2 ? ? ?
RF 10 ui + uo
uo1 = 1+ u1+ = 1 +
R1 10 2
= ui + uo
uo1 − uo ui + uo − uo ui
iL = = =
RL RL RL

8
16.2.17

在图9所示的电路中,电源电压为+15V ,ui1 = 1.1V ,ui2 = 1V 。试问接入


输入电压后,输出电压uo 由0上升到10V 所需的时间。
[解]
运算放大器A1 接成差分运算电路,A2 为一积分运算电路。

图 9: 习题16.2.17图

由A1 可得
? ?
RF R3 RF
uo1 = 1+ ui2 − u
R1 R2 + R3 R1 i1
R 20
= F (ui2 − ui1 ) = (1 − 1.1)V = − 0.2V
R1 10
由A2 可得
Z
1 uo1
uo = − u o1 dt = − t
R 4 CF R 4C F
0.2
= t V = 10t V
20 × 103 × 1 × 10−6
uo 由0上升到10V 所需时间为

uo 10
t= = s = 1s
10 10

16.2.18

按下列运算关系式画出运算电路,并计算各电阻值:
(4)uo = 0.5ui ;(5)uo = 2ui2 − ui1 。已知RF = 10kΩ。 [解]
(4) 电路如图10所示

R1 R1 RF RF
u o= − u o1 = − ( − ui ) = ui
R1 R1 R1 R1

9
图 10: 习题16.2.18图

图 11: 习题16.2.18图


RF R 10
= 0.5 R1 = F = kΩ = 20kΩ
R1 0.5 0.5

(5) 电路如图11所示
? ?
RF ui2 − RF
uo = 1 + ui1 = 2ui2 − ui1
R1 R1

RF RF
1+ =2 =1
R1 R1
R1 = RF = 10kΩ

16.2.20

在教材图16.2.9所示的积分运算电路中,如果R1 = 50kΩ,CF = 1µF ,ui 如


图12(a)所示,试画出输出电压uo 的波形。
[解]
(1) t = 0 ∼ 10ms时:
Z Z
1 1
uo = − ui dt = − 5dtV
R 1 CF 50 × 103 × 1 × 10−6
5
= − tV
50 × 103 × 1 × 10−6
= − 100t V

10
图 12: 习题16.2.20图

(2) t = 10 ∼ 20ms时
Z
1 (− 5)dt = 100t + K
uo = −
R1 CF

1 s时,u = − 1V ,故K = − 2。于是得


当t = 10ms = o
100

uo = (100t − 2)V

uo 的波形如图12(b)所示。

16.2.22

在图13中,求uo 。
[解]

图 13 : 习题16.2.22

11
i = ic + if
ui − uc duc uc
=C +
R dt R
duc
ui = RC + 2uc
dt

解之,得

3
(1 − e− RC t)mV
2
uc =
2
4R
uo = − uc = − 4uc
R
= 6(e− RC t − 1)mV
2

16.2.25

图14是应用运算放大器测量小电流的原理电路,试计算电阻RF 1 ∼ RF 5 的阻
值。输出端接有满量程5V ,500µA的电压表。
[解]

图 14 : 习题16.2.25

12
Uo 5
RF 1 = = Ω = 1 × 103 Ω = 1kΩ
I1 5 × 10−3
? ?
Uo 5
RF 2 = − RF 1 = − 1 × 10 Ω3
I2 0.5 × 10−3
= 9 × 103 Ω = 9kΩ ? ?
Uo 5
RF 3 = − (RF 1 + RF 2 ) = − (1 + 9) × 10 Ω
3
I3 0.1 × 10−3
= 40 × 103Ω = 40kΩ ? ?
Uo 5
RF 4 = − (RF 1 + RF 2 + RF 3 ) = − (1 + 9 + 40) × 10 Ω
3
I4 0.05 × 10−3
= 50 × 103 Ω = 50kΩ
Uo
RF 5 = − (RF 1 + RF 2 + RF 3 + RF 4 )
I5
? ?
5
= − (1 + 9 + 40 + 50) × 10 Ω 3
0.01 × 10−3
= 400 × 103 Ω = 400kΩ

16.3 运算放大器在信号处理方面的应用

16.3.3

图15是火灾报警电路的方框图。ui1 和ui2 分别来自两个温度传感器,它们安


装在室内同一处:一个安装在塑料壳内,产生ui1 ;另一个安装在金属板上,产
生ui2 。无火情时,ui1 = ui2 ,声光报警电路不响不亮。一旦发生火情,安装在
金属板上的温度传感器因金属板导热快而温度升高较快,而另一个温度上升较
慢,于是产生差值电压(ui2 − ui1 ),当这差值电压增高到一定数值时,发光二极
管发亮,蜂鸣器发响,同时报警。请按图示方框图设计电路。
[解]
所设计火灾报警电路如图16所示,请自行读图分析。

图 15: 习题16.3.3图

13
图 16: 习题16.3.3图

14
15
目录
第17章 电子电路中的反馈 3
第17.2节 放大电路中的负反馈 . . . . . . . . . . . . . . . . . . . . . . 3
第17.2.1题 . . . . . . . . . . . . . . . . . . . . . . . . . . . . . . 3第
17.2.3题 . . . . . . . . . . . . . . . . . . . . . . . . . . . . . . 3第
17.2.5题 . . . . . . . . . . . . . . . . . . . . . . . . . . . . . . 3第
17.2.6题 . . . . . . . . . . . . . . . . . . . . . . . . . . . . . . 4第
17.2.7题 . . . . . . . . . . . . . . . . . . . . . . . . . . . . . . 4第
17.3节 振荡电路中的正反馈 . . . . . . . . . . . . . . . . . . . . . . 5
第17.3.1题 . . . . . . . . . . . . . . . . . . . . . . . . . . . . . . 5
第17.3.4题 . . . . . . . . . . . . . . . . . . . . . . . . . . . . . . 6

1
List of Figures
1 习题17.2.1图 . . . . . . . . . . . . . . . . . . . . . . . . . . . . . . . . 3
2 习题17.2.3图 . . . . . . . . . . . . . . . . . . . . . . . . . . . . . . . . 4
3 习题17.3.1图 . . . . . . . . . . . . . . . . . . . . . . . . . . . . . . . . 5

2
17 电子电路中的反馈
17.2 放大电路中的负反馈

17.2.1

试判别图1所示的放大电路中引入了何种类型的交流反馈。
[解] 图中RF 为反馈电阻。设在ui 的正半周,晶体管各级交流电位的瞬时极性为

图 1: 习题17.2.1图

B1 (⊕ ) −→ C1 (?) −→ B2 (?) −→ E2 (?) −→ B1


(⊕ )

即可看出,发射极E2 交流电位的负极性反馈到基极B1 ,降低了B1 的交流电位,


使Ube1 减小,故为负反馈。 另外,反馈电路从发射极引出,引入到基极,故为并
联电流反馈。

17.2.3

为了实现下述要求,在图2中应引入何种类型的负反馈?反馈电阻RF 应从何
处引至何处?(1)减小输入电阻,增大输出电阻;(2)稳定输出电阻,此时输入电
阻增大否?
[解]
(1) RF 应从E3 引至B1 ,并联电流负反馈;
(2) RF 应从C3 引至E1 ,串联电压负反馈,此时输入电阻增大了。

17.2.5

当保持收音机收听的音量不变时,能否在收音机的放大电路中引入负反馈来

3
减小外部干扰信号的影响?负反馈能不能抑制放大电路内部出现的干扰信号?

4
图 2: 习题17.2.3图

[解] 收音机的放大电路中引入负反馈后,对外来正常信号和干扰信号的放大
倍数同时降低,不能保持音量不变。但引入适当的负反馈能抑制放大电路内部
出现的干扰信号。

17.2.6

有一负反馈放大电路,已知A = 300,F = 0.01。试问:(1)闭环电压放大倍


数Af 为多少?(2)如果A发生±20% 的变化,则Af 的相对变化为多少?
[解]
(1)

A 300
Af = = = 75
1 + AF 1 + 300 × 0.01

(2)

∆Af ∆A 1
= ·
Af A 1 + (A + ∆A)F
1
= ±20% ×
1 + (300 ± 300 × 20%) × 0.01
= +4.34%和 − 5.88%

17.2.7

有 一 同 相 比 例 运 算 电 路 , 如 教 材 图17.2.1所 示 。 已 知Auo = 1000,F =


+0.049。 如 果 输 出 电 压uo = 2V , 试 计 算 输 入 电 压ui , 反 馈 电 压uf 及 净 输 入
电压ud 。

5
[解]

Auo 1000
Auf = = = 20
1 + Auo F 1 + 1000 × 0.049
uo 2
ui = = V = 0.1V
Auf 20
uf = uo F = 2 × 0.049V = 0.098V
ud = ui − uf = (0.1 − 0.098)V = 0.002V

17.3 振荡电路中的正反馈

17.3.1

图3是用运算放大器构成的音频信号发生器的简化电路。(1)R1 大致调到多
大才能起振?(2)RP 为双联电位器,可从0调到14.4kΩ,试求振荡频率的调节范
围。
[解]

图 3: 习题17.3.1图

(1) 电压放大倍数按同相输入计算,即

RF
Auf = 1 +
R1

因 为 产 生 振 荡 的 最 小 电 压 放 大 倍 数 为3, 所 以RF ≥ 2R1。 刚 起 振 时 ,


振荡幅度小,不足以使二极管导通,这时RF = RF 1 + RF 2 = 3kΩ,所
以R1 ≤ 1.5kΩ时才能起振。

6
(2) 振荡频率为
1
f0 =
2πRC
当将RP 调到最小时,f0 最大,即
1
f0max = H z = 995.2H z ≈ 1000H z
2π × 1.6 × 103 × 0.1 × 10−6
当将RP 调到最大时,f0 最小,即

1
f0min = H z = 99.5H z ≈ 100H z
2π × 16 × 103 × 0.1 × 10−6

17.3.4

在调试教材图17.3.3所示电路时,试解释下列现象:
(1) 对调反馈线圈的两个接头后就能起振;
(2) 调RB1 ,RB2 或RE 的阻值后就能起振;
(3) 改用β较大的晶体管后就能起振;
(4) 适当增加反馈线圈的圈数后就能起振;
(5) 适当增大L值或减小C 值后就能起振;
(6) 反馈太强,波形变坏;
(7) 调整RB1 ,RB2 或RE 的阻值后可使波形变好;
(8) 负载太大不仅影响输出波形,有时甚至不能起振。
[解释]
(1) 原反馈线圈接反,对调两个接头后,满足相位条件;
(2) 调阻值后,使静态工作点合适,以满足起振条件;
(3) 改用β较大的晶体管,以满足幅度条件;
(4) 增加反馈线圈的圈数,即增大反馈量,以满足幅度条件;
(5) 因为LC 并联电路在谐振时的等效阻抗模为

L
|Z0 | =
RC
当适当增大L值或减小C 值后,可使|Z0 |增大,因而就增大了反馈幅度,
容易起振;
(6) 反馈线圈Lf 的圈数过多或管子的β太大,使反馈太强而进入非线性区,使
波形变坏;
(7) 调阻值,使静态工作点在线性区,使波形变好;
(8) 负 载 大 , 就 是 增 大 了LC 并 联 电 路 的 等 效 电 阻R。R的 增 大 , 一 方 面
使|Z0 |减小,因而使反馈幅度减小,不易起振;也使品质因数Q减小,因
而降低了选频性,使波形变坏。

7
8
目录
第18章 直流稳压电源 3
第18.1节 整流电路 . . . . . . . . . . . . . . . . . . . . . . . . . . . . . 3
第18.1.1题 . . . . . . . . . . . . . . . . . . . . . . . . . . . . . . 3
第18.1.2题 . . . . . . . . . . . . . . . . . . . . . . . . . . . . . . 3
第18.1.3题 . . . . . . . . . . . . . . . . . . . . . . . . . . . . . . 4
第18.1.7题 . . . . . . . . . . . . . . . . . . . . . . . . . . . . . . 4
第18.1.8题 . . . . . . . . . . . . . . . . . . . . . . . . . . . . . . 5
第18.2节 滤波器 . . . . . . . . . . . . . . . . . . . . . . . . . . . . . . 5
第18.2.4题 . . . . . . . . . . . . . . . . . . . . . . . . . . . . . . 5
第18.3节 直流稳压电源 . . . . . . . . . . . . . . . . . . . . . . . . . . 6
第18.3.1题 . . . . . . . . . . . . . . . . . . . . . . . . . . . . . . 6
第18.3.4题 . . . . . . . . . . . . . . . . . . . . . . . . . . . . . . 7
第18.3.6题 . . . . . . . . . . . . . . . . . . . . . . . . . . . . . . 7

1
List of Figures
1 习题18.1.1图 . . . . . . . . . . . . . . . . . . . . . . . . . . . . . . . . 3
2 习题18.1.7图 . . . . . . . . . . . . . . . . . . . . . . . . . . . . . . . . 4
3 习题18.2.4图 . . . . . . . . . . . . . . . . . . . . . . . . . . . . . . . . 6
4 习题18.3.1图 . . . . . . . . . . . . . . . . . . . . . . . . . . . . . . . . 6
5 习题18.3.6图 . . . . . . . . . . . . . . . . . . . . . . . . . . . . . . . . 8

2
18 直流稳压电源
18.1 整流电路

18.1.1

在图1中,已知RL = 80Ω,直流电压表V 的读数为110V ,试求:(1)直流电流


表A的读数;(2)整流电流的最大值;(3)交流电压表V1 的读数;(4)变压器二次侧
电流的有效值。二极管的正向压降忽略不计。
[解]

图 1: 习题18.1.1图

(1)
Uo 110
Io = = A = 1.38A
RL 80
(2)

Iom = πIo = 3.14 × 1.38A = 4.33A

(3)
Uo 110
U1 = = V = 244.4V
0.45 0.45
(4)

I = 1.57Io = 1.57 × 1.38A = 2.16A

18.1.2

在 教 材 图18.1.1所 示 的 单 相 半 波 整 流 电 路 中 , 已 知 变 压 器 二 次 侧 电 流 的 有
效 值U = 30V , 负 载 电 阻RL = 100Ω, 试 问 :(1)输 出 电 压 和 输 出 电 流 的 平 均
值Uo 和Io 各为多少?(2)若电源电压波动±10%,二极管承受的最高反向电压为多
少?
[解]

3
(1)

Uo = 0.45U = 0.45 × 30V = 13.5V


Uo 13.5
Io = = A = 0.135A
RL 100
(2)

UDRM = 1.1 2U = 1.1 × 1.414 × 30V = 46.7V

18.1.3

若采用教材图18.1.3所示的单相桥式整流电路,试计算上题。
[解]
(1)

Uo = 0.9U = 0.9 × 30V = 27V


Uo 27
Io = = A = 0.27A
RL 100
(2)

UDRM = 1.1 2U = 1.1 × 1.414 × 30V = 46.7V

18.1.7

图2是二倍压整流电路,Uo = 2 2U ,试分析之,并标出Uo 的极性。
[分析]

当u在正半周时,D1 导通,D2 截止,C1 被充电到u的最大值 2U (极性左正

图 2: 习题18.1.7图 右负)。到u负半周时,D1 截止,D2 导通,u和C1 两端电压

相加。这时,电源

一方面经D2 供给RL 电流,同时经D2 给C2 充电,充到等于u的最大值 2U 和C1 两

端电压之和。由于C1 放电很慢,其上电压近似为 2U 。因此,C2 两端电压接

近2 2U (极性下正上负),此即为RL 上的电压Uo 。当D2 截止时,C2 通过RL 放
电,但因一般RL 阻值较大,放电时间常数很大,所以C2 两端电压Uo 衰减很小,

4

基本维持在2 2U 值。由于Uo 基本上为变压器二次绕组电压最大值的两倍,故


为二倍压整流电路。 倍压整流电路中,每个二极管所承受的最大反向电压为2

2U ,电容C1 的耐
√ √
压应大于 2U ,C2 的耐压应大于2 2U 。

18.1.8

有一电解电源,采用三相桥式整流。如要求负载直流电压Uo = 20V ,负载电


流Io = 200A,(1)试求变压器容量为多少千伏安;(2)选用整流元件。考虑到变
压器二次绕组和管子上的压降,变压器的二次侧电流要加大10%。 [解]查教材表
18.1.1
(1) 变压器二次侧相电压的有效值为
Uo 20
= V = 8.55V
U=
2.34 2.34
考虑到变压器二次绕组和管子上的压降,变压器的二次侧电压要加
大10%,即

U = 1.1 × 8.55V = 9.4V


变压器二次侧相电流的有效值为
I = 0.82Io = 0.82 × 200A = 164A
三相变压器的容量为

S = 3U I = 3 × 9.4 × 164V · A = 4624.8V · A ≈ 4.6kV · A

取5kV · A容量的变压器。
(2) 通过每管的电流平均值为
1 1
Io = × 200A = 66.7A
ID =
3 3
管子截止时承受的最高反向电压为
UDRM = 2.45U = 2.45 × 9.4V = 23V
可选用100A、50V 的整流管。

18.2 滤波器

18.2.4

在图3具有π形RC 滤波器的整流电路中,已知交流电压U = 6V ,今要求负载


输出电压Uo = 6V ,负载输出电流Io = 100mA,试计算滤波电阻R。
[解]
5
图 3: 习题18.2.4图

取 Uc1 = 1.2U = 1.2 × 6V = 7.2V


Uc1 − Uo 7.2 − 6
故 R = = Ω = 12Ω
Io 100 × 10−3

18.3 直流稳压电源
18.3.1

稳 压 二 极 管 稳 压 电 路 如 图4所 示 , 已 知u = 28.2 sin ωt V ,Uo = 6V ,RL =


2kΩ,R = 1.2kΩ。试求:(1)S1 断开,S2 合上时的UI 和IZ ;(2)S1 和S2 均合上时
的UI 和IZ ,并说明R = 0和DZ 接反两种情况下电路能否起稳压作用。
[解]

图 4: 习题18.3.1图

(1) S1 断开,S2 合上,此时无电容滤波

UI = 0.9U = 0.9 × 28.2


√ V = 0.9 × 20V = 18V
2
UI − Uo 18 − 6
IR = = A = 10 × 10−3 A = 10mA
R 1.2 × 10 3

Uo 6
Io = = A = 3 × 10−3 A = 3mA
RL 2 × 103
IZ = IR − Io = (10 − 3)mA = 7mA

6
(2) S1 和S2 均合上,此时带电容滤波

UI = 1.2U = 1.2 × 20V = 24V


UI − Uo 24 − 6
IR = = A = 15 × 10−3A = 15mA
RL 1.2 × 103
Io = 3mA
IZ = IR − Io = (15 − 3)mA = 12mA

如R = 0,因UI = 24V ,它直接加在UZ 为6V 的稳压管上,引起很大的反


向击穿电流而使稳压管损坏。一般而言,R = 0,电路失去稳压作用。 如
将DZ 反接,Uo = 0.7V ,达不到输出6V 的要求。

18.3.4

电 路 如 教 材 图18.3.3所 示 。 已 知UZ = 6V ,R1 = 2kΩ,R2 = 1kΩ,R3 =


2kΩ,U1 = 30V ,T 的电流放大系数β = 50。试求:(1)电压输出范围;(2)当UO =
15V ,RL = 150Ω时,调整管T 的管耗和运算放大器的输出电流。
[解]
(1) 求电压输出范围 电位计R1 调到最上端
UOmin = UZ = 6V
R1 + R2 2 + 1 × 6V = 18V
电位计R1 调到最下端UOmax = UZ =
R2 1
故Uo 的输出范围为6 ∼ 18V 。
(2) 求T 的管耗和运算放大器的输出电流
由于RL 比R1 ,R2 ,R3 都小的多,故

I ≈ I ≈ 15
= A = 0.1A = 100mA
Uo 150
C o
RL
T 的管耗为

PC = UC E IC = (30 − 15) × 0.1W = 1.5W

运算放大器的输出电流为
IC 100
IB = = mA = 2mA
β 50

18.3.6

在图5中,试求输出电压Uo 的可调范围为多少?
[解] 运算放大器接成电压跟随器,输出端电位(即W 7805的3端电位)和同相
输入端电位相同,而u+ ≈ u− ,故图中UX X = 5V 。

7
图 5: 习题18.3.6图

R1 + Rp + R2 3.3 + 5.1 + 3.3


UOmin = UX X = × 5V = 6.96V
R1 + Rp 5.1 + 3.3
R1 + RP + R2 3.3 + 5.1 + 3.3
UOmax = UX X = × 5V = 17.73V
R1 3.3

8
9
目录
第20章 门电路和组合逻辑电路 4
第20.2节 基本门电路及其组合 . . . . . . . . . . . . . . . . . . . . . . 4
第20.2.3题 . . . . . . . . . . . . . . . . . . . . . . . . . . . . . . 4
第20.3节 TTL门电路 . . . . . . . . . . . . . . . . . . . . . . . . . . . . 4
第20.3.2题 . . . . . . . . . . . . . . . . . . . . . . . . . . . . . . 4
第20.5节 逻辑代数 . . . . . . . . . . . . . . . . . . . . . . . . . . . . . 5
第20.5.5题 . . . . . . . . . . . . . . . . . . . . . . . . . . . . . . 5第
20.5.6题 . . . . . . . . . . . . . . . . . . . . . . . . . . . . . . 5第
20.5.7题 . . . . . . . . . . . . . . . . . . . . . . . . . . . . . . 5第
20.6节 组合逻辑电路的分析和综合 . . . . . . . . . . . . . . . . . . . 7
第20.6.1题 . . . . . . . . . . . . . . . . . . . . . . . . . . . . . . 7
第20.6.3题 . . . . . . . . . . . . . . . . . . . . . . . . . . . . . . 8
第20.6.4题 . . . . . . . . . . . . . . . . . . . . . . . . . . . . . . 9
第20.6.5题 . . . . . . . . . . . . . . . . . . . . . . . . . . . . . . 10
第20.6.10题 . . . . . . . . . . . . . . . . . . . . . . . . . . . . . 11
第20.6.13题 . . . . . . . . . . . . . . . . . . . . . . . . . . . . . 12
第20.6.14题 . . . . . . . . . . . . . . . . . . . . . . . . . . . . . 13
第20.7节 加法器 . . . . . . . . . . . . . . . . . . . . . . . . . . . . . . 14
第20.7.1题 . . . . . . . . . . . . . . . . . . . . . . . . . . . . . . 14
第20.7.2题 . . . . . . . . . . . . . . . . . . . . . . . . . . . . . . 14
第20.8节 编码器 . . . . . . . . . . . . . . . . . . . . . . . . . . . . . . 16
第20.8.1题 . . . . . . . . . . . . . . . . . . . . . . . . . . . . . . 16
第20.9节 译码器和数字显示 . . . . . . . . . . . . . . . . . . . . . . . . 17
第20.9.2题 . . . . . . . . . . . . . . . . . . . . . . . . . . . . . . 17
第20.9.4题 . . . . . . . . . . . . . . . . . . . . . . . . . . . . . . 19
第20.9.5题 . . . . . . . . . . . . . . . . . . . . . . . . . . . . . . 20

1
List of Tables
1 逻辑状态表 . . . . . . . . . . . . . . . . . . . . . . . . . . . . . . . . 7
2 逻辑状态表 . . . . . . . . . . . . . . . . . . . . . . . . . . . . . . . . 8
3 逻辑状态表 . . . . . . . . . . . . . . . . . . . . . . . . . . . . . . . . 9
4 逻辑状态表 . . . . . . . . . . . . . . . . . . . . . . . . . . . . . . . . 11
5 逻辑状态表 . . . . . . . . . . . . . . . . . . . . . . . . . . . . . . . . 12
6 逻辑状态表 . . . . . . . . . . . . . . . . . . . . . . . . . . . . . . . . 13
7 逻辑状态表 . . . . . . . . . . . . . . . . . . . . . . . . . . . . . . . . 15
8 逻辑状态表 . . . . . . . . . . . . . . . . . . . . . . . . . . . . . . . . 16
9 编码表 . . . . . . . . . . . . . . . . . . . . . . . . . . . . . . . . . . . 17
10 状态表 . . . . . . . . . . . . . . . . . . . . . . . . . . . . . . . . . . 18
11 逻辑状态表 . . . . . . . . . . . . . . . . . . . . . . . . . . . . . . . . 20

2
List of Figures
1 习题20.2.3图 . . . . . . . . . . . . . . . . . . . . . . . . . . . . . . . . 4
2 习题20.3.2图 . . . . . . . . . . . . . . . . . . . . . . . . . . . . . . . . 4
3 习题20.5.7图 . . . . . . . . . . . . . . . . . . . . . . . . . . . . . . . . 6
4 习题20.5.7图 . . . . . . . . . . . . . . . . . . . . . . . . . . . . . . . . 6
5 习题20.6.1图 . . . . . . . . . . . . . . . . . . . . . . . . . . . . . . . . 8
6 习题20.6.3图 . . . . . . . . . . . . . . . . . . . . . . . . . . . . . . . . 8
7 习题20.6.4图 . . . . . . . . . . . . . . . . . . . . . . . . . . . . . . . . 9
8 习题20.6.5图 . . . . . . . . . . . . . . . . . . . . . . . . . . . . . . . . 10
9 习题20.6.10图 . . . . . . . . . . . . . . . . . . . . . . . . . . . . . . . 11
10 习题20.6.13图 . . . . . . . . . . . . . . . . . . . . . . . . . . . . . . 12
11 习题20.6.14图 . . . . . . . . . . . . . . . . . . . . . . . . . . . . . . 14
12 习题20.7.2图 . . . . . . . . . . . . . . . . . . . . . . . . . . . . . . . 16
13 习题20.8.1图 . . . . . . . . . . . . . . . . . . . . . . . . . . . . . . . 17
14 习题20.9.2图 . . . . . . . . . . . . . . . . . . . . . . . . . . . . . . . 18
15 习题20.9.4图 . . . . . . . . . . . . . . . . . . . . . . . . . . . . . . . 20

3
20 门电路和组合逻辑电路
20.2 基本门电路及其组合

20.2.3

在图1所示的门电路中,当控制端C = 1和C = 0两种情况时,试求输出Y 的


逻辑式和波形,并说明该电路的功能。输入A和B的波形如图中所示。
[解] 由图得出Y 的逻辑式

图 1: 习题20.2.3图

Y = AC · BC
C =1 Y = A ·1= A = A 传送信号A
C = 0 Y = 1·B = B = B 传送信号B

20.3 TTL门电路

20.3.2

用 内 阻 为50kΩ/V 的 万 用 表 的 直 流 电 压 挡(0 ∼ 10V )去 测 量T T L与 非 门


的 一 个 悬 空 输 入 端 与“地”之 间 的 电 压 值 , 在 下 列 情 况 下 , 估 计 该 表 的 读
数。(1)其余输入端全悬空时;(2)其余输入端全接电源(+5V )时;(3)其余输入端
全接“地”时;(4)其余输入端中有一个接“地”时;(5)其余输入端全接0.3V 时。 [解]
根据教材21.4节的分析,可画图如图2所示:

图 2: 习题20.3.2图

4
20.5 逻辑代数

20.5.5

应用逻辑代数运算法则化简下列各式:(1) Y = AB + A B + AB;
(3) Y = (A + B) + AB;(5) Y = ABC + A + B + C + D.
[解]

(1) Y = AB + A B + AB = AB + (A + A)B = AB + B = A + B
(3) Y = (A + B) + AB = A B + AB = A B · AB = (A + B)(A + B)
= AA + AB + AB + BB = AB + AB = A ⊕ B
(5) Y = ABC + A + B + C + D = ABC + ABC + D = 1 + D = 1

20.5.6

应用逻辑代数运算法则推证下列各式:(3) AB + A B = AB + AB;
(5) (A + B) + (A + B) + (AB) · (AB) = 1。
[解]

(3) AB + A B = AB + A B = AB · A B

= (A + B)(A + B) = AA + AB + AB + BB
= AB + AB
(5) (A + B) + (A + B) + (AB) · (AB)

= (AB) + (AB) + (AB) · (AB)


= (AB) · (AB) + (AB) · (AB)
= 1

20.5.7

应用卡诺图化简下列各式:(1) Y = AB + ABC + ABC ;(3) Y = AB +


BC D + ABD + ABC D。
[解]
(1) 将逻辑函数化为最小项表示式

Y = AB + ABC + ABC
= AB(C + C ) + ABC + ABC
= ABC + ABC + ABC + ABC

5
图 3: 习题20.5.7图

画出卡诺图,如图3所示。
应用卡诺图化简,得

Y =B

(3) 可用三种方法画卡诺图
a 用逻辑状态表
四 输 入 变 量 有16种 组 合 , 由 每 组 输 入 变 量 取 值 求 出 输 出 变
量Y 为1
或0,由逻辑式(3)得出的状态表如表1所示。
由逻辑状态表画出卡诺图,如图4所示。

图 4: 习题20.5.7图

b 用最小项表达式 将逻辑函数化为
最小项表达式:

Y = AB + BC D + ABD + ABC D
= ABC D + ABC D + AB C D + AB C D + ABC D
+ ABC D + ABC D + ABC D + ABC D

即可画出图4所示的卡诺图。

6
表 1: 逻辑状态表
A B C D Y
0 0 0 0 0
0 0 0 1 0
0 0 1 0 0
0 0 1 1 0
0 1 0 0 1
0 1 0 1 1
0 1 1 0 0
0 1 1 1 0
1 0 0 0 1
1 0 0 1 1
1 0 1 0 1
1 0 1 1 1
1 1 0 0 1
1 1 0 1 1
1 1 1 0 0
1 1 1 1 1

c 直接写入
逻 辑 式 第 一 项AB占 最 下 行 四 个 小 方 格 ; 第 二 项BC D占 最 左 列
中 间 两 个 小 方 格 ; 第 三 项ABD占 第 三 行 中 间 两 个 小 方 格 ; 第 四
项ABC D占一个小方格。 用卡诺图化简时,可将图中取值为1的小
方格圈成三个圈,如图4所 示,由此得出

Y = AB + BC + AD

20.6 组合逻辑电路的分析和综合

20.6.1

(1)根据逻辑式Y = AB + A B列出逻辑状态表,说明其逻辑功能,并画出
用与非门和非门组成的逻辑图;(2)将上式求反后得出的逻辑式具有何种逻辑功
能?
[解]
(1) 逻辑状态表和逻辑图分别如表2和图5所示。

7
图 5: 习题20.6.1图

表 2: 逻辑状态表
A B Y
0 0 1
0 1 0
1 0 0
1 1 1

(2)

Y = AB + A B = AB · A B
= (A + B) · (A + B) = AB + AB

(1)是同或门Y = A ? B,(2)是异或门Y = A ⊕ B。

20.6.3

列出逻辑状态表,分析图6所示电路的逻辑功能。
[解]

图 6: 习题20.6.3图

Y = A ⊕ (BC + BC )

= A(BC + BC ) + A(BC + BC )

8
表 3: 逻辑状态表
A B C Y
0 0 0 0
0 0 1 1
0 1 0 1
0 1 1 0
1 0 0 1
1 0 1 0
1 1 0 0
1 1 1 1

其逻辑状态如表3所示,它是一判奇电路。当输入有奇数个1时,输出为1,否则
为0。

20.6.4

化简Y = AD + C D + A C + B C + DC ,并用74LS20双4输入与非门组成
电路。
[解]

图 7: 习题20.6.4图

9
Y = AD + C D + A C + B C + DC
= AD + C (D + D) + A C + B C
= AD + C + A C + B C
= AD + C (1 + A) + B C
= AD + C + B C
= AD + C (1 + B)
= AD + C

要用74LS20与非门组成电路,须将上式变换为与非门逻辑式

Y = AD + C = AD + C = AD · C

用74LS20与非门的连线如图7所示。

20.6.5

某一组合逻辑电路如图8所示,试分析其逻辑功能。
[解]

图 8: 习题20.6.5图

(1) 由逻辑图列出逻辑状态表(如表4)所示
8421编码表见教材表20.8.2,当十进制数5接高电平时,DC BA = 0101,
由图20.3.10分析,可知输出Y = 1,发光二极管亮;当十进制数6接高电
平时,DC BA = 0110,Y = 0,发光二极管不亮。

10
表 4: 逻辑状态表
十进 D C B A Y
制数
0 0 0 0 0 0
1 0 0 0 1 1
2 0 0 1 0 0
3 0 0 1 1 1
4 0 1 0 0 0
5 0 1 0 1 1
6 0 1 1 0 0
7 0 1 1 1 1
8 1 0 0 0 0
9 1 0 0 1 1

(2) 分析逻辑功能
由表4可知,凡是十进制的奇数接高电平时,Y = 1,否则Y = 0。故该电
路为判奇电路。

20.6.10

旅客列车分特快、直快和普快,并依此为优先通行次序。某站在同一时
间只能有一趟列车从车站开出,即只能给出一个开车信号,试画出满足上
述要求的逻辑电路。设A,B,C 分别代表特快、直快、普快,开车信号分别
为YA ,YB ,YC 。
[解]

图 9: 习题20.6.10图

(1) 列逻辑状态表(如表5所示)

11
表 5: 逻辑状态表
A B C YA YB YC
0 0 0 0 0 0
0 0 1 0 0 1
0 1 0 0 1 0
0 1 1 0 1 0
1 0 0 1 0 0
1 0 1 1 0 0
1 1 0 1 0 0
1 1 1 1 0 0

(2) 写逻辑式

YA = AB C + ABC + ABC + ABC


= AB(C + C ) + AB(C + C )
= AB + AB
= A(B + B) = A
YB = ABC + ABC = AB
YC = A BC

(3) 画逻辑图(如图9所示)

20.6.13

某 汽 车 驾 驶 员 培 训 班 进 行 结 业 考 试 , 有 三 名 评 判 员 , 其 中A为 主 评 判
员,B和C 为副评判员。在评判时,按照少数服从多数的原则通过,但主评判员
认为合格,亦可通过。试用与非门构成逻辑电路实现此评判规定。
[解]

图 10: 习题20.6.13图

(1) 列逻辑状态表(如表6所示) 当A、B、C 为1时,认为合格;当A、


B、C 为0时,认为不合格。

12
表 6: 逻辑状态表
A B C Y
0 0 0 0
0 0 1 0
0 1 0 0
0 1 1 0
1 0 0 1
1 0 1 1
1 1 0 1
1 1 1 1

(2) 写逻辑式

Y = ABC + AB C + ABC + ABC + ABC


= ABC + AB + AB
= ABC + A
= A + BC
= A · BC

或取Y = 0列逻辑式:

Y = A B C + A BC + ABC
= A B + ABC
= A(B + C )
= A · BC
Y = A · BC

(3) 画逻辑图(如图10所示)

20.6.14

某同学参加四门课程考试,规定如下:
(1)课程A及格得1分,不及格得0分; (2)课
程B及格得2分,不及格得0分; (3)课程C 及
格得4分,不及格得0分; (4)课程D及格得5
分,不及格得0分。
若总得分大于8分(含8分),就可结业。试用与非门实现上述要求的逻辑电路。
[解]

13
图 11: 习题20.6.14图

(1) 列逻辑状态表(如表7所示)
(2) 写逻辑式

Y = A BC D + ABC D + ABC D + ABC D + ABC D


= A BC D + ABC D + ABC D + ABC D + ABC D + ABC D
= AC D + AC D + ABD
= C D + ABD
= C D + ABD
= ABD · C D

(3) 画逻辑图(如图11)

20.7 加法器

20.7.1

十六进制是“逢十六进一”,是以16为底数的计数体制,它有0、1、2、3、
4、5、6、7、8、9、A、B、C 、D、E、F 共十六个数码。试将(7E6AD)16 转换
为十进制数和二进制数。
[解]

(7E6AD)16 = 7 × 164 + 14 × 163 + 6 × 162 + 10 × 161 + 13 × 160


= 458752 + 57344 + 1536 + 160 + 13
= (517805)10
(7E6AD)16 = (0111 1110 0110 1010 1101)2

20.7.2

仿照全加器画出1位二进制数的全减器;输入被减数为A,减数为B,低位
来的借位数为C ,全减差为D,向高位的借位数为C1 。

14
表 7: 逻辑状态表
A B C D 总分 Y
0 0 0 0 0 0
0 0 0 1 5 0
0 0 1 0 4 0
0 0 1 1 9 1
0 1 0 0 2 0
0 1 0 1 7 0
0 1 1 0 6 0
0 1 1 1 11 1
1 0 0 0 1 0
1 0 0 1 6 0
1 0 1 0 5 0
1 0 1 1 10 1
1 1 0 0 3 0
1 1 0 1 8 1
1 1 1 0 7 0
1 1 1 1 12 1

[解]
(1) 列状态表 根据二进制减法规则列出如表8所示的全减器逻辑
状态表。
(2) 写逻辑式

D = A BC + ABC + AB C + ABC
= (AB + AB)C + (AB + A B)C
= (A ⊕ B) · C + (A ⊕ B) · C

= (A ⊕ B) ⊕ C = B ⊕ (A ⊕ C )
C1 = A BC + ABC + ABC + ABC
= AC (B + B) + B(A C + AC )
= AC + B · A ⊕ C

= AC · B · A ⊕ C

(3) 画逻辑图(如图12所示)

15
图 12: 习题20.7.2图

表 8: 逻辑状态表
A B C D C1
0 0 0 0 0
0 0 1 1 1
0 1 0 1 1
0 1 1 0 1
1 0 0 1 0
1 0 1 0 0
1 1 0 0 0
1 1 1 1 1

20.8 编码器

20.8.1

试 设 计 一 个4/2线 二 进 制 编 码 器 , 输 入 信 号 为I 3 ,I 2 ,I 1 ,I 0 , 低 电 位 有
效。
[解]
将4个输入信号编成对应的4个二进制代码输出,输出的应是2位(2n = 4, n =
2)二进制代码Y1 Y0 ,它的4种组合表示4个输入信号。编码表如表9所示。
由编码表写出Y1 和Y0 的逻辑式

Y1 = I 3 + I 2 = I 3 · I 2

Y0 = I 3 + I 1 = I 3 · I 1

16
图 13: 习题20.8.1图

表 9: 编码表
I3 I2 I1 I0 Y1 Y2
1 1 1 0 0 0
1 1 0 1 0 1
1 0 1 1 1 0
0 1 1 1 1 1

由逻辑式画出逻辑电路(如图13所示)。
I 0 的编码是隐含的,当其它输入信号无效时,电路的输出就是I 0 的编码。

20.9 译码器和数字显示

20.9.2

试设计一个能驱动七段LED数码管的译码电路,输入变量A,B,C 来自
计 数 器 , 按 顺 序000 ∼ 111计 数 。 当ABC = 000时 , 全 灭 ; 以 后 要 求 依 次 显
示H 、O、P 、E、F 、U 、L七个字母。
[解]

(1) 根据要求列出状态表(如表10所示)

17
图 14: 习题20.9.2图

表 10: 状态表
输入 输出
A B C a b c d e f g
0 0 0 0 0 0 0 0 0 0
0 0 1 0 1 1 0 1 1 1
0 1 0 1 1 1 1 1 0 0
0 1 1 1 1 0 0 1 1 1
1 0 0 1 0 0 1 1 1 1
1 0 1 1 0 0 0 1 1 1
1 1 0 0 1 1 1 1 1 0
1 1 1 0 0 0 1 1 1 0

18
(2) 由状态表写出各个输出变量的逻辑式

a = ABC + ABC + AB C + ABC


= AB + AB = AB · AB
b = A BC + ABC + ABC + ABC
= AC + BC = AC · BC
c = A BC + ABC + ABC
= A BC + BC = A BC · BC
d = ABC + AB C + ABC + ABC
= ABC + AB C + AB = ABC + A(B + C )
= ABC + AB + AC = B(A + C ) + AC
= AB + AC + BC = AB · AC · BC
e = f =A B C
g = A BC + ABC + AB C + ABC
= AC + AB = AC · AB

(3) 由逻辑式画出逻辑电路(如图14)

20.9.4

试用74LS138型译码器实现Y = A B C + ABC + AB的逻辑函数。


[解]
将逻辑式用最小项表示

Y = A B C + ABC + AB
= A B C + ABC + ABC + ABC

由教材表20.9.1得出

Y0 = A B C Y3 = ABC
Y6 = ABC Y7 = ABC

因此得出

Y = Y0 + Y3 + Y6 + Y7 = Y0 · Y3 · Y6 · Y7

用74LS138型译码器实现上式的逻辑图如图15所示。

19
图 15: 习题20.9.4图

20.9.5

试 设 计 一 个 用74LS138型 译 码 器 监 测 信 号 灯 工 作 状 态 的 电 路 。 信 号 灯 有
红(A)、黄(B)、绿(C )三种,正常工作时只能是红、绿、红黄、绿黄灯亮,其他
情况视为故障,电路报警,报警输出为1。
[解]
(1) 按题意列出状态表(如表11所示)

表 11: 逻辑状态表
A B C Y
0 0 0 1
0 0 1 0
0 1 0 1
0 1 1 0
1 0 0 0
1 0 1 1
1 1 0 0
1 1 1 1

(2) 由状态表写出逻辑式

Y = A B C + ABC + ABC + ABC

20
由教材表20.9.1得出

Y0 = A B C Y2 = ABC
Y5 = ABC Y7 = ABC

由此得出

Y = Y0 + Y2 + Y5 + Y7 = Y0 · Y2 · Y5 · Y7

(3) 用74LS138型译码器实现监测信号灯的电路可参照上题的图15
画出。

21
22
目录
第21章 触发器和时序逻辑电路 4
第21.1节 双稳态触发器 . . . . . . . . . . . . . . . . . . . . . . . . . . 4
第21.1.7题 . . . . . . . . . . . . . . . . . . . . . . . . . . . . . . 4第
21.1.8题 . . . . . . . . . . . . . . . . . . . . . . . . . . . . . . 4第
21.1.9题 . . . . . . . . . . . . . . . . . . . . . . . . . . . . . . 5第
21.1.10题 . . . . . . . . . . . . . . . . . . . . . . . . . . . . . 5第
21.1.11题 . . . . . . . . . . . . . . . . . . . . . . . . . . . . . 7第
21.2节 寄存器 . . . . . . . . . . . . . . . . . . . . . . . . . . . . . . 8
第21.2.1题 . . . . . . . . . . . . . . . . . . . . . . . . . . . . . . 8
第21.3节 计数器 . . . . . . . . . . . . . . . . . . . . . . . . . . . . . . 8
第21.3.1题 . . . . . . . . . . . . . . . . . . . . . . . . . . . . . . 8
第21.3.4题 . . . . . . . . . . . . . . . . . . . . . . . . . . . . . . 11
第21.3.5题 . . . . . . . . . . . . . . . . . . . . . . . . . . . . . . 11
第21.3.6题 . . . . . . . . . . . . . . . . . . . . . . . . . . . . . . 12
第21.3.8题 . . . . . . . . . . . . . . . . . . . . . . . . . . . . . . 12
第21.3.9题 . . . . . . . . . . . . . . . . . . . . . . . . . . . . . . 13
第21.6节 应用举例 . . . . . . . . . . . . . . . . . . . . . . . . . . . . . 13
第21.6.1题 . . . . . . . . . . . . . . . . . . . . . . . . . . . . . . 13
第21.6.3题 . . . . . . . . . . . . . . . . . . . . . . . . . . . . . . 15
第21.6.4题 . . . . . . . . . . . . . . . . . . . . . . . . . . . . . . 16
第21.6.5题 . . . . . . . . . . . . . . . . . . . . . . . . . . . . . . 16

1
List of Tables
1 状态表 . . . . . . . . . . . . . . . . . . . . . . . . . . . . . . . . . . . 5
2 移位(右移)状态表 . . . . . . . . . . . . . . . . . . . . . . . . . . . . . 9
3 4位二进制减法计算器的状态表 . . . . . . . . . . . . . . . . . . . . . 10
4 状态表 . . . . . . . . . . . . . . . . . . . . . . . . . . . . . . . . . . . 12
5 状态表 . . . . . . . . . . . . . . . . . . . . . . . . . . . . . . . . . . . 13
6 状态表 . . . . . . . . . . . . . . . . . . . . . . . . . . . . . . . . . . . 14
7 六拍通电环形分配器的状态表 . . . . . . . . . . . . . . . . . . . . . . 15
8 状态表 . . . . . . . . . . . . . . . . . . . . . . . . . . . . . . . . . . . 17

2
List of Figures
1 习题21.1.7图 . . . . . . . . . . . . . . . . . . . . . . . . . . . . . . . . 4
2 习题21.1.8图 . . . . . . . . . . . . . . . . . . . . . . . . . . . . . . . . 4
3 习题21.1.9图 . . . . . . . . . . . . . . . . . . . . . . . . . . . . . . . . 5
4 习题21.1.9图 . . . . . . . . . . . . . . . . . . . . . . . . . . . . . . . . 6
5 习题21.1.10图 . . . . . . . . . . . . . . . . . . . . . . . . . . . . . . . 6
6 习题21.1.10图 . . . . . . . . . . . . . . . . . . . . . . . . . . . . . . . 6
7 习题21.1.11图 . . . . . . . . . . . . . . . . . . . . . . . . . . . . . . . 7
8 习题21.1.11图 . . . . . . . . . . . . . . . . . . . . . . . . . . . . . . . 7
9 习题21.1.11图 . . . . . . . . . . . . . . . . . . . . . . . . . . . . . . . 8
10 习题21.2.1图 . . . . . . . . . . . . . . . . . . . . . . . . . . . . . . . 8
11 习题21.3.1图 . . . . . . . . . . . . . . . . . . . . . . . . . . . . . . . 9
12 习题21.3.4图 . . . . . . . . . . . . . . . . . . . . . . . . . . . . . . . 11
13 习题21.3.5图 . . . . . . . . . . . . . . . . . . . . . . . . . . . . . . . 11
14 习题21.3.6图 . . . . . . . . . . . . . . . . . . . . . . . . . . . . . . . 12
15 习题21.3.8图 . . . . . . . . . . . . . . . . . . . . . . . . . . . . . . . 13
16 习题21.3.9图 . . . . . . . . . . . . . . . . . . . . . . . . . . . . . . . 14
17 习题21.6.1图 . . . . . . . . . . . . . . . . . . . . . . . . . . . . . . . 15
18 习题21.6.3图 . . . . . . . . . . . . . . . . . . . . . . . . . . . . . . . 16
19 习题21.6.4图 . . . . . . . . . . . . . . . . . . . . . . . . . . . . . . . 16
20 习题21.6.4图 . . . . . . . . . . . . . . . . . . . . . . . . . . . . . . . 17
21 习题21.6.5图 . . . . . . . . . . . . . . . . . . . . . . . . . . . . . . . 18

3
21 触发器和时序逻辑电路
21.1 双稳态触发器

21.1.7

根据图1(a)的逻辑图及图1(b)所示相应的C P ,RD 和D的波形,试画出Q1 端


和Q2 端的输出波形,设初始状态Q1 = Q2 = 0。
[解] Q1 和Q2 的波形如图1(b)所示。

图 1: 习题21.1.7图

21.1.8

电 路 如 图2(a)所 示 , 试 画 出Q1 和Q2 的 波 形 。 设 两 个 触 发 器 的 初 始 状 态 均


为0。
[解] J K 触发器在下降沿触发,D触发器在上升沿触发。也可先列出状态表

图 2: 习题21.1.8图

(如表1所示),而后由此在时钟脉冲C P 的上升沿和下降沿处画出Q1 和Q2 的波

4
形。如图2(b)所示。 来两个时钟脉冲循环一次,输出
的是正交波形。

表 1: 状态表
时钟脉冲数 CP J1 = Q2 K1 = 1 D = Q1 Q1 Q2
0 0 0 1 1 0 0
1 0→1 1 1 1 0 1
1→0 1 1 0 1 1
2 0→1 0 1 0 1 0
1→0 0 1 1 0 0

21.1.9

图3所示电路是一个可以产生几种脉冲波形的信号发生器。试从所给时钟脉
冲C P 画出Y1 、Y2 、Y3 三个输出端的波形。设触发器的初始状态为0。
[解]

图 3: 习题21.1.9图

Y1 = Q Y2 = C P · Q Y3 = C P · Q

因J = Q,K = Q,故 可 先 画 出Q( 即Y1 ) 的 波 形 。 而 后 由Y2 = C P · Q,Y3 =


C P · Q画出Y2 和Y3 的波形(如图4所示)。

21.1.10

试分析图5所示的电路,画出Y1 和Y2 的波形,并与时钟脉冲C P 比较,说明


电路的功能。设初始状态为Q = 0。
[解] 由J = Q和K = Q可画出Q和Q的波形。再由输出逻辑式

Y1 = C P + Q Y2 = C P + Q

5
图 4: 习题21.1.9图

图 5: 习题21.1.10图

图 6: 习题21.1.10图

6
画出Y1 和Y2 的波形,如图6所示。
从波形图上看,时钟脉冲C P 经过图5所示的逻辑电路后变为两个不同相的
脉冲,所以上述电路称为双相时钟脉冲发生器。

21.1.11

图7是一单脉冲输出电路,试用一片74LS112型双下降沿J K 触发器(其外引
线排列见教材习题21.1.11)和一片74LS00型四2输入与非门[见教材图20.3.3(b)]连
接该电路,画出接线图,并画出C P 、Q1 、Q2 、Y 的波形图。
[解] 设两触发器的初始状态均为0。接线图和波形图分别如图8和图9所

图 7: 习题21.1.11图

图 8: 习题21.1.11图

示。

7
图 9: 习题21.1.11图

21.2 寄存器

21.2.1

试用D触发器组成4位移位寄存器。
[解] 逻辑图和移位状态如图10和表2所示。设输入数据为1101。

图 10: 习题21.2.1图

21.3 计数器

21.3.1

教材图21.3.1是由主从型J K 触发器组成的4位二进制加法计数器。试改变级
间的连接方法,画出也是由该触发器组成的4位二进制减法计数器,并列出其状
态表。在工作之初先清零,使各个触发器的输出端Q0 ∼ Q3 均为0。
[解] 4位二进制减法计数器的逻辑图和状态表分别如图11和表3所示。

8
表 2: 移位(右移)状态表
移位脉冲数 寄存器中的数码
Q3 Q2 Q1 Q0
0 0 0 0 0
1 1 0 0 0
2 0 1 0 0
3 1 0 1 0
4 1 1 0 1
5 0 1 1 0
6 0 0 1 1
7 0 0 0 1
8 0 0 0 0

图 11: 习题21.3.1图

9
表 3: 4位二进制减法计算器的状态表
计数脉冲数 二进制数
Q3 Q2 Q1 Q0 十进制数
0 0 0 0 0 0
1 1 1 1 1 15
2 1 1 1 0 14
3 1 1 0 1 13
4 1 1 0 0 12
5 1 0 1 1 11
6 1 0 1 0 10
7 1 0 0 1 9
8 1 0 0 0 8
9 0 1 1 1 7
10 0 1 1 0 6
11 0 1 0 1 5
12 0 1 0 0 4
13 0 0 1 1 3
14 0 0 1 0 2
15 0 0 0 1 1
16 0 0 0 0 0

10
21.3.4

试用74LS161型同步二进制计数器接成十二进制计数器:(1)用清零法;(2)用
置数法。
[解] 用清零法和置数法将74LS161型计数器接成十二进制计数器的逻辑图分

图 12: 习题21.3.4图

别如图12(a)和(b)所示。

21.3.5

试用两片74LS290型计数器接成二十四进制计数器。
[解] 二十四进制计数器的接线图如图13所示。两片74LS290均按8421码十进

图 13: 习题21.3.5图

制计数方式连接,其中片(1)为个位,片(2)为十位。计数脉冲由片(1)的C P0 端
输 入 , 片(2)的 计 数 脉 冲 由 片(1)的 最 高 位Q3 输 出 提 供 。 当 片(1)输 入 第 十 个
脉 冲 时 ,Q3 Q2 Q1 Q0 由1001回 到0000,Q3 由1变 为0。 此 下 降 沿 使 片(2)由0000变
为0001。 当 片(1)输 入 第 二 十 个 脉 冲 时 , 片(2)变 为0010。 再 输 入 四 个 脉 冲 ,

11
片(1)的状态为0100。片(2)的Q1 和片(1)的Q2 均为1,立即反馈置0,从而完成一
个计数循环。

21.3.6

试列出图14所示计数器的状态表,从而说明它是一个几进制计数器。
[解] 图14所示计数器的状态表如表4所示,它是一个七进制计数器。设初始

图 14: 习题21.3.6图

状态均为0。
注意,当Q1 由1变为0时,触发器F F2 才翻转。

表 4: 状态表

计数脉冲数 J2 = 1 K2 = 1 J1 = Q0 K1 = Q2 Q0 J0 = Q2 Q1 K0 = 1 Q2 Q1 Q0
0 1 1 0 0 1 1 0 0 0
1 1 1 1 1 1 1 0 0 1
2 1 1 0 0 1 1 0 1 0
3 1 1 1 1 1 1 0 1 1
4 1 1 0 1 1 1 1 0 0
5 1 1 1 1 1 1 1 0 1
6 1 1 0 1 0 1 1 1 0
7 1 1 0 0 1 1 0 0 0

21.3.8

逻辑电路如图15所示。设QA = 1,红灯亮;QB = 1,绿灯亮;QC = 1,

12
黄灯亮。试分析该电路说明三组彩灯点亮的顺序。在初始状态,三个触发器
的Q端均为0。此电路可用于晚会对彩灯的控制。
[解] 图15所示逻辑电路的状态表如表5所示,由此可知,三组彩灯点亮的顺

图 15: 习题21.3.8图

序为:红灯亮→ 绿灯亮→ 黄灯亮→ 全亮→ 全灭→ 红灯亮→ . . . 。

表 5: 状态表

CP JA = QB KA = 1 JB = QA + QC KB = 1 JC = QB KC = QA QA QB QC
0 1 1 0 1 0 0 0 0 0
1 1 1 1 1 0 1 1 0 0
2 0 1 0 1 1 0 0 1 0
3 1 1 1 1 0 0 0 0 1
4 0 1 1 1 1 1 1 1 1
5 1 1 0 1 0 0 0 0 0

21.3.9

分析图16所示的逻辑电路,说明发光二极管做亮3s、暗2s的循环。
[解] 设 初 始 状 态 为000。 图16所 示 逻 辑 电 路 的 状 态 表 如 表6所 示 , 由 此 可
知,发光二极管做亮3s、暗2s的循环。

21.6 应用举例

21.6.1

图17是步进电机六拍通电方式的环形分配器的逻辑电路,请分析之。

13
图 16: 习题21.3.9图

表 6: 状态表
CP DC = QA QC + QB DB = QC DA = QA QB QC QB QA
0 1 0 0 0 0 0
1 1 1 0 1 0 0
2 1 1 1 1 1 0
3 0 1 0 1 1 1
4 0 0 1 0 1 0
5 1 0 0 0 0 1
6 1 1 0 1 0 0

14
[解] 先 在 复 位 、 置 位 端 加 负 脉 冲 , 使 初 态QA QB QC = 100, 而 后 输 入 脉

图 17: 习题21.6.1图

冲,按J K 触发器的逻辑功能逐步分析,得出如表7所示的状态表。由表可看出
是步进电机六拍通电方式:
U1 → U1 V1 → V1 → V1 W1 → W1 → W1 U1 → U1 . . .

表 7: 六拍通电环形分配器的状态表
脉冲数 JA KA JB KB JC KC U1 V1 W1
0 1 0 1 0 0 1 1 0 0
1 0 1 1 0 0 1 1 1 0
2 0 1 1 0 1 0 0 1 0
3 0 1 0 1 1 0 0 1 1
4 1 0 0 1 1 0 0 0 1
5 1 0 0 1 0 1 1 0 1
6 1 0 1 0 0 1 1 0 0

21.6.3

试设计一个三人抢答逻辑电路,要求: (1)每位参赛者有一个按钮,按下就
发出抢答信号; (2)主持人另有一个按钮,按下电路复位; (3)先按下按钮者将
相应的一个发光二极管点亮,此后,他人再按下各自的按 钮,电路不起作
用。 (建议:其中一种三人抢答电路可用两片74LS00组成的三个基本RS触发器
和由 两片74LS20组成的三个与非门来实现)

15
图 18: 习题21.6.3图

[解] 根据题中建议所实现的三人抢答逻辑电路如图18所示,关键是根据要
求(3)如何连接三个与非门。

21.6.4

试设计一个由两个T 触发器组成的逻辑电路,能实现三个彩灯A、B、C 作
图19所示的顺序点亮。
[解] 所设计的逻辑电路如图20所示。表8是状态表。先清零,三灯全暗。

图 19: 习题21.6.4图

21.6.5

试用由与非门组成的RS触发器并用起动按钮SB2 和停止按钮SB1 来控制电


动机的起停。

16
图 20: 习题21.6.4图

表 8: 状态表
CP Q1 Q2 A B C
0 0 0 • • •
1 1 0 • ◦ •
2 0 1 ◦ • ◦
3 1 1 ◦ ◦ ◦
4 0 0 • • •

17
[解] 逻辑电路如图21所示。

图 21: 习题21.6.5图

18
19

You might also like